第二版 数理化自学丛书 代数 第三册 余元希编 上海科学技术出版社 ==========第1页========== 数理化目学丛书 第二版 代 数 107 第三册 余元希编 上海科学技术出版社 ==========第2页========== 内容提要 本书是数理化自学丛书中代数第三册,内容包括集合的初步知识,不等式,函数,指数和对数,数列,数学归纳法等九章,书中有详细的说明和分析,并附有大量例题和习题、复习题、测验题,供练习巩固之用. 本书供学完本丛书代数一、二册的青年工人、在职干部、知识背年自学之用,也可供中等学校青年教师参考。 数理化自学少书第二版 代数 第三册 余元希编数理化自学丛书编委会审定上海科学技术出版社出版 (上海瑞金二路450亭) 业专专上海发行所发行上海商务印厂广印刷 开本787×10921/32印张14.3学数385.000 1963年12月第】版 1982年11月第2版1982年11月第9次印删 印数:588,00180,500 统一书号:18119.544定介:(科二)1.00元 ==========第3页========== 第二版出版说明 《数理化自学丛书》第二版是在第一:的基础上编写而成的.考虑到我社已出版大学数、理、化自学丛书,中学数学中的微积分内容没有另编分册.第二版仍包括代数》四册、《平面儿何》两册、《平面三角》、《立体儿何》、《平面解析儿何》、《物理》四册和《化学》四册,共十七册 由黄丹鞭、杨荣祥、余元希、杨逢闼桂君协等同志主编的第一版,自1963年陆续出版后,受到广大读者的欢迎.特别是1977年重排、重印以来,受到杜会:各方面极为广泛的关注,在广大读者中有了相当的影响。许多在职职工、农村青年和在校学生,自学了这套书以后,理化知识水平有了一定的提高. 第二版由杨荣样、余元希、束世杰、季文德等同志主编,数理化自学丛书编委会审定.它保留了第一版在编写上“详尽在先、概括在后、通俗到底”和“便于自学:、无师自通”的特色,仍是一套与现行中学课本并行的自学读物。第二版仍从读者的实际情况出发,按传统的教学体系编写。但这次参照新的试行教学大纲的要求,与第一版相比,数学各分猎的编写内容作了适当的增删和调整,基础知识和运算技能的训练有了进一步加强;物理各分册在内容的取舍、习慧的更新、插图的选配、实验的描述等方面均有较大的改进,化学各分册还增加了反映现代科学技术水平的基础理论知识,在理论和实践相结合的原则下,内容体系均有新的特色.此外,各册的例题和习题选配得力求恰当、合理,知识 ==========第4页========== 论述力求通俗、严密;并按章增加了测验题。在各册编者的话中,还有供读者自学时参考的指导性意见 自学要有成就,必须刻苦勤奋、踏实认真、持之以恒、知难而进。刻苦自学、学有成就者不乏其人,愿广大读者努力学好. 《数理化自学丛书》出版以来,全国各地的读者给以热情的鼓励和有力的支持,特在此表示衷心感谢 上海科学技术出版社 ==========第5页========== 编者的话 本书是《数理化自学丛书》中的代数第三册,内容主要是对一些常见的初等函数(不包括三角函数和反三角函数)作系统的研究,并把数列作为一种特殊的函数,引进了关于数列、数列极限的知识.由于研究函数的需要,在引进函数概念之前,系统地研究了各种代数不等式的解法以及不等式的证明,所以在内容安排与处理方法上与全日制中学数学课本不尽相同,但是具有初中二年级水平以上的读者,自学本书将不感到困难 本书第二版在修改中,除了保持了第一版原有的一些特点以外,许多地方都有较大的变动,主要是: 1.在中学数学教学改革中,由于集合这一概念已渗透到中学数学教材中,因此本书在开头就介绍了关于集合的初步知识,并把引进的一些术语、符号在以后各章中加以应用。 2。对于函数这一概念,本书中仍采用“y是心的函数”这一习惯上使用的提法,但是在讲有理数指数的幂函数之前,也对这一概念应用集合的观点作了阐述 3.在第一版中“常用对数”是安排在学过“指数函数和对数函数”之后才引进的,现在把它提到了研究这两种函数之前.这样处理,目的是使之与现行中学数学课本比较接近。 4.考虑到结合数列来学习数学归纳法,读者比较容易接受,因此把原来安排在代数第四册里的这一部分内容,移到了本书中, 6,在不等式、有理数指数的幂函数、数列等各章中都 ==========第6页========== 增加了一些新的内容 6.把各章复习题、总复习题分成了A、B两组,并增 加了各章的测验题和总测验题,所以在习题的数量上比过去有所增加 读者自学本书,除了可以按照全书的安排顺序以外,也可根据实际情况作-一些调整.例如在学习了第一章集合的初步知识以及第二章巾一元一次不等式组的解法之后,繁接着就可学习第三章、第四章、第六章;书中有些打有号的内容,可暂时不学.书中所附的习题也可选做其中的一部分. 在这次修改中,本丛书编委杨荣样、赵宪初同志曾提了不少有益的意见,王抒同志通读了全书,并代我核验了全部习题的答案,这里谨向他们表示衷心的谢意.在修改中,编者虽然作了一定的努力,但是限于水平,难免还存有不少峡点甚至错误,希望读者给以指正 编者 1981年3月 ==========第7页========== 目 录 第二版出版说明 400…1 编者的话ii 1。集合的初步知识4●001 §11集合的意义和表示法…1§12有限集合、无限集合、空集合……5§13子集、真子集、集合的相等 …7 §14交集、并集 …11 §15全集、补集 …14 本章提要… …18 复习题一A………19 复习题一B…,…20 第一章测验题………21 2。不等式 ……022 82…1不等式的概念…22 82.2不等式的基本性质…27§23一元不等式的解和解集…30豸24一元一次不等式组33825区间………42826一元二次不等式…45*827一元二次不等式组…54*82.8一元高次不等式 56 多29一元分式不等式 …59 210不等式的其他一些性质 …64 §2,11一元无理不等式… …72 §2.12不等式的证明… 75 “8213代数式的最大值和最小值…828214含有绝对值的不等式…87 ==========第8页========== 本章提要 …97 复习题二A …102 复习题二B …104 第二章测验题 …105 3.函数的初步知识… …107 §31常量和变量… …107 83,2函教… …110 §3.3平面上的直角坐标系 …118 834函数关系的表示法… …124 83.5正比例函数 0128 &3.6反比例函数 …139 本章提要 …144 复习题三A……146 复习题三B…148 第三章测验题 …149 4。一次函数和二次函数…… …150 §4.1一次函数的图象和性质…150 84.2根据已知条件确定一次函数…157 84.3二元一次方程的图象……159844二次函数的图象和性质… …167 84.5二次函数图象的作法 …176 §4.6根据已知条件确定二次函数 …179 847二次函数的极值… …182 §48二次函数图象的应用 …189 本章提要… …193 复习题四A 4…195 复习题四B …198 第四章测验题 0*…200 5。有理数指数的幂函数……202851对应和函数……202 ==========第9页========== §5,2函数的一些重要性质4…212853整数指数的幂函数、有理函数……219§54分数指数的幂函数、无理函数…224§55反函数… …230 856单值函数和多值函数…237本章提要……240 复习题五A……243 复习题五B…245 第五章测验题 ……246 6.指数和对数 …247 §6,1幂的概念的扩展……247 86.2对数 …251 §63关于对数的定理 …255 86•4常用对数 …261 §65对数表… …267 §66常用对数的求法… …270 §6.7反对数表 …278 868利用对数进行计算…280§69对数的换底公式…283本章提要…287 复习题六A …288 复习题六B …291 第六章测验题… …292 7.指数函数和对数函数……29明 §7,1指数函数……29387…2对数函数…304§73指数方程…310§7●4对数方程… …315 *§75指数方程和对数方程的图象解法…32087,6指数和对数方程组…322多77指数和对数不等式…325 。yig ==========第10页========== 本章提要328 复习题七329 复习题七B… …331 第七章测验题 …332 8。数列、数学归纳法… …833 §81数列333§8,2等差数列…342§83等比数列… …351 §84等差中项和等比中项 …360 88.5数学归纳法 …366 §86数列前n项的和 379 本章提要 …386 复习题八A…… …388 复习题八B …390 第八章测验题 …391 9.*数列的极限…393§91数列极限的意义…14…393§9.2关于数列极限的一些定理…401§93无穷递缩等比数列…405 89.4化循环小数为分数…409§95无穷数列各项的和4413本章提要…414 复习题九A…416 复习题九B 4…418 第九章测验题 419 总复习题A… …420 总复习题B… …426 总测验题 ……431 习题答案 …438 ●vi● .t:i ==========第11页========== 1 集合的初步知识 集合是数学里一个重要的基本概念。这一章里,我们将学习集合的一些初步知识,包括集合的意义,集合的表示法,以及集合的一些常用术语和符号,这些都是学习下面各章内容的基础. §1·1集合的意义和表示法 1.集合的意义 什么叫做集合?粗略地来回答,所谓“集合”,也就是我们通常所说的“全体”或者“整体”.例如 ()右手掌上五个手指:大拇指、食指、中指、无名指和小指; (2)五个英文元音字母:a、e、1、0、u (3)车间里所有的工人; (4)所有的自然数:1,2,3,…,%, (5)和线段AB的两个端点等距离的所有的点; (⑥)一切大小不同的等边三角形 等等;当我们分别把它们看成是一个整体来考察时,我们就说这些对象(手指、字母、工人、数、点、图形)分别组成了一个集合,并把它们分别叫做 (I)右手掌上手指的集合; (2)英文元音字母的集合; (3)车间里工人的集合; ==========第12页========== (4)自然数的集合; (5)和线段AB的两个端点等距离的点的集合; (⑥)等边三角形的集合. 在每个集合里的各个对象,叫做组成这个集合的元素 一般地说:当我们把具有某种共同属性的对象看成是 一个整体来考察,就说这些对象组成了一个集合,集合里的各个对象叫做集合中的元素, 有时,我们也把集合简单地叫做集。例如,由一切自然数组成的集合,简称自然数集.自然数1,2,3,…都是自然数集中的元素、 当我们一般地研究某一个集合时,常用大写英文字母 A、B、…表示集合,小写英文字母a,b,…表示集合中的元素.如果a是集合A中的元素,就说:“a属于A”,并把它表示为 a∈A. 如果b不是集合A中的元素,就说:b不属于A”,并把它表示为 b年A(或bEA). 这里符号∈读做“属于”,符号丰(或)读做“不属于”。 例如,用符号W表示自然数集,那末 3∈N,중 应该注意:数学里所称的集合,它的元素必须是确定的.也就是说,只有当我们能够判断出任何考察的对象是不是这个集合中的元素时,才能说确定了一个集合.例如,我们能够判断出一个数是自然数或者不是自然数,所以我们可以说一切自然数组成了自然数集;但是我们无法判断出怎样的数是大数或者不是大数,所以我们就不能说所有 ==========第13页========== 的大数组成了“大数的集合”。 2.集合的表示法 对于一个具体的集合,常用的表示法有两种: (1)列举法.把集合中的元素一一列举出来,写在大括号内,用来表示集合的方法叫做列举法。例如: 1°英文字母中元音字母的集合,可以记做 {a,e,元,o,u; 2°比10小的质数的集合,可以记做{2,3,5,7);3°方程2一1=0的根的集合,可以记做1,一1)当集合里的元素很多时,只要我们能够判断出它指的是什么,也可只写出其中的几个元素,而把其它的元素用省略号来代替.例如: 比100小的自然数的集合,可以记作{1,2,3,,99};自然数的集合可以记做{1,2,8,…,见,…小. 应用列举法来表示集合时,要注意两点: 1°集合里元素出现的先后顺序是不考虑的,只要元素完全相同,就认为是同一个集合.例如{1,一1和{一1,1}表示同一个集合 2°一个元素在集合里不能重复出现. (②)描述法.表示一个集合,也常常采用把集合中元素的公共属性或约束条件在大括号里写出的方法,这种方法叫做描述法.例如: 1°由所有等边三角形组成的集合,可以表示为 {等边三角形}; 2°由所有比10小的质数组成的集合{2,3,5,7},可以表示为 {比10小的质数}: 3°一切正数组成的集合,可以表示为 ==========第14页========== {xx>0}.① 在这一表示法中,竖线前面的表示这个集合中的元素,后面那个不等式心>0,就表示这种元素所受到的约束条件、例1 把下面这两个集合,分别用列举法表示出来: (1){20以内的正偶数}; (2){xx2-3+1=0}. [解] (1)这个集合就是:{2,4,6,8,10,12,14,16,18} (②)因为方程-3c+1-0有两个根1,a=3士√5 2 所以这个架合诚起②+Y百,8-Y 例2 如果用字母A表示由方程组 「c3+y2=25, x-y=1 的解所组成的集合 {(,y)x2+y2=25且x-y=1}, 判断下列各数对是不是属于这个集合,用符号∈或者庄表示出来 (1)(4,3): (2)(3,4): (3)(-4,-3); (4)(-3,-4). [解] 解这个方程组,得到它有两个解: 4,和{-3, g=3 =-4。 所以 (1)(4,3)∈A; (2)(3,4)丰A; (3)(-4,-3)A;(4)(-3,-4)∈A ①有的书上,也把它写成{必x>0}。 ● ==========第15页========== 1,用列举法写出由下列各题中的事物所组成的集合: 11 (1)比20小的质数; (2)绝对值小于10的奇数 (3)16和24的公约数; (4)540的质因数; (5)方程x4-62+8=0的根; (6)多项式3-62+9x一4的一次因式.2写出下面各集合中的元素: (1){x-32}; (2){12,16的公约数}; (3){12,16的公倍数}. ==========第16页========== [解] (1)由-3>2可知>5,这个集合是由一切比5大的实数组成的,它是一个无限集. (2)12,16的公约数只有1,2,4等3个,这个集合就是{1,2,4},它是一个有限集. (3)12,16的最小公倍数是48,一-切能被48整除的自然数都是这个集合中的元素,所以它是一个无限集, 2.单元素集合和空集合 现在我们来考察下面这两个集合: (1){xx-3=0};(2){xx2+1=0}. 在(1)中,因为方程x一3=0只有一个根x=3,所以这个集合中只含有一个元素3.象这类只含有一个元素的有,限集,叫做单元素集合.一般地,以为元素的单元素集合,可以记做{a}. 应该注意a与{a}的意义是不同的.表示元素,而{a}表示以a为元素的集合,它们间的关系是 a∈{a. 在(2)中,因为在实数范围里,方程x2+1=0是无解的,也就是说,没有一个实数花,能够满足条件x2+1=0,所以在{xx2+1=0)里,实际上并不包含任何元素.为了讲法上的方便,我们也把它看做是一个集合.象这种不含有任何元素的集合,叫做空集合(简称空集).空集一般可用符号来表示.与之区别,我们把至少含有一个元素的集合,叫做非空集合(或非空集).例2 说明符号0,{0},0各有什么意义.并用符号∈或者庄表示出0与0)以及0与0间的关系[解] 0表示一个元素,{0}表示以0为元素的单元素集合,0表示不含有任何元素的空集. 0∈{0},0年0. ==========第17页========== 1.判断下面各集合,哪些是有限集,哪些是无限集,如果是有限集 习 题 12 把它改用另一种方法表示出来 (1){x0≤x<3}; (2){x0sx<3,且x为整数}; (3){ax2+x+1=0}; (4){x3-1=0} 2.如果把二元一次方程+by+c=0的解的集合,用符号{(x,y)|ax+by十c=0}来表示,判断下面的集合是有限集还是无限集,如果是有限集,把它改用列举法表示出来 (1){(,y)12x+3y=5}; (2){(c,y)12x+3y=5,且x,y为正整数}. §13子集、真子集、集合的相等 我们来考察下面这两个集合 {1,2,3,4,5,{1,3,5 和集合 {5,4,3,2,1 之间的关系。 很明显,集合{1,2,8,4,5}和集合{1,3,5]中的每 一个元素,都是集合{5,4,3,2,1}中的元素;反过来,集合{5,4,3,2,1的每一个元素,也都是集合{1,2,3,4,中的元素,但却不都是集合{1,3,5}中的元素(例如4就不是集合{1,3,5中的元素). 为了描述两个集合之间的这几种相互关系,下面我们引进儿个重要的概念, 1.子集 对于两个集合A和B,如果集合A的任何一个元素都 是集合B的元素,集合A就叫做集合B的子集.集合A是 集合B的子集,表示为 A드B,或BA ==========第18页========== 读做“A包含于B”或“B包含A”. 例如:{1,2,3,4,5}是{5,4,3,2,1的子集,可以记作 {1, 2, 3,4, 5}드{5, 4, 3,2,1}, 或 {5,4,3,2,12{1,2,3,4,5}:{1,3,5是5,4,3,2,1}的子集,可以记作 {1, 3, 5}드{5, 4, 3, 2, 1}, 或 {5,4,3,2,1}2{1,3,5}. 根据上面所说的子集的意义,容易知道:要证明一个集合是另一个集合的子集,只要证明这个集合中的任一个元素都属于另一个集合. 例如,对于任何一个非空集合A,因为它的任何一个 元素都属于集合A,所以 A드A. 也就是说,任何一个集合就是它本身的子集 另外,空集0是不含有任何元素的集合,所以它可以看成是任何一个集合的子集, Ø드A. 例1 证明如果A드B,BC.那末AC. [证] 设c是集合A的任意一个元素,x∈A. 为A드B,以∈B. 因B드C,以EO. 既然集合A的任意一个元素都属于集合0,从而可知 A드C 2,集合的相等 前面我们己经看到: {1,2,3,4,5已{5,4,3,2,1, 而 5,4,3,2,1}{1,2,3,4,5}. 事实上,这两个集合是由相同的元素所组成的,只是元素排 ==========第19页========== 列的位置不同而已,象这样的两个集合,我们把它称做是相等的,并且也用“=”号来表示它们之间的这种关系,记作 {1,2,3,4,5}={5,4,3,2,1. 一般地,如果A三B,同时B三A,那末就称集合A与 集合B相等,记作 A=B, 读做“集合A等于集合B”. 3.真子集 前面我们还曾看到: {1,3,5}二{5,4,3,2,1}、 但是,因为{5,4,3,2,1}中的元素不都属于{1,3,5.例如2庄{1,3,5].所以{5,4,3,2,1}不是{1,3,5}的子集.描述两个集合间的这种关系,我们把{1,3,5}叫做{5,4,3,2,1}的真子集,并且把它记作 {1,3,5}{5,4,3,2,1}. 一般地,如果A是B的子集,并且B中至少有一个元 素不属于A,那末集合A就叫做集合B的真子集,记作 ACB或 BつA. 读做“A包含于B,但不等于B”,或者“B包含A,但不 等于A”. 例如,到现在为止,我们已经学过的数的集合,主要有自然数集、整数集、有理数集和实数集,它们是一个被一个包含着的,如果设 N={自然数},J={整数}, Q={有理数},R={实数}. 它们间的关系,就可以表示为 NCJCQCR. 〔注意〕 本书中今后提到这些数集,规定就用这些大写字母来 ==========第20页========== 分别表示。在有些书上,也常用大写学母Z来表示整数 集 集合间的这种包含关系,也可 以用图1·1中两个圆A和B间的 位置关系来说明,这里圆A表示集 合A,圆B表示集合B,这个图就清 楚地说明了集合A的元素都是集合 B的元素,即ACB 图1.1 这种用封闭曲线(一般用圆)来表示集合的方法,叫做集合的文氏图表示法.利用它可以帮助我们理解集合的意义. 例2 已知有两个集合: (1)A={cx=a+b,且a,b∈Q}; (2)B={gy=心,且c∈②}. 分别说明集合A、B与有理数集Q间的关系,并写出关系 式. [解] (①)因为任意两个有理数心,b的和都是有理数,都属 于集合Q,所以A二. 反之,任意一个有理数”都可以表示成两个有理数的 和,都属于集合A,所以Q二A, 由此可知,A与Q是相等集合,即A=Q. (②)因为任意一个有理数的平方,也是有理数,都属于 Q,所以B드Q 但是,并不是所有的有理数都属于集合B,例如一1 庄B. 由此可知,B是Q的真子集,即BCQ. 〔注意〕 判断一个集合A是另一个集合B的真子集,需要说明 两点: 1°A二B,2°B中至少有一个元素不属于4、 ●10● ==========第21页========== 1.在下面各题中的处填上适当的符号(∈,年,,P,C) 13 (1)11}: (2)9{o}; (3)a_{a,b,c; (4){a}{a,b,c; (5){a,b}{a,b,c; (6){c,b,a}{a,b: (){c,b,a{a,b,c;(8)0{a,b,c}. 2.把下面的三个集合A,B,C间的关系,用文氏图表示出来: ACBCO 3.写出集合{1,2,3}的所有的子集,其中有多少个非空真子集? 4.证明:{이x-3x+2-0아っ{xー1-아。 §1·4交集、并集 上一节里,我们所学习的子集、真子集、集合的相等都 是关于两个集合间的包含关系.对于两个集合A与B, 我们还常常要研究它们间的另一类重要关系一一交和并 先来看下面的例子:设有四个集合: A={1,2,3,4,5;B={3,4,5,67; 0={3,4,5;D={1,2,3,45,6,7. 我们来分别考察集合O、D的元素与集合A及B的元素间 有怎样的关系 很明显:第合C的元素3、4、5,既是集合A的元素,同 时也是集合B的元紫,也就是说,集合C是由A与B这两 个集合的公共元素所组成的;而集合D的元素,有的是集 合A的元素(如1,2,3,4,5),有的则是集合B的元素 (如3,45,6,7),也就是说,集合D可以看成是由A、B 这两个集合的元素并起来组成的(不过相同的元素只用一次).为了描述集合间这两种不同性质的关系,下面我们引进交集和并集这两个重要概念、 11 ==========第22页========== 1。交集 由同时属于集合A和集合B的一切元素所组成的集 合,叫做集合A与B的交集.表示为 A0B 读做“A交B”,或者“A与B的交”. 交集的意义,用式子来表示,也就是 AnB={c∈A且c∈B}. 图1.2中的斜线部 分,就表示集合A与B的 交集A∩nB. B 根据交集的定义,容 易知道,对于任何集合A, 总有 A∩A=A,An0=0 图12 例1 设A={x>-2},B={心<3},求A∩B,并利用数轴把图象面出来. [审题] A是由一切比一2大的数组成的,数轴上可以用表示 一2的那个点P为端点向右方向的射线(不包括P点)来 表示:B是一切比3小的数组成的,数轴上可以用表示3的 那个点Q为端点向左方向的射线(不包括Q点)来表示. A∩B是由一切比一2大同时又比3小的数组成的,数轴 上可用不包括端点在内的那条线段P?来表示 解] A∩B-{xx>-2}n{aco<3} ={-2<<3}. P -3-2-1012345 图13 •12● ==========第23页========== 2,并集 由属于集合A或者属于集合B的一切元素所组成的 集合,叫做集合A与B的并集,表示为 AUB, 读做“A并B”,或者“A与B的并” 并集的意义,用式子来表示,也就是 AUB-={x∈A或x∈B}. 图14中的斜线部分,表示集合A与B的并集 AUB. 根据并集的定义,容易知 道,对于任何集合A,总有 AUA-A,AU0=4. 例2 设:A={锐角三角形}, B={直角三角形), 0={钝角三角形}, 图14 D={斜三角形},I={三角形}. 求(1)AUC:(2)BUD 并说明理由。 [解] (1) {锐角三角形}U{钝角三角形} ={锐角三角形或钝角三角形}={斜三角形, .'AUC-D. (2) {直角三角形}U{斜三角形} ={直角三角形或斜三角形}={三角形}, BUD-I. 例8 证明:{x心≤-2}U{x≥2}*{llx≥2},并利用数轴说明这个关系式的意义。 18 ==========第24页========== [解] .·{cx≤-2}U{xx≥2☒={xc≤-2或≥2幻, {xx≥2}={c≤-2或c≥2}.'.{x心≤-2}U{xx≥2}={xlx≥2}. Cベ-2 x≥2 0 图15 这说明数轴上在原点左边且与原点的距离不小于2个单位的点的集合和在原点右边的且与原点的距离不小于2个单位的点的集合并起来就是与原点的距离不小于2个单位的点的集合 习 1.在下面各题的{}内填上适当的语句,并用文氏图表示出来: 1•4 (1){菱形}n{矩形}={}; (2){等腰三角形∩{直角三角形={; (3){等腰三角形}U{等边三角形}={ }; (4){质数}U{合数}={. 2.设A=1,2,3,4},B={3,45,6},C={6,7,8},求 (1)AUBUC; (2)AnBnC; (3)(AnC)UB; (4)(BnC)UA. 3.在下面各题的处,填上适当的符号(C,一,=) (1)AUB BUA; (2)AnBBnA; (3)AUBA; (4)A0BA. 4.设A={(c,y)川x+2y=3},B={(c,y)川2x一y=0}, C={(x,y)1x2+4y2=5}, 求:(1)AnB(2)AnC §15全集、补集 上一节里,我们学习了求两个集合A与B的交集或者 并集这两种关于集合的运算一“交”和“并”。这一节里,我们将学习关于集合的另一种运算。 14 ==========第25页========== 先来看一个例子. 在实数范围里解方程c一2=0,因为当心=√2或花=一√2时,这个等式都能成立;但是当心≠士√2时,这个等式就不成立,所以我们可以确定: 1°一切属于集合A={√2,一√2}的元素,都是 这个方程的解; 2°一切属于集合R,但不属于集合A的元素都不是 这个方程的解 由此我们也就可以说,在实数集R里,这个方程的解 集是A={√2,-√2}. 但是,如果在有理数范围里解这个方程,因为没有一个有理数的平方能够等于2,这个方程就没有解.应用集合 的语言来说,也就是这个方程在有理数集Q里的解集是空 集0. 从这个例子中可以看到,在研究方程的解时,总是在某 一个给定的集合里讨论的,这个集合包含了未知数心的一切允许值.这样的一个预先给定的集合,叫做全集。方程的解集,以及不属于这个解集的元素所组成的集合,都是它的子集. 一般地说,所谓全集,也就是预先给定的包含有我们所 要研究的各个集合全部元素的集合.全集用符号I表示, 在文氏图中,通常画出一个矩形来表示. 例如在实数范围里研究方程的解集时全集I=R,而 在有理数范围里研究方程的解集时全集I=Q 从上面所举的例子中,还可看到方程心一2=0的解集 A={/2,一W2}把全集I=R分成了两个部分,R中 的一部分元素属于A,而另一部分元素不属于A,我们把 由这些不属于A的元素所组成的集合,叫做集合A关于 R的补集,并且用符号A来表示。 o156 ==========第26页========== 一般地,已知全集I和集合A(A二),由I中所有不 属于A的元素所组成的集合, 叫做集合A关于I的补集(简 称集合A的补集),表示为A. 图16中的矩形表示全集 I,圆表示集合A,斜线部分就 表示集合A的补集五, 补集也可以叫做余集,求 图1.6 集合A的补集互的运算,叫做“余”. 根据补集的定义,并利用图16容易知道对于任意集 合A,总有 AUA-I,A04-0. 例1 设I={1,2,3,4,5,6,7,8,9,10}, A={L,3,5,7,9},B={3,6,9}, 求 (1)AUD和A∩B; (2)AUB和AnB. [解] (1).·A={2,4,6,8,10}, D={1,2,4,5,7,8,10}, .AUE={2,4,6,8,10 U{1,2,4,5,7,8,10} ={1,2,4,5,6,7,8,10}。 AnB={2,4,6,8,10 ∩{1,2,4,5,7,8,10 ={2,4,8,10}. (2).·AUB={1,3,5,7,9}U3,6,9} ={1,3,5,6,7,9, .AUB={2,4,8,10}. ●18· ==========第27页========== ,·A∩B={1,3,5,7,9}∩3,6,9} ={3,9}, ∴.AnB={1,2,4,5,6,7,8,10}. 例2 设I=N,A={质数},B={合数}, 求AUB [解] ·自然数可以分为1,质数和合数三类, AUB={质数}U{合数}={质数或合数}, ..AUB={1. 〔注意) 本题结果不能写成AUB=1, 习题 1.图中I是全集,用关于A,B的集合符号把斜线部分表示出来 15 (2) 3) ) (第1题) 2.设1=R={实数},A=见={有理数},求A,并用文氏图表示出 来. 3.I=a,b,c,d,e,f,gh,A=ta,b,ch,B={c,d,e,f,求:A,B,AUB,AnB,AUB,An 4.设1=R={实数},A={xx≤一3或x≥3},求A。并在数轴上表示出来 017● ==========第28页========== 本章提要 1.集合的概念 集合是数学中一个重要的不定义概念,意思是指具有某种属性的事物的整体。 2.集合的表示法 (1)列举法;(2)描述法;·(3)文氏图表示法。 3.元素与集合间的关系 a∈A--a是集合A的元素(a属于A).a庄A--a不是集合A的元素(a不属于A). 4.集合间的包含关系 A三B-A是B的子集(a∈A必有a∈B). ACB-A是B的真子集(a∈A必有a∈B,但B中至少有一元素b丰A). A=B—集合A与B等(A드B,且B드A). 5.集合的交与并 (1)交集 A∩B={x∈A}∩{x∈B} ={xx∈A且x∈B. (2)‘并集 AUB={x∈A}U{xx∈B} ={xx∈A或x∈B}, 6.全集、补集、空集 (1)全集I一所考察的各集合的全部元素所组成的集 合 ==========第29页========== (2)补集-一二I,五称A关于1的补集。 A-{cx年A,AEI}. (3)空集0不含有任何元素的集合。 1=A,则A=0 复习题一A 1.回答下面的问题: (1)什么叫做有限集、无限集?各举一个例子; (2)什么叫做单元素集合?举一个例子. 2,下面写出的表达式是不是正确?如果不正确把它改正过来: (1)9={0}: (2)0={03; (3)AUACI; (4)ArA=0. 3.在下面表格的空格里,填入适当的集合: (1) A B (2)U A B 0 0 1 A0B A AUB B BOA B BUA 4.在下面表格的空格里,填入适当的集合: (1) (2)U 0 A 0 0 A A 5.用列举法把下面的集合表示出来: (1){xc>-3且x∈J}n{x≤2且x∈J}; (2){al<3且x∈V}U{xx≤5且x∈N. 6.在下面的处填上适当的符号(U,∩,C:一): (1){xx<-2或x>3}={c<-2}cx>3}; ●19● ==========第30页========== (2){c>-2且x<3}={cc>-2}{x<3}; (3){ix<-4}_{x<一3或>3}; (4){x>-2}.{x>2且x<3}. [提示:要注意“或”和“且”这两个字的用法,也可以利用数轴找 出答案.] 复习题一B 1.用列举法写出下面的果合: (1){xa3-32+x+1=0}; e{ov小厚+层ac+w- 2.如果用符号(X)表示有限集X中的元素的个数,今有有限集 A=a,b,c,d},B=c,d,e,f,gh, (1)证明n(AUB)=2(A)+%(B)-n(AnB). (2)用文氏图来验证这个等式对于一般的有限集合A、B都成 立, 3. 设I={1,2, 3…,9}, A={1, 2,3,4},B={3,4, 5,6,가, (1)验证下面这两个等式成立. AnB-AUB,AUB-AnB; (2)利用文氏图验证这两个等式,对于一般的集合4和B都成 立. 注:这两个等式叫做德·摩根法则 4.班级里有45个学生,其中有15个学生有哥哥,13个学生有姐姐,其中有2个学生既有哥哥又有姐姐.如果用1表示全班学 生的集合,A表示有哥哥的学生的集合,B表示有姐姐的学生 的集合 (1)n(A),n(B),n(AnB),n(A),n(B); (2)问这班学生中: ()有哥哥或有姐姐的,(i)没有哥哥且没有姐姐的,(ii)有哥哥但没有姐姐的, “◆20· ==========第31页========== (iv)有姐姐但没有哥哥的学生各有几人? 第一章测验题 1.在下各题的处填入适当的符号: (1)1W; (2)0_N; (3)32N; (4)JW; (⑤.{1}{1,2,3}; (6)1_{1,2,3} (7){1,2,3}-{3,2,1: (8)AA=I; (9)A五=0; (10)AA=4A-A. 2.在下面的{}内填入集合中的元素的限制条件: (1){axlr<0}U{>5}={x|}; (2){xx>0}n{xx<5}={x|. 3.已知 1={0,1,2,3,4,5,6,7,8,9}, A={1,3,5,7,9},B={1,2,3,4,5}, C={2,4,6,8}. 求(1)AUB; (2)AnC; (3)(AnB)UC; (4)AUB; (5)AnB; (6)(AUB)nC. 4.用两种方法(描述法,列举法)写出下列方程或方程组的解集 (ーy+2)x=0, (1)2x4-3x2+1=0; (2)(c-y-2)y=0, 021● ==========第32页========== 2 不等式 在代数第二册里,曾经学过关于不等式的一些概念、性质和一元一次不等式的解法.本章将在复习这些知识的基础上,学习关于不等式的其它一些重要性质、各种代数不等式的解法以及不等式的证明, 不等式与等式有许多类似的性质,但也有一些不同的性质.关于不等式的两类问题:“解不等式”和“不等式的证明”,与等式中的“解方程”和“恒等式的证明”也很相象。对比着等式来学习,可以帮助我们更好地理解以下所讲的内容。 §2·1不等式的概念 我们先来复习一下过去学过的一些知识。 1.实数大小的比较 我们知道,两个实数a与b之间,存在且只存在下面 三种关系中的一种: (1)a大于b,记作a>b; (2)a小于b,记作ab,如果a一b是负的,那末 ·220 ==========第33页========== ab,那末a-b是正的,如果a0, >b, 如桌 a-b=0, 那末 a{=b, <0, b, >0, 反过来,如果 :b, 那末a-b3=0, 0, >b, aー=0,a{=b, <0, <6. 实数的运算比较性质 这样的两个命题,叫做等价命题。 2.代数式的值的大小比较 有时候,我们也要比较两个代数式的值的大小.这时,可以根据一个代数式的值大于、小于、或者等于另一个代数式的值,而分别用符号“>”,“<”,或者“=”把它们连接起来. 象比较两个实数的大小一样,比较两个代数式的值的 ·28● ==========第34页========== 大小,也只要考察它们的差就可以了.例1 比较(+3)(-5)和(+2)(-4)的大小. [解] (x13)(-5)-(+2)(-4) (2.-2x-15)-(2-2c-8)=-7<0, .(+3)(x-5)<(2)(-4). 例2 比较(21)和1的大小. [解] (2+1)2-(4+2+1) (4-2x2+1)-(x4+心2+1)=x2. (1)如果2=0,那末22.0,这时有 (22+1)2-(42+1)=0, .(x2+1)2=42+1. (2)如果心≠0,因为不等于零的任何实数的平方都是正数,所以22>0,这时有 (x2-1)8-(x4+22+1)>0, .(21)2>+2+1. 注 上面这两种情况,合在一起可以写做 (02+1)24+c2+1, 这个式子表示(2+1)的值不小于+x2+1的值. 例8 比较(a-1)和a2+1的大小. [解] (g-1)2-(z21)=(a2-2a1)-(a2+1)=-2a.因为字母,可能表示正数或负数,也可能表示零,所以要分三种情况来考察: (1)如果x是正数,那末一2a就是负数,这时有 (a-1)9.a2+1. 上面讨论的三种情况,合在一起就是 0240 ==========第35页========== 「0, (a-1)3{=a2+1如果a=0, >a2+1如果a<0, 3.i严格不等式和非严格不等式 我们知道:用等号“=”把两个代数式连接起来所成的式子叫做等式。类似地,用不等号“>”或者“<”把两个代数式连接起来所成的式子我们叫做不等式.注 因为这里所指的等式或者不等式都是由代数式组成的,所以明确些说,可以把它们称做代数等式或者代数不等式,以后我们还将学到由其它数学式子所组成的等式和不等式. 在比较两个代数式的值的大小的时候,有时我们只簫作出判断:前一个代数式的值不小于(或者不大于)后一个代数式的值,这时我们可以用符号“≥”(或者“≤”)把它们连接起来.例如,在上面例2中,我们曾判断过,当心≠0的时候,总有 (2+1)≥+x2+1, 象这种用符号“≥”(读做大于或等于)或者“≤”(读做小于或等式)把两个代数式连接起来的式子,也叫做不等式.为了区别,我们把用符号“>”或者“<”连接而成的不等式叫做严格不等式,而用符号“≥”或者“≤”连接面成的不等式叫做非严格不等式。注 从例2的解答中,可以看到非严格不等式实际上是由一个等式和一个严格不等式并合起来的,所以它具有等式和不等式所公有的性质.在以下我们讨论不等式的性质时,只需着重讨论严格不等式所具有的性质. 4。绝对不等式、条件不等式和矛盾不等式 我们知道等式是在给定的数集里研究的,它可以区分为三类: ·26· ==========第36页========== 1°如果不论用什么数值代替等式中的字母(只要是允许的)它都能成立,这样的等式叫做恒等式; 2”如果只有用某些数值代替等式中的字母,它才能够成立,这样的等式叫做条件等式; 3”如果不论用什么数值代替等式中的字母,它都不能成立,这样的等式叫做矛盾等式 同样的,不等式也是在给定的数集里研究的,它也可以分成三类,就是: 1°如果不论用什么数值代替不等式中的字母(只要是允许的)它都能成立,这样的不等式叫做绝对不等式;2°如果只有用某些范围内的数值代替不等式中的字母,它才能够成立,这样的不等式叫做条件不等式; 3°如果不论用什么数值代替不等式中的字母,它都不能成立,这样的不等式叫做矛盾不等式 例如,任何一个实数的平方都不能是负数,根据这一性质,我们就能判断出: (1)a2+b+1>0是一个绝对不等式; (2)a2+b2<0是一个矛盾不等式; (3)(a-b)2>0是一个条件不等式.(因为只有当a≠b时这个不等式才能够成立.) 比较下列各题中两个代数式的值的大小(15): 题 21 1.(&-5)(a-7)和(a-6)2. 2.(&+1)(a2-u+1)和(a-1)(a2+a+1). 3,(a2+V2x41)(x2-V2x+1)和(c2+c+1)(c2-x+1)(x≠0) 4.2+b2和2ab.[提示:按照u=b或者u≠b分别考察.] 5.√x2-4c+4+2c和V1-2x+c2,(10: (2)x2-x+1<0; (+8+是: (倒9++是0, §2·2不等式的基本性质 对于等式来说,我们已经知道它具有下面这些基本性质: (1)如果a=b,那末b=;反过来,如果b=a,那末a=b. (2)如果a=b,b=C,那末g=c(相等的传递性). (3)如果a=b,那末a十c=b十c. (4)如果心=b,那末0=bc. 等式的基本性质 不等式也有一些类似的基本性质: 性质1,如果a>b,那末b<;反过来,如果 bb 性质2.如果a>b,b>C,那末a>c(不等的 传递性)。 性质3.如果>b,那末a+c>b十C,性质4.如果a>b,那末 >b(当>0的时候),ac{=bc((当c=0的时候),b,.a-b是正数. (1)如果c是正数,那末因为两个正数的积仍是正数,所以(a-)c>0,这时ac>bc. (②)如果¢是负数,那末因为一个正数与一个负数的积是负数,所以(a一b)c<0,这时ac2c,3c-2c>1和>1 是同向不等式,而不等式 一7c>一14和化<2 是异向不等式 这样,我们也可把不等式的基本性质4说成: 不等式的两边同乘以一个正数,那末得到和原不等式同向的不等式;如果同乘以一个负数,那末得到和原不等式异向的不等式;如果同乘以零,那末得到一个等式. 在研究不等式时,经常要用到以上这些性质: (1)利用性质1,我们只需改变一下不等号的方向,就 ◆28· ==========第39页========== 可以从不等式>b所具有的性质,推出不等式)顺次地连接起来.例如 a+1a+3>a+1. (3)利用性质3,我们就可以推出不等式的移项法则:不等式中任何一项可以把它的符号变成相反符号后,从一边移到另一边 (④)利用性质4,我们可以在不等式的两边同乘以(除以)一个相同的正数或负数.但是必须注意乘以(除以)一个正数不等号的方向不变,乘以(除以)一个负数,不等号要改变方向.因为乘以0,不等式就变成了等式,所以在不等式的变形中,这是不允许的.例1 已知不等式 mx-2>心-3m(9n≠1). 应用不等式的基本性质,证明: 2-3m m-1’ 化 <2-3m m-1. [证明] 由 mx-2>x-3m, (1) 移项后得 (m-1)a>2-3m. (2) 1°如果m>1,这时m-1>0,在不等式(2)的两边同乘以正数1 %1,得 e2-3m m-1。 ·29.· ==========第40页========== 2°如果m<1,这时m一1<0,在不等式(2)的两边同乘以负数1 饥,得t<2-3n m-1。 习题 1.求证: 2·2 (1)如果a>b,b>c,那末a>c; (2)如果a>b,b=c,那末a>c; (3)如果a=b,bb,b>c '.a-b>0,b-c>0. 今 &-C=(-b)+(b-c), 因为两个正数的和仍旧是正数,所以&一c>0,由此可知a>c.] 2.(1)如果a>b,c=d,是否一定能得出ac>bu,为什么? (2)如果ac>bc,是否一定能得出a>b,为什么? (3)知果abd这一结论.] 3.应用不等式的基本性质,证明: (1)如果ac2+b2>bx+a2,且u>b,那末x>u十b; (2)如果mx-2<2x一3,且m<2,那末 x>-(m2+n+n2). §2·3一元不等式的解和解集 在代数第二册里,我们曾经学过在有理数的范围里解 一元一次不等式;并且已经看到解一元一次不等式和解一元一次方程有许多相似的地方,现在我们进一步来学习有 ●80● ==========第41页========== 关解不等式的一些重要概念。 1.一元不等式的解和解集 含有一个未知数的不等式叫做一元不等式;在给定的未知数的允诈值集合里能够使不等式成立的未知数的值,叫做不等式的解;这些值的全体,叫做不等式的解集;求不等式的解集的过程,叫做解不等式。 例如对于不等式x-2>0来说,因为3一2=1>0,所以心=3就是它的一个解;又因一切大于2的实数减去2都大于0,所以它们都是这个不等式的解.但是2或比2小的数减去2都不大于0,所以它们都不是这个不等式的解,因此这个不等式的解集是{xx>2}. 又如,对于不等式≥1来说,根据绝对值的意义,可以知道它的解应该是满足条件 x≤-1或x≥1 的实数,反之,满足条件x≤一1或c≥1的一切实数,也都满足不等式x≤1.所以它的解集是 {a.x≤-1}U{cc≥1}={xc≤-1或≥1}. 一元不等式的解集,可以利用数轴把它表示出来.例如图2.1(1)和(2)中黑线画出部分就分别表示了不等式 E-2>0和x≤1的解集 (1) (②) 图21 2。不等式的同解变形 在代数第二册里学习方程的时候,曾经讲过:设有两个· 810 ==========第42页========== 方程,如果第一个方程的解都是第二个方程的解,并且第二个方程的解也都是第一个方程的解,那末这两个方程就叫做同解方程。把一个方程变形成它的同解方程的过程叫做方程的同解变形. 同样的,设有两个含有未知数的不等式,如果第一个不等式的獬都是第二个不等式的解,并且第二个不等式的解也都是第一个不等式的解,那末这两个不等式就叫做同解不等式.把一个不等式变形成它的同解不等式的过程,叫做不等式的同解变形. 因为在§2.2里所提出的不等式的性质2和性质3(除去乘以0以外)中,已知条件和得出的结论都是可逆的,也就是 1°a>b台&+c>b+C, 2°a>b(c>0)→ac>bc(c>0),3°a>b(c<0)台a0-그 (1) [解] (1)式两边都乘以正数6,得 12(x+1)+2(x-2)>21x-6, (性质2) 就是 14x+8>21x-6. (2) 因为当x取确定值时,整式21x+8是一个确定的值,在(2)式两边都加上整式一(21x+8),得 14x+8-(21x+8)>-6-8, (性质1) 就是 ◆32● ==========第43页========== -7>-14, (3) (3)式两边都除以负数一7,得 <2. (性质3)(4) 因为上面的变形,每一步骤都是可逆的,所以不等式 (1)、(2)、(3)、(4)都是同解不等式. 由此可知不等式(1)的解集是{<}. 习题 1.讨论不等式(x+√Z); (5)5(g-1)-x(7-)(x2+1)(3x-4). 3.解下列关于x的不等式: (1)k(-1)>x-2; (2)(p-9)x<2-g(p+q)。 4.确定取仆么数值时,方程 2+(2k+1)e+2+1=0 (①)有两个不等的实数根; (2)有两个相等的实数根; (3)没有实数根. §2·4一元一次不等式组 在解一些具体问题时,有时根据问题中的条件,未知数的数值范围,儒要同时满足儿个不等式。.例如:某天的天气预报,当天最低温度是摄氏16度,最高温度是摄氏22度,如果用心表示当天福度度数,那末化可以取值的范围,需要同时满足下面这两个不等式: ≥16, (1) c≤22. (2) ●33· ==========第44页========== 我们说,不等式(1)和(2)组成一个不等式组. 不等式(1)的解集,可用数轴上从表示数16的点A开 始向右的射线AX来表示.不等式(2)的解集,可用数轴 上从表示数22的点B开始向左的射线BX'来表示.这 两条射线的公共部分是线段AB(图2·2). 图2·2 由此容易看到,如果:取大于或者等于16同时又小于或者等于22的一切数值,不等式(1)和(2)就能都成立,“可以取值的范围,就是16≤c≤22,在这个范围里的每一个实数心,都是这个不等式组的解,所以这个不等式组的解集是{x16≤≤22}.为了方便,通常我们也可省去上面的集合符号,把这个不等式组的解集就用{}里这个限制条件16≤≤22来表示.把答案说成:“答:16≤≤22” 一般地说:含有相同未知数的儿个一元一次不等式所组成的不等式组,叫做一元一次不等式组;能使不等式组里的各个不等式都成立的未知数的值,叫做这个不等式组的解;这些值的全体,叫做不等式组的解集 从上面的例子可以看到,求不等式组的解集,就是要找出不等式组里各个不等式的解集的公共部分;换句话说,也就是要找出各个不等式的解集的交集.在初学时,利用数轴可以帮助我们容易找到所求的这种交集。例1 解不等式组: 5(x-3)>3(2w-3), 5(-2)<3(-1). [解] 原不等式组可以化成: ·340 ==========第45页========== 6, R<32 *6 3 图2,3 答:<-6。 注 应用集合的运算来叙述,就是解集为 くー아rels8}=<-6) 例2 解不等式组: (-1)2<(+1)3-4, (1) (-1)(+2)>(+3)(x-4)+20 (2) [解] 由(1),2-2x+1<2+2+1-4,移项得 -4<-4, 所以 2>1. (3) 由(2),x2+化-2>2-x-1220,移项得 2a>10, 所以 x>5. (4) 因此,原不等式组可以化成: x>1, x>5. 5 图24 答:x>5. ·880 ==========第46页========== 注 应用集合的运算来叙述,就是解集为 {xc>1}n{cc>5}={xx>5}. 〔说明〕 原来的不等式组里的两个不等式,经过变换以后都是 一次不等式,所以这个不等式组是一元一次不等式组. 例3 解不等式组: (x-1)2<(c+1)2-4, (1) 1(-1)(+2)<(x+3)(-4)+20. (2) [解] 由(1),得2-2+11, (3) 由(2),得2+x-21, 25. 图2.5 答:1<花<5. 注 应用集合的运算来叙述,就是解集为 cla>1n1la<5==1l>1<5 ={l19. [解]·原不等式组可以化成 <-1, x>1. •36● ==========第47页========== 图2.6 答:没有解。 注 应用集合的运算来叙述,就是解集为 {cx<-1}n{xx>1}=0. 观察上面的这些例子,可以发现由两个一元一次不等式组成的一元一次不等式组的解集有下面儿种基本情况: (1)a<).它的解集是>,可以好 ab 成>b(比大的还大). (2)<,(a<).它的解集是<,可以写成w∠bm之a,(a<).它的解集是{a<0, (1) 3+1<0, (1) (2) 21<; l1-2>0: 2-30, 3-4x<0, (3) (4) 3.-50; 2+3<0, 2.:下列各不等组: 23>5, 6x+2<4, (1)3、 (2) 4 2x-1<+1 5 5(-1)-1>2(r+1) 6 3 (3)2+3r+1<3--1 8 #1+43, x (4)6 3-1-13-2>7-1x+32; 2 3 6 5(+1)+6(x+2)>9(x+3), (5)7x-32x+3)>2(x-1); ー—3<3+4-4, 2 5 (6)(写c+5(生-x)<2(4-x)片 (3r+5+10-3r>23x+77 5 3 -8, (7)7=11(x÷3)>3-1-13-;( 6 52 33-7+x+1>4-7-3 (8) 10 5 7(3x-6)+4(17-)>11-5(x-3): (9)(x-1)(2+x-+1)>(3:+1)3-3.x, (c+1)(x2-x+1)≤(x-1)3+3.x;(x-2)2>(x+1)月, (10)1(r+1)2<(-1)3. 如果不等式组里不等式的个数超过两个时,一般来说,借助于数轴来解容易把各不等式的解集的父集找出来,因 ·88· ==========第49页========== 此也容易防止发生错误。例5 解不等式组: xー1>0 「x1>0 [x-1<0, (1)x-3>0, (2) -3>0, (3){x-3<0, a-5>0; x-5<0 c-5>0, [解] (1)原不等式组就是 x>1, 比>3, x>5. 由数轴.上可以看出,所求的解集是c>5 图2.7 (2)原不等式组就是 [w>1, c>3, <5. 由数轴上可以看出,所求的解集是 35. 。.39● ==========第50页========== 由数轴上可以看出,这个不等式组没有解 图2.9 上面的这些例子里,未知数都可以取满足条件的实数值.但有时根据题目中的条种,可能只要我们求出未知数的整数值或者正整数值,这时可以先求出未知数的一般解,然后再在其中找出满足条件的数值来. 例6 求不等式组 「(2+1)≥(-1)(2++1), (1) (1-0)3+x(33+1)ン3(2+『-9) (2) 的整数解. [解] 由(1)得 3x云x3-1, 就是 x≥-1. (3) 由(2)得1-3x十3x9-x+23+x>32+3x-27,就是 心学 (4) 所以原不等式组可以化成不等式组 [c≥-1, (e<8 5 它的解集是 28 5· 在这范围里的整数有-1,0,1,2,3,4,5等七个,所以所求的整数解是-1,0,1,2,3,4,5. 例7 车间因准备制作1(00个零件,如果按照定额平均分配给6个小组,那末不能完成;如果每一小组多分配1件,那 ==========第51页========== 末就可以超额完成.问按照定额每一小组要完成多少个零件? [解] 设按照定额每一小组要完成c个零件,那末按照题意可列出不等式组: 6x<1000, 6(x+1)>1000. 这个不等式组可以化成 2 f<1663 >1652 3 所以原不等式组的解集是 -1, [c+1<0, (1)x>-2, (2)然+2<0, >-35 +3<0; [g<-1, 〔+1>0, (3){x<一2, (4)x+2>0, ℃>-3; +3<0 从解题过程中,你能总结出解这类由二个以上的简单一元 一次不等式所组成的不等式组有什么简便的方法吗? 2.已知a>b>c,解不等式组: [x >a, g<, (1)≥b, (2)xc3 s, [xc 3.求下列各不等式组的整数解: 3x-10>0, (1)16x-10<4; 2x-11+19-2<2, 4 2 (2) 号-)亭 4.小组里有6个工人,按照指标每天要生产零件1000个.如果按原来的定额平均分配给工人,就不能完成指标.如果把每人的定额增加2件,那末就将超过指标.问原来的定额每个工人每天要生产零件多少个? §25区 间 在前面我们解一元一次不等式和不等式组的过程中,可以发现所求不等式或不等式组中未知数化可以取值的范围都是由下面这类不等式确定的(这里a, x≥a, aa [a,o)={a} (-心,)={-2}, ..∈(-2,十0). 习题 1.把下列不等式所示的x的取值范围用区间符号表示出来: 25 (1)x<3; (2)x≥-2; (3)-30, rk-4<0, (I)6+4<0 +4>0. 这里不等式组(I)没有解,而不等式组(I)的解集是 一40 ax2+6x+c<0, 这里a≠0. 从上面的例子可以看到,如果这两种形式的不等式中 二次三项式ax2+x十c能够分解为因式,那末解一元二次不等式的问题,就可以转化为解两个一元一次不等式组的问题.因为只要这两个不等式组中有一个能够成立,原不等式也就能够成立,所以我们只要先分别找出每一个不等 ●48 ==========第57页========== 式组的解集,然后再找出他们的并集,就能得到原来这个一元二次不等式的解集 例如,要求一元二次不等式 x2-7x+12>0 的解集,首先把左边分解因式,得 c2-7x+12=(-3)(x-4). 所以原不等式就是 (x-3)(x-4)>0. (1) 因为要两个数的积是正数,必须而且只须这两个数都是正数或者都是负数,所以求不等式 (x-3)(-4)>0 的解集,也就可以归结为求不等式组: -3>0, () 和(I1) x-4>0 的两个解集的并集. 解(I),得它的解集是:{c>4;解(II),得它的解集是:{xc<3}.所以所求的解集是: {ex>4U{x>3}={xc<3或x>4}.这个解集可以在数轴上表示出来,如图216, 图2.16 这里应该注意不等式组()和(I)的解都是不等式 (1)的解,所以求不等式(1)的解集,应该求它们的解集的 并集;但是求不等式组()或不等式组(I)的解集,那末 就要求出每一个不等式组里二个不等式的解集的交集.两者不能混淆. 象解一元一次不等式一样,在解一个一元二次不等式的问题时,通常也只需把确定解集的那个条件求出,并说这 ◆47● ==========第58页========== 就是所求的解集,如下面各例所示。 例1 解不等式 2-7c十12<0. [解] 因为2-7x+12=(x一3)(x一④).所以原不等式就是 (x-3)(x-4)<0. 因为要两个数的积是负数,必须而且只须这两个数中 一个是正数…个是负数,所以从这个不等式可以得出两个不等式组: c-3>0, -3<0, (I)lL-4<0 (I)L-4>0. 不等式组(I)的解集是31. (1) [解] 把不等式的左边展开得x2-4>1.移项得 2-5>0, 就是 (c-5)(+N5)>0 (2) 不等式(2)可以分成下面两个不等式组来解: の{e-V5>0,(I1)-√6<0, l+W5>0 lax+W√5<0 解(1),得x>√5;解(II)得<-百. 所以原不等式的解集是:<一√5或>√, 注 因为由两个数的积大于1,并不能肯定这两个数都大于1,或者都小于一1.所以把不等式(1)化成不等式组 ∫-2>1, x+2>1和-2<-1,1+2<-1 来解是错误的, e48● ==========第59页========== 1.解下列不等式,并把解在数轴上表示出来: (1)(x-2)(x+4)>0; (2)(+3)(a-+1)<0: (3)(1-2c)(1+2x)<0; (4)(2-c)(1+3x)<0: (5)2-13.x+36>05 (6)2-7-8<0; (7)2ax2-0-+7>0; (8)3+5c>22; (9)(x-3)(x+3)<1; (10)(2x-1)(x-3)>-2 2.已知方程2十px÷9=0有两个不相等的实数根,解下列不等 式: (1)2+-p.c+q<0; (②)x2+px+9>0. [提示:先把左边分解因式,得x2一2px+g=(x一a)(c一),(a< B),再解-元二次不等式(-a)(c-B)<0,(a<3或(-x)(-B)>0,(&≤B).] 从前面我们解答过的这个习题中,可以发现一个重要的事实,就是: 如果二次三项式2+心+q中项的系数p和常数项9满足条件4=p2一4q>0,那末只要先解出一元二次方程2+p心十q=0的两个根: x=二p-Np-4q 和 B=“p+Yp-4g 2 2 这样一元二次不等式 (1)2+2p+g<0(p2一4gq>0)就能变形成 (-a)(-B)<0(a&0(p2-4q>0)就能变形成 (c-a)(c-B)>0(aB}. 例3 獬不等式 (1)1-3.x<2;(2)(+1)(-2〉<5. [解] (1)原不等式可变形成+3w一1>0. ◆49● ==========第60页========== .”4=32-4(-1)=13>0, 方程x2+3x一1=0有两个实数根: 贴二3V182=-3+√1g 2 2 .原不等式的解集是t<-3-1g 2 或t>二3+√13 2 (2)原不等式可变形成x一c一7<0. .·4=(-1)3-4(-7)=29>0, 方程x2一x一7=0有两个实数根: x1-1-V2 2 ,4-1+2西 2 '.原不等式的解集是 1-/2图<<1+网 2 2 例4 解不等式 (1)4-3x-22>0;(2)1-320,方程+多。-2-0有两个实数根: =-3-4红 4 ,购=-3+√4红 4 ".原不等式的解集是 -8√厘<<-3+√厘 4 (2)原不等式可以变形成32+x-1>0,即 ·50· ==========第61页========== 4(得》-(-80 方程。+了。-日-0有两个实数根: 1=1-32=-1+/1g 6 6 ·.原不等式的解集是 <-1-18或c>-1+1g 6 6 3 从上例的解答中,可以发现方程w+之心一2=0的两 个根,也就是方程2心2+3心一4=0的两个根;方程 2+2-是-0 33 的两个根,也就是方程3c2+化一1=0的两个根.这也就进 一步启发我们:用一般形式 ax2+bx+c<0(a>0) (1) 或 aa2+bx+c>0(a>0) (2) 给出的-一元二次不等式,当=b2-4ac>0时,只需先求出方程aa2+by+c=0的两个实数根 a=-6-V万-46Bx-b+√b3-4ac 2a 2a 即可写出: 不等式(1)的解集是{xa<<};不等式(2)的解集是{x}, 这样解题可以更加简洁.但是应用这一法则时,必须先使2的系数为正数,否则就会发生错误. 习题 1.直接应用法则,解下列不等式 26 2) d(e-(-)②(e++号》小 (3)(c+1.7)(x+0.5)<0(+g》+0.4到>0: ●81● ==========第62页========== (5(2-3)(x+1)≥0; (6)(c-1)(3c+2)≤0; (7)(1x)(2年c)0; (8)(2-x)(3x+1)≥0. 2.解下列不等式: (1)6x2<7x1-3; (2)5+x-6x2≤0; (3)12-5x-22>0; (4)5x+6≥6x2, 3,解下列不等式: (1)3x2-3x>1; (2)c2+2<4; (3)2x2-5<1; (4)2x-2x2≥-1, 在前面我们解过的这些一元二次不等式中,不等号左边的那个二次三项式都是能够分解成两个不同的一次因式的,也就是说,与这种一元二次不等式相对应的方程 x2+px+g=0的判别式4=p°-4q, 或者方程 a.x2+b+c-0(a>0)的判别式△=b9-4ac都具有大于0这一条件.现在我们进一步研究,如果这种判别式等于0或若小于0,怎样来解和它相应的一元二次不等式.先来看一个判别式等于0的例子. 例如,我们要解不等式: (1)c2-2√5x+5>0,(2)2-2W5x+5<0这里因为4=(一2√5)-4×5=0,我们就不能应用前面学过的法则.但是在这情况下,不等式左边的二次三项式可以分解成两个相同的一次因式,即 x2-2√5+5-(-√5), 出此根据任何一·个实数的平方都不能是负数,0的平方是0这一性质即可得出: (1)不等式x2-2W√5x+5>0,即不等式(x-√5)>0的解是一切不等于√5的实数.也就是说这个不等式的解集是 {xx≠√/5}. 通常我们也可以简单地把它说成:这个不等式的解集是 052◆ ==========第63页========== ≠√5 (2)不等式2-2√5x+5<0,即不等式(一√5)<0没有解.也就是说这个不等式的解集是一个空集. 下面我们再来看一个判别式小于0的例子 例媚,我们要解不等式x2+心+1>0.这里,因为4=(一)3-4=一3<0,我们也不能够应用上面学过的法则,而且由此还可知道,左边的这个二次三项式,在实数集里不能分解成一次因式(这种二次三项式叫做二次质因式).为了解这个不等式,我们可以先仿照解一元二次方程那样,把 二次三项式的前面两项配成完全平方,得 1-(e+)-是+1(e+是)》°4星, 由此把原不等式变形成 这样只要再根据任何一个实数的平方都不是负数,它与一个正数的和总大于0这一性质,就能得出任何实数都是这 个不等式的解,即这个不等式的解集是实数集R.实际上, 这个不等式是一个绝对不等式 同样的,如果我们要解不等式+心十1<0.那末,因 为这个不等式可以变形成(e+》+是<0,由此可以得 出结论,这个不等式无解,即它的解集是一个空集 一般地,我们可以证明: (1)当判别式4=b2一4ac=0时,不等式aw2+b+c>0,(a>0)的解集是 不等式ax2+ba+c<0,(a>0)没有解(解集是0). (2)当判别式=b3-1c<0时, 不等式c2+bc+c>0(a>O)是绝对不等式(解集是 ·53 ==========第64页========== R). 不等式ax2+bx+c<0(a>0)没有解(解集是),(证明留给读者.)例5 解下列不等式: (1)a+2(W√2+1)x+3+2W2>0; (2)2x2-3x+4<0 C解] (1)这里=4(2+1)3-4(3+2√2)=0,原不等式可变形成 [x+(W2+1)]2>0. 所以这个不等式的解集是{xx≠-(W√2+1). (2)这里4=(一3)8-42.4<0,这个不等式没有解。 习题 1.应用配方法解下列不等式: 26 (1)2x2+4x+3>0; (2)(1-2x)(3+2x)-5>0, (3) 2.证明第53页中的结论 3.解下列不等式(用最简便的方法) (1)x2-3x+4>0; (2)2c2-5.x+3≤0; (3)32-x-3<0; (4)3x2-x-4>0; (5)x2+2x>6x-15; (6)3x2-7x<6; (7)3-x>3x+5x2; (8)(3-1)(x+1)>4; (9)x2-3>3x-9; (10)1-4x2>4z+2. “§2·7一元二次不等式组 学会了解一元一次不等式、一元二次不等式以及一元 一次不等式组的解法,我们也就可以来解由一元一次不等式和一元二次不等式所组成的,或者都是由一元二次不等式所组成的不等式组.这样的不等式组都叫做一元二次不等式组 象解一元一次不等式组一样,解一元二次不等式组也只需先找出不等式组中各个不等式的解集,然后再求出它 064● ==========第65页========== 们的交集, 例1 解不等式组 fx-1>0, (1) (x+1)(x-2)>0 (2) [解] 不等式(1)的解集是x>1.不等式(2)的解集是心<一1或x>2, 把这两个不等式的解集在数轴上表示出来如图2·17, (2) (1) -(2) 图217 从图中可以看出,它们的公共部分是x>2。所以原不等式组的解集是x>2. 注 详细地叙述就是 解集是{xx>1}∩{x:<-1或x>2={xx>2}. 例2 解不等式组 fc2-2x-3<0, (1) 2十3x-4>0 (2) [解] 不等式(1)就是(x+1)(x一3)<0,它的解集是 -1<化<3 不等式(2)就是(x-1)(+4)>0,它的解集是 x<一4,或>1. 这两个不等式的解集的公共部分是10, (1) +1<0, (2) (+2)(-3)<0; (x+2)(-3)>0: 1c2-2x-3>0, c2-2x-3<0, (3){c2+3-4>0: (4)x2+3x-4<0. 2.下列代数式在x取怎样的数值时才有意义? (1)√1-o2+/422-1; ②)V-花-直+V正-1 √/2+x+1+1 *§2·8一元高次不等式 利用解不等式组的方法,我们还可以解一些简单的一元高次不等式 例1 解不等式(2-1)(x+2)>0. [解] 这个不等式可以分成下面的不等式组(①)和(I)来 解: e2-1>0, (1) (I) (G1-2>0, (2) 解不等式(1)得<一1或>1.解不等式(2)得>÷2, 因此,不等式组(①)的解集是一2<<一1或≥1(图2·19). (2) (2) 图2.19 ()3-1<0, (3) +-2<0. (4) 解不等式(3)得一11. 在解上面这种不等式的时候,用列表的方法来寻找所求的解,可以更加方便.现在把解法步骤说明如下: (1)先把原不等式变形成(+1)(一1)(x+2)>0. (2)令(x+1)(一1)(x+2)=0,求出它的三个根: -L,1,-2. (3)以这三个值为界,把全体实数(除去这三个数)按照数轴上从左到右的顺序顺次地分成4个部分, x<-2 -21 2 图221 (4)分别考察当<-2,一2<<-1,-11的时候,因式心+2,8+1,x一1的值的符号,由此求出在每-一个情况下积(c+2)(+1)(x-1)的符号,填入下面的表格 E<-2 -21 x十2 + g + + x-1 积 例如耿比-2小的数-3来考察.可以看到x+2,g十1,然一1的值都是负数,所以在表格中第二直行的中间一栏里都填入“一”号,然后根据三个负数的积应该是负数,在第二直行最后一栏里也填入“一”号。 57 ==========第68页========== (5)现在要求的是取什么值的时候,积 (x+2)(x+1)(x-1) 的值是正数.从表中可以看出x可取的值应该满足条件 一21.由此即得原不等式的解集是 {-21}。 例2 解不等式(2+1)(2x-1)(x+2)<0. [解] ②x+1)2x-1)(x+2)-0的三个根是-牙,是和 -2. x<-2 2<<--是<<宣 花+2 十 + + 2-1 × + 2c-1 十 积 十 。原不等式的解集是 x<-2或1 一す<く 注 第一直行里各个一次因式的次序,要按照根从小到大的顺序来排列,就是把有最小根的一次式排在最上面,依次排下去,这样考察起来就更方便(读者可考虑一下,这样安排以后将有怎样的规律出现?) 例3 解不等式(2-4)(9-2)>0. [解] 原不等式就是 (x2-4)(a2-9)<0, 也就是 (e-+3)(x+2)(x-2)(c-3)<0. 0<-一3 3<<ー2一2<无<2}20;(2)(x-1)(x-3)(x-5)<0: (3)(x2-1)(x2-4)>0 (4)(1-2)(4-x2)<0 2.解不等武: (1)34-x22>05 (2)x3+2x2-x-2<0; (3)2x-3x2+2<0; (4)x3+2x2-1>0. §2·9一元分式不等式 我]来看下面这个问题:x是什:么值的时候,分式 2x-3 3x-5 (1) 的值是正的? 容易道,这个问题就是要找出x的值的范围,使不等式 음 (2) 能够成立.这个不等式含有分式,我们把它叫做分式不等式. 现在我们来解这个不等式 因为要使左边这个分式有意义,3心-5≠0,(3x一5)>0,所以我们可以用(3x-5)乘不等式的两边,得出和它同解的不等式 (2x-3)(3ax-5)>0. (3) 这样,只要解这个一元二次不等式,就能求出不等式(②)的解集是 a<是或>) ●59● ==========第70页========== 由此可,当<号或2>号时分式(团的值是正的, 从上面的解法中,可以看出解形似 P(<0或者P(x)0 (c) Q(x) 的分式不等式,可以在Q()≠0的条件下,用[Q(x)门3乘不等式的两边,把它转化成整式不等式 P()Q(x)<0或若P()Q(x)>0 来解.这里符号P()和Q()表示心的两个多项式。 例1 解不等式 t含00、3 (1) [解] t_3+4,=(-2)+3(+2)+4 +2十-2+-4 (x+2)(…2) 22+5x+6(a+2)(+3) (+2)(x-2)(+2)(-2) =十3 心-2 在2一1≠0的条件下不等式(①)可以变形成与它同解的不等式 化+3<0 8-2 (2) .·-2÷0,用(2一2)乘(2)的两边得同解不等式 (+3)(x-2)<0 (3) 解之,即得原不等式的解集是{c一3<<2}. 思考题:本题如果改为解不等式 ·3,4 +2c-2中-40, 结果应该是什么? 从这个例了子的解法中,可以看出解-·元分式不等式的 一般法则是: 060· ==========第71页========== 1°把原不等式变形成 P(x)∠0Q() 或 P(0 Q() 的形式。这里P四是一个既约分式; Q() 2°在Q()≠0的条件下解整式不等式 P()(x)<0或P()Q(x)>0. 例2 解不等式 3x42∠1. 8-3 (1) [解] 移项,得 3c+2 -3 -1<0. 通分,得 3x+2-(x-3)<0, -3 就是 2x+5<0. -3 (2) 以(如-一3)乘(2)的两边得同解不等式 (2x+5)(x-3)<0. (3) 解之,即得原不等式的解集是-5<<3}. 〔注意) 在解分式方程 3x+2=1 一3 时,通常我们可以在假定心一3≠0的条件下,两边同乘以整式如一3,得方程 3x十2=D-3, 由此解得 5 ●61● ==========第72页========== 在解分式不等式时,不能套用这个方法。因为虽然我们也可以假定x一3≠0,但却不能肯定x一3的值是正的还是负的,因此不可能直接应用不等式的性质4. 有些分式不等式,也可以仿照解一元高次不等式那样直接利用列表法来解.如下例所示。 例3 解不等式 ℃-1 (E1)t-2)≥0. C解] 当x=1的时候,分式x+(-x-1 的值是0,当 =一1或者x=2的时候,分式(x十)红-2)化一1 失去意义, 我们以这三个数一1,1和2为界把全体实数(除去这三个数)划分成4个部分,列成下表来考察. t<1 -i<化<1 1<红<2 x>2 x+1 x-1 メ + 花-2 化一1 (x+1(c-2 ,原不等式的解集是 -1<<1或x>2. 综合应用解分式不等式和解不等式组的知识,我们还可以解某些简单的分式不等式组例4 一个分数的分子和分母都是正整数,并且分子比分母小1.如果分子加上2,那末分数就大于1.1;如果分子和分母都加上2,那末分数就大于0.9.求原来的分数[解] 设所求的分数是年工,这里女是止整数。根据条件 可列出不等式组 ·62· ==========第73页========== [+2 +1 >1.1, (1) x+2>0.9. x+3 (2) 因为是正整数,所以心1和x+3的值都是正整数.把不等式(1)的两边同乘以÷1,不等式(2)的两边同乘以x+3,得 fx+2>1.1x+1.1, (3) lx+2>0.9c+2.7. (4) 移项得 -0.1x>-0.9 0.1>0.7. 所以 x<9, (c>7. 这个不等式组的解集是70和x+3>0. 习题 1.解下列不等式: 29 (1)4-2x>0; 1+3x (2)3x+7>0; 2-6c (3)5-2x 8+5x<0; (4)2+、0 5x-8 2.解下列不等式: 四 (②)心+1< x-15 (3)3ax-1 x-5<2; (4)1 x-4 1-g 4-红。 3,解下列不等式: (1)- (2)2 3 3-6-3>0. ·63◆ ==========第74页========== 4.解下列不等式: (1) x早1 (2) r=>0:x2-2c-3 (c+2)(-8)<0: (3) 2 2+3C-3>0y (4)四十14+2 c-3>1; (5)c2-2 <0(08+24 2 5 c2-4 5.是什么数的时候,方程c+2=5x+的解是正数?是负数?是零? 6.…个分数的分子和分母都是自然数,并且分子比分母小1.如果分子和分母分别加上1,那末所得的分数就大于是:如果分子和分母分别减去1,那末所得的分数就小于号、求这个分数 §210不等式的其他一些性质 在研究等式的时候,有时我们需要把两个等式的等号两边分别相加、相减、相乘、相除,或者把一个等式的两边分别自乘同次方、开同次方.我们知道,通过这些运算,结果还是得到一个等式,就是 如果a=b,c=d,那末-定有 (1)a+c=b+d;(2)a-c=b-d (3)a0=b; (4)= c d (c≠0,d≠0); (⑤)a=bn(n是自然数); (6)Ya=石(a>0,b>0,n是大于1的整数).同样地,在研究不等式的时候,有时我们也需要把两个不等式的左右两边分别相加、相减、相乘、相除,或者把一个不等式的左右两边分别自乘同次方、开同次方. 现在,我们来研究这些运算在什么情况下才能够进行, 、并且通过这些运算,将会产生怎样的结果 ●84· ==========第75页========== 1.不等式两边分别相加和分别相减 很明显,两个大的数的和总比两个小的数的和大。例如: 5>3,4>2, 而 5+4>3+2; -3>-5,-2>-4, 面 (-3)+(-2)>(-4)+(-5). 这个事实指出了不等式有下面的性质: 性质5 如果a>b,c>d,那末a十c>b+d. 这个性质,我们可以从不等式的基本性质3和2中推导出来,证法如下: [证明] .'>b,..a+0>b+0. (性质3) (1) 又 c>风,..b+c>b+d.(性质3) (2) 从(1)和(2)得 a十c>b+d.(性质2) 很明显,不等式的这个性质可以推广到任意多个(有限的)同向不等式两边分别相加的情形.这就是说: 两个或者几个同向不等式两边分别相加,仍得同向不等式. 〔注意) 把两个异向不等式两边分别相加,就不能作出一般的结论.例如: 3,2<3,这时5+2>343;53,2<4,这时5+2=3+45,3,2<5,这时5+2<3+5. 所以仅知Ia>b,cb,cb-d, 这个性质可以从不等式的性质4和5推导出来.证法如下, [证明] '.∵c-见.(性质4) 又,·a>b,-c>-d; .∴.a+(-0)>b+(-d).(性质5) 就是 a-c>6-d 不等式的这个性质也就是说: 两个异向不等式两边分别相减,得到的是和被减的不等式同向的不等式 〔注意〕 两个同向不等式两边分别相减,就不能作出一般的结论 例1 已知√2的值比1.4大而比1.5小,√3的值比 1.7大而比1.8小.问√2+√3的值比什么数大,比什么数小? [解] .·√2>1.4,W3>1.7, 两边分别相加得 W2+√3>3.1. 又 .·/2<1.5,W③<1.8, 两边分别相加得 √2+W3<3.3 所以Σ+3一定比3.1大面比3.3小.这个结 果可以写做3.1<√2+W3<3.3. 例2 问√3-√Σ的值一定介于哪两个数之间(精确到 0.1)。 [解] /8>1.7, N/2<1.5, ◆66● ==========第77页========== 两边分别相减得 W3-/2>0.2, (1) 因为 /3<1.8, 而 2>1.4, 两边分别相减得 3-√/z<0.4. (2) 把(1)和(2)结合起来,就得到 0.2<√3-W2<0.4. 这就是说√3-√Σ的值一定介于0.2和0.4之间. 习题 1.已知√5的值介子2.23和2.24之间,V2的值介于1.41和 210 1.42之间,求 ①) (1)W5+√/2的值介于哪两个数之间? (2)√5一√2的值介于哪两个数之间?(精确到0.01). 2.已知a2+b2>2ab,b2+c2>2bc,c2+a2>2ca,求证 a2+62+c2>ab+bc+ca, 3.已知u3,4>2,而5×4>3×24>3,6>5,而4×6>3×5, 在这两个例子里我们看到:两个两边都是正数的同向不等式,如果把它们的两边分别相乘,可以得到一个和原不等式同向的不等式.这就是说,不等式有下面的性质: 性质7 如果a>b,c>d,并且a,b,G,d都是正数,那末 ac>bd. [证明] .a>b,c>0,'.ac>bc,(性质4) (1) ·67● ==========第78页========== 又 >d,b>0,..bc>b.(性质4)(2)从(1)和(2)得 ac>bd.(性质2) 注意) 这里如果没有>0,>0这一条件,就不能作出一般的结论、例如: 5>-3,1>-2,这时5×1<(-3)×(-2):5>-2,1>-2,这时5×1>(-一2)×(-2).所以只知道a>b,c>d,就不能肯定ac与bd哪一个大 对于两个异问不等式a>b,c<见,即使知道了a,b,c,d都是正数,一般也不能判断出ac和bd之间哪一个大.例如5>3,2<4,这时5×2<3×4而5>3,2<3,这时却有5×2>3×3. 很明显,不等式的这个性质可以推广到任意多个(有限的)两边都是正数的同向不等式的情形。这就是说: 两个或者几个两边都是正数的同向不等式两边分别相乘,仍得同向不等式 例8 已知a>b>0,求证a3>b3. [证明] 因为a,都是正数,把下面这三个不等式: a>b,a>b,a>b 的两边分别相乘,就得到a>b3. 为了研究两个不等式两边分别相除的问题,我们先引进不等式的下面这个性质, 性质8 如果a>6,2,力都是正数,那末<云 [证明] 因为a,6都是正数,所以6>0,因此品>0.在不等式>6的两边同乘以正数,得 ·68· ==========第79页========== a(品)>(6) 就是 (性质1) 应用这个性质和性质7,容易得出不等式的下面这个性质 性质9 如果a>b,c()(性质7) 就是 不等式的这个性质就是说:两个两边都是正数的异向不等式两边分别相除,得到和被除的不等式同向的不等式. 例4 已知一个圆和一个正方形有同样的周长,求证这个圆的面积大于这个正方形的面积. [证明] 设圆和正方形的周长都是p.那木 阅的半径品国的面积S-(会厂=会 义,亚方形一边的长Q宁,正方形的面积4(留). 。:匹<4,并且和4都是正数, ·69· ==========第80页========== 1>1 4 (性质8) 又 29 >0. p.1>.1 4匹44 (性质4) 就是 S>4. 这就证明了有同样周长的圆的面积大于正方形的面积。 习题 1.已知a+b>2√ab,b+c>2Vbc,c+a>2Vca,且&,b,c都 210 是正数,求证: ②) (a+b)(b+c)(c+&)>8abc. 2.知果a>b,c>见,并且b>0,d>0,能不能作出判断ac>bd?为什么? 3.如果u>b,c<,并且c,d都不等于零,能不能作出判断。>?为什么0 4.a>b,4,b同号,那末}和石之间有怎样的关系? 3.不等式两边分别自乘同次方和开同次方 在例3里我们曾看到,如果a>b,w和b都是正数,那末一定有3>3.很明显,这个结论可以推广到把不等式a>b(a>0,b>O)的两边分别自乘%次方的情形.这就是说,不等式有下面的性质 柱质10 如果>b,a,b都是正数,m是自然数,那末 a>b". 现在,我们来看如果把不等式a>b(a>0,b>0)两边分别开m次方,将会有怎样的结果.先来看几个具体的例 て 子 8>1,而/8>T; 16>4,而16>W4, ●700 ==========第81页========== 从这两个例子可以看到,把两边都是正数的不等式两边分别开同次方,得到和原来不等式同向的不等式,就是说,不等式有下面的性质: 性质11 如果a>b,a,b都是正数,是大于1的整数,那未 a>万, 这个性质不容易直按证明,我们改用间接证法,就是证明:如果这个结:仑不对,那末就会引出矛盾。我们可以这样想: /a和/6只能有下面三种关系之一: (1)减者/a>V; (2)或者Ya=b; (3)或者Vz<. 要证明第1种关系成立,只裙证明后面这两种关系都不成立.也就是要证明:如果后面这两种关系成立,就会引出矛盾 [证明] a和五之间的关系,只可能是下面这三种情况之 (1)或者&>V万;(2)或者a="万; (3)或者Va<五. 因为2,b是正数,所以a,石都是正数,如果=b,那么就有(W)=(石)",就是 a-b, 这和已知条件a>b矛盾,所以是不可能的. 果a力矛盾,所以也是不可能的. 既然这两种关系都不能成立,所以一定有 a>/6. 例5 求证√6+√7>2W/2+W5. [证明] (W6+W√7)3=13+2√42, (2√2+W5)=13+2W40, .:42>40,.‘.√42>W40. (性质11) .2W42>2W/40, (性质4) ●71● ==========第82页========== .13+2W42>13+2W4⑩, (性质3) .W13+2W42>W13+2/40,(性质11) 就是 W6+W7>2√2+/5, 把上面所讲的这些归纳起来,我们就可以得到下面这些关于不等式的运算性质: (5)(加法)a>b,c>d=→a+G>b+. (6)(减法)a>b,cb-d. (7)(乘法)a>b>0,c>d>0→0>bd. (8)(倒数)>b>0→1<1(⑨)(除法)a>>0,0<0日 (10)(乘方)a>b>0→a">b" (肌是大于1的整数), (11)(开方)>6>0→G>b (n是大于1的整数)不等式的性质 习题 1.比较(一2)”和(3)n的大小,这里是自然数. 210 [提示:分2是偶数和奇数两种情况来研究 ③)2.如果a>b,2是自然数,能不能作出判断αm>b"?为什么? 3。求证: (1)V3+W<4; (②)√2+V3<1o. 4.如果w>b,a,b都是正数,2是自然数,求证: a-nb,a,b都是正数,m,是自然数,并且>1,求证: 4>b号 §2-11一元无理不等式 在代数第二册里,我们知道被开方数中有未知数的方 ●72◆ ==========第83页========== 程叫做无理方程.类似地,被开方数中有未知数的不等式就叫做无理不等式.例如: Wx-2<2,W/2-6>3,W/3x-5-√e-4>0,等等,都是无理不等式. 一些简单的无理不等式,可以根据算术根的意义,利把不等式变形成有理不等式的方法,归结到解一元一次或二次不等式组来解。下面我们举几个例子来说明这种不等式的解法 例1 解不等式 √x-2<2. (1) [审题] 根据算术根的意义,要使√一2有意义,必须x一2≥ 0.在这种情况下,√x-2的值总不是负数.因此可以用两边同时平方的方法,把原不等式变形成一个有理不等式x一2<4.这样,解由不等式x-2≥0和心一2<4组成的不等式组,就可以得到原不等式的解。[解] 不等式(1)成立的条件是 x-2≥0, -2<4; ≥2, 就是 x<6 解这个不等式组即得原不等式的解集是2≤心<6.例2 解不等式 W√3-5-W-4>0. (1) [解] 由(1)得 /3x-5>√/-4, (2) 这个不等式成立的条件是 3x-5≥0, (3) 优4≥0, (4) 3c-5>u-4; (5) 0730 ==========第84页========== 就是 5 (6) () 1 (8) 解这个不等式组即得原不等式的解集是x≥4,注 从这个例子可以看到,如果不考虑在什么条件下式子Vx-5, V√红-4有意义,仅仅从V3x-5和Vc一4都不是负数,把不等式 (2)的两边同时平方,再解不等式(5),就会引进增解. “例3 解不等式 /2c-1>x-2. (1) 〔审题) 根据算术根的意义,可知女可取值的范围是≥受但是当≥号时一2的值不一定是正数。所以不能直接用 两边平方的方法去掉根号,而需根据心一2<0和心一2≥0的情况分别加以考虑.[解] 根据算术根的意义,可知2c一1≥0, 1°如果心一2<0,因为2x-1≥0,这时,不等式(1)成立的条件可以归结为不等式组 (I)2-1≥0, c-2<0. 解之,得 是<<2. 2°如果x一2≥0,把不等式(1)的两边平方得2cx-1>(-2),即x2-6心+5<0.这时不等式(1)成立的条件可以归结为不等式组 r2x-1≥0, (HI) 元-2≥0 (-1)(e-5)<0. 。74● ==========第85页========== 解之,得 2≤x<5. 原不等式的解巢,应该是不等式组(I)和(II)的解集 的并集,所以把1°和2°并合起来,即得不等式(1)的解集 是立5 〔注意〕 本题如果不分两种情况来考虑,就会失去一部分解, 习题解下列不等式:211 2.V2-x-3>0. 3.√3xー5>√+4 4.Vx-4-√3x-i>0. 5.Vx-6<√②. 6.Vx+8:-3>0. 7.V2xー1にxー2。 *8,/x2十x+1>x, §212不等式的证明 在前面我们做过的一些习题中,曾经遇到过证明一…个不等式是绝对不等式的问题.这类问题,象证明恒等式一样,具有一定的技巧性,而没有一般都可通用的方案.下面我们举例说明证明不等式的常用方法,同时还将引进儿个重要的不等式 例1 已知G和b是不相等的正数,求证: atb-ab. 2 我们用几种不同的方法来证明. (①)第-一种证法:把不等式左边的代数式减去右边的代数式,根据已知条件证明这个差总大于零. [证明1] +b-√a6=a+6-2Wa5 2 (1) 因为已知a,b都是正数,所以可以把a和b分别写做(√a)8和(√b),由此得 ●76· ==========第86页========== a+8-2Vab=(va)34(vB)3-2ab =(√G-6) (2) 因为a≠b,所以√a-√万是不等于零的实数, .(√6-W万)>0. (3) 从(1),(2)和(3)得 a+b-√i≥0. (4) 由此可知 营、元 注 这种证法,通常把它叫做比较法 (②)第二种证法:我们可以这样想,假定不等式 +b>√ab 2 (1) 能够成立,那末就可顺次推出: (맞)>(の)()>ab→(a+b)2>4ab =→a2+2ab462>4ab→(a-b)2>0 (2) 因为a,b是不相等的正数,不等式(2)是能够成立的.所以划果能够从不等式(2)反过来推出不等式(①),那末问题就解决了. [证明2] 因为a,b是不相等的正数,所以 (a-b)2>0, 就是 a2-2ab-b2>0, 在上式的两边各加上4ab得 a2+2a0+2>4ab, 就是 (a+b)24ab. 两边同除以正数4,得 (a+b) 4->ab 因为不等式两边都是正数,我们分别取它们的算术平 76· ==========第87页========== 方根,得 a+b>√ab. 2 注 这个证法通常叫做综合法.应用这种证法,我们先要设法找出 一个已知能够成立的不等式,然后逐步推出所要证明的不等式在这个例子里,为了找出作为出发点的不等式(a一b)>0,我们在证明前,先假定所要证明的不等式能够成立,然后逐步推出这个所需要的不等式,这一思考过程叫做“分析”. 比较上面的“分析”和“证明”这两个步骤,可以看到,在这两个步骤里,我们思考的顺序洽巧是相反的, 在应用综合法的时候,如果作为出发点的不等式能够直接观察出来,那末“分析”的步骤就可略去 本题如果把已知条件,改为α,五都是正数,就可得到 一个重要不等式 a+b≥√ab(a>0,b>0) (等号在=b时成立). 上附的不等式中,告2表示正数②,》的第术平均 数,ab表示正数a与b的几何平均数.这个绝对不等式说明:两个正数的算术平均数不小于它们的几何平均数例2 (1)求证:a2+b3+c2≥ab+bc+ca. (2)上式等号在什么时候成立? C证明] 因为任何实数的平方都不是负数,所以(a-b)2≥0,(b-c)2≥0,(c-a)2≥0. 等号仅在a=b=c时成立. 因此,a2+b2≥2ab,b2+c2≥2bc,c2+a2=÷2c%.把这三个不等式的两边分别相加,得 2a2-263+202>2a6+200+2ca, 77 ==========第88页========== 两边都除以正数2,得 a2+b2-i.c≥ab+t0+ca. 等号仅在a=b=c时成立.例8 如果a和b都是正数,或者都是负数.求证: 等号在什么时候成立? [证明] 不论a和b是什么实数,都有 (a-b)2≥0, 就是 a2-+b2≥2ab. 因为两个正数或者两个负数的积总是正数,所以αb是正数.以正数ab除上式的两边,得 22, ab 就是 等号在a=b时成立.注 本题证得的不等式,可以作为定理来应用,意思就是说:一个正数与它的倒数的和不小于2. 从上面这些例子的解答过程中,可以看到要证明一个不等式是绝对不等式,常常要用到: (1)实数的性质:1°a2≥05 2°a2+62=0→a=b=0. (2)实数的运算比较性质: >b, [>0, a{=b,←→a-b=0, 2-1; (2) 2a ①) 1+1, 2.已知x是不等于1的正数,求证: x+1>2 3.已知a,飞是不相等的.正数,求证: (1)(u+b)(1+-1)>4;(2)ax3+b3>a2b+b2, (3)(+b)(+b)w3+b3)>8ab3 4.求证: a2-x+1星: (2)1-x-x2≤11 5.已知a、b、都是正数,并且a√a+Vm; 2 (②)n+6+e+d≥a0. 以上这两个不等式中,等号在什么时候成立? 例4 已知a、b、c都是正数,求证 a3+-b3-+c3≥3abc. 上式中等号在什么时候成立? [审题] 本题可以利用因式分解的公式a34-63+c3-3abo =(a+6+c)(a24-63+c2-ab-bc-ca) 来证明。 [证明] ,°a3+-☑8+c3-3abc =(a+6-G)(a2+62+03-a8-6c-ca) 受(の++(a-b)"+(6-)+(ーの(1) •79● ==========第90页========== 今a、b、c都是正数,所以a+b+c>0.又(a一b)9+(b-c)3+(-a)≥0(等号在a=b=c时成立).所以由 (1)可知 a3+b3+c8-3ubc≥0. ..a8+b3+c3≥3abc(等号在a=b=c时成立).在上面这个不等式里,如果把a、b3、c3分别用a、b、c来代替,并以3除两边,就可导出一个重要不等式: atbtcabc (a-0,6>0,c-0) 3 (等号在=b=c时成立). 这个不等式指出:三个正数的算术平均数不小于他们的几何平均数.一般地,可以证明(证明略)以下的定理:定理 n个(≥2)正数的算术平均数,不小于它们的几何平均数.即 a1+a购十+n≥/aag…an, 亿 这里1,ag,,an都是正数,等号在a1=g=…=a,时成立 例5 已知a、b、c是三个互不相等的正数,他们的乘积是1,求证: (1+a+b)(1+b+c)(1+c+a)>27, [审题] 根据上面这个重要不等式,可知当≠b时必有 1+a+b>/币.因为左边三个因式是a6、c的对称式,3 所以同理可推得1+台牛e>b,L+g+a>ya.把三3 3 式两端分别相乘,再应用abc=1这一条件,就可得到证明. [证明] 因为a、b、c是互不相等的正数,所以 ·80● ==========第91页========== 1+a+6>ya6,1+6+0>3,1+8+a>/a.3 3 不等号两边分别相乘得 (1*g)(+3ー으)(+g+)>マa 又因bc=1,代入上式,得 (1+a+)(1+6+)1+0-+)>1. 27 .'.(1.+a+b)(1÷b+c)(1+c+a)>27 例6 已知三个正数a、b、G的和是1,求证这三个正数的倒数的和必不小于9. [审题] 本题就是要在a>0,b>0,c>0及a+b+c=1的条 作下明++0一个对不学式 [证明] 根据题设条件a十b十c=1,可以推得 3+(응+)+(움+)+(+) (1) .'a>0,b>0,c>0, ++2。응+을>2(+룸)-(움)+(+)6 (2) 由(1)和(2)可知 공공+공> (证毕) 注 本题也可从证明是+号+-9>0若手,证明留给读者。 从上面这些例子中,可以看到,在证明不等式的过程中,我们也常常要灵活地应用已知的恒等式和某些重要的绝对不等式。 81 ==========第92页========== 习题 1.已知+0,求证: 212 (0+山 (2)2+3 >2, ②) Vx3+2 2.设&、b、c是互不相等的正数,求证: ①)b+g-&+c+2-b-u+6=e>3; (2)(a+6+c)(ab+6c+ca)>9abc 3.已知、b都是正数,求证: (1)(1.+a+b)(1+a2+b2)≥9ab, (2)(am6+a+b2)(cb2+2+b)≥9ax262 4.已知4,b,C,C,y,?都是正数,且 증~품용延증<+능 5.已知a2+b2+c2=1,c2+y2+2=1,求证 ax+by十c2≤1, *§2·13代数式的最大值和最小值 不等式的重要应用之一是可以利用它来求某些代数式的最大值或最小值.例如,上节例6中我们已经证明了三个正数a、乃、c的和如果等于1,那末它们的倒数的和必不 小于9,脚台+号-名9,这里等号当a=-时城立.这就指出了当0+b+01时,代数式是+名+是的袋小a 值是9.如果进一步追问当a、b、c是什么数时,才能取得最小值,那末就可根据这个不等式只有当a=b=c时等号才能 成立这一论断,从已知条件a++e=1推得a=6=0=景 象这类问题就属于求代数式的最小值或最大值的问题 应用不等式求代数式的最大值或最小值,最常用的方法有两种:一种是利用§2.12里引进过的%个正数的算术 ·82· ==========第93页========== 平均数不小于它们的几何平均数这一定理;另一种是应用 一元二次方程的根的判别式.下面我们分别举例说明. 1.利用算术平均数和几何平均数的性质,求代数式的最大值或最 小值 我们知道:如果a,飞是两个正数,那末 2"≥a6. a+b、 等号在a=b时成立.由此可以推得如果a,b是正数,ab=P是一个常数,那末当a=b时,和a+b就有最小值2P;相反的,如果a、b是正数,a+b=S是一个常数,那 木当口6时,积0d也额有最大位答。 同样的,如果a、b、c是三个正数,那末 a+b+e≥3abc. 3 等号当==c时成立.由此可以推得 1°如果a、b、c是正数,abc=P是常数,那末当a=bc时和a+b+c有最小值3P. 2°如果a、b、c是正数,a+b+c=S是常数,那末 当a-b=c时积a6c就有最大值7 利用上面的结论,可以解某些求最大值、最小值的问题. 例1 当取什么数值时,代数式4÷号有最小值,这个 最小值是什么? [审题] 因为4、号都是正数,且((9)36是个常数, 所以可以应用生≥V@这一不等式来解, ·83· I. ==========第94页========== [解] 4,9都是正数, 9 4x2+-2 2 =V36=6, 即 4x2+9、2≥12. 上式中等号在4x=9时成立.解方程42一9 函,得x=土√6 2 由此可知当x=±6 时,42+9有最小值12. 例2要把一定量的汽油装在长方体形状的铅皮箱里,这个容 器应该怎样设计,才能使所用的材料最省?[解] 设长方体的长、宽、高分别是、y、名,那末它的容积就是V=y必所用材料的面积S=2(y+2+). 现在V是一个定值,x,y,2都是正数.要使S最小,只需y+yz十最小. y+y2+≥3/心y2=/7T. 3 上式中当y=y%=时,也就是=y=2时等号成立。 ∴.当=y=名时,y十y2+有最小值3/V产. 从而可知,这时S有最小值63 所以容器应该设计成一个长、宽、高相等的正方体,才能使所用的材料最省. 2。应用一元二次方程根的判别式,求代数式的最大值或最小值. 我们知道,在实数集内的一元二次方程 ax2+bx÷c=0(a≠0) 当它的判别式4=b2-4ac≥0时,有两个实数根(相等的或 0.840 ==========第95页========== 不等的),其逆命题也成立.这个结论也可以说成: 如果实数系数的方程aw2+b心十c=0(a≠0)有实数根, 那末的最小值是4ac,或者a0的最大值是车, 利用这一性质,我们可以求某些代数式的最大值或最小值. 例3 取什么值时,分式心十有最大值或最小值? 2-+x+1 最大值或最小值是什么?[解] 设这个分式的值是y,即 2+4+1 02+花+影 那未就有 +4ax+1=y(2+化+1), 即 (则-1)2十(y-4)x+(gy-1)=0. 因为化,都是实数,这个关于如的二次方程有实数根, ∴.4:(y-4)2-4(y-1)2≥0. 即 2-1≤0,∴.-2≤y≤2. 由此可知y的值在:一2与2之间. 令g-2,即计-2,解之得-1,由 02+8+1 此可知当=一1时,这个分式有最小值一2。· 令一2,即-2,解之得1.由此可知B 当x=1时,这个分式有最大值⑦ 例4 由沿河的城市A运 货物到另一地点B,B到 河岸的距离BC是30公 里,从A城到C点的距离 40 是40公里.如果水路运 图222 费是公路运费的一半,应该从B点怎样筑一条公路到河 ·85· ==========第96页========== 岸,才能使由A到B的运费最少? [解] 图222里,4C表示河岸,BD表示求筑的公路, BCAD. 设AD=龙公里,那末DC=(40-)公里, BD=√厂(40-x)3+303公里, 因为公路公里的运费相当于水路每公里运费的2倍,所以总的运费是水路每公里运费的[x÷2√(40一)3+30]倍。 设 y=龙+2N(40-2x)+30巴, (1) 现在要求使y的值是最小的的值. 由(1),得 yーz=2√(40-x)2+303, 两边分别平方,整理后得 3x2-2(160-y)x+(10000-y)=0. (2) 解(2),得 x=160-y±√160-0-3(10000=y万, (3) 3 因为x必须是实数,所以 (160-y)2-3(10000-y2)≥0, 就是 y2-80y-1100≥0. (4) 因为y必须是正数,解(4)得 y≥40+30W3. 由此可知y的最小值是 y=40+30N3. 代入(3),得 -号160-(40+808)1-40-10V月≈28. 答:应该在AC间取离A约23公里的一点D筑公路 BD,才能使A到B的运费最少. ◆86 ==========第97页========== 习题 1.当龙取什么值时,下列各式有最小值或最大值,并求出这个最小 213 值或最大值: (1)a+x+6 (b>0,x>0; (2)x4-g2+4 (3)(ax-b)(c-ax) (4)(Vx+1-2)(6-V√x+1)30,求证”+2:+号的值不小于6 (2②)6是不是2+2x+是(x>0)的最小值?为什么? 3、当x取什么值时,下列各分式有最小值或最大值,并求出它的最小值或最大值: ()2x+1 (2) 3x 22+x+13 02+2x+1, 4.分20为两个正数,使他]的平方和为最小 5.下水道的截面是一个矩形上面加一个半圆,周界都用砖砌成,当截面积$一定时,矩形的底边和高应该设计成怎样的比例,才能使所用的材料为最省? [提示:设矩形的底边为x高为y,在 S=ッ+ 为定值的条件下,求x和y成怎样的比例 2 时,周长最小.门 6.建造一面靠培的长方形小屋一间,面积为 (第5题) 24平方米,房屋的正面用木板修建,平均每米造价为4a元,房屋的两个侧面用土墙,平均每米造阶为3a元,问这房屋应怎样设计,才能使造价最省?造价最省要多少? §214含有绝对值的不等式 在代数第二册里,我们曾经学过可以化为一元一次不 ·B7· ==========第98页========== 等式来解的某些简单的含有绝对值的不等式的解法,并且知道: |x>a(a>0)的解集是{<一a或>a.(①)|x0)的解集是{x一a&>0) |xl0) 图2.23 关于含有绝对值的不等式的问题,也有两类,一类是解不等式,另一类是证明它是绝对不等式.下面我们分别来讨论这两类问题. 1.含有绝对值的不等式的解法 解在绝对值符号内含有未知数的不等式,关键就在于去掉绝对值的符号,把它转化成一个普通的不等式。主要的依据是绝对值的意义: 如果a>0, 0 如果a=0, 如果a<0. 射果给定的这个不等式里,只有一个绝对值符号内含有未知数,那末这类问题,就可以直接应用上面(①)和()这两个基本不等式来解.例1 解不等式 (1)3.-12-x. [解] (1)根据绝对值的意义,得 (x+2)<3ー1<+2, 3c-1>(c+2), 就是 3c-1-1, 也就是 2x<3. 解这个不等式组即得原不等式的解集是 -<<4 (2)根据绝对值的意义,得 3-1>2-x,解之得 4 1 或 3r-1<-(2-c),解之得<一 所以原不等式的解集是 或公景 注 从上面的例子,可以看到利用基本不等式{xn{la<)-{-급a导出的则是两个独立的不等式,所求的解集应是这两个不等式的解集的并集例如在(②)中,所求的解集是 ri>}ua<ー}ー{이ー或>] 这一点,在解题时必须充分予以注意. 例2 解不等式x2-2c-4<1. [解 原不等式可以变形成一1<2-2w一4<1,也就是不等式组 [o-2x-4>-1, 2-2-4<1. 即 a2-2c-3>0, (1) 2-2-5<0. (2) 解不等式(1)得 ==========第100页========== c<一1或x>3. 解不等式(2)得 1-w61. [解] 原不等式可以变形成两个独立的不等式: x-2-3<-1, (1) /3w-2-3>1. (2) 由(1) 3x-2<2→ -29→景<2 r3w-2≥0 由(2) N3x-2>4→ →x>6 3a-2>16 因为能使不等式(1)或(2)成立的:的值,也都能使原不等式成立,所以原不等式的解集是两个区间 [学2)U6,+), (2) 2 6 图225 ●0e ==========第101页========== 注 在解题时,只要说理清楚,可以选用不同的叙述方法。以上两例,都用区间符号来表示不等式的解集,这种表示方法,今后常要应用,也应熟悉 从上面所举的这些例子,可以看到解绝对值符号内含有未知数的不等式,关键就在于利用绝对值的意义设法去掉绝对值符号,把它转化成一个普通不等式或不等式组.掌握了这-一点,我们也就不雄解其它一些比较复杂的含有绝对值的不等式. 例4 獬不等式:|2x+1|+|3x-2≥5 [审题] 根据绝对值的意义,有 x2x十1, 如果 12x+1=-2+1, 21 如果 1 ≤ 化≥ 23x-2,如果 3-2=1- 3 3c-2),如果 2 为了去掉绝对值符号,应该把实数集分成三个区间,即 2 然后分别处理 [解] 原不等式可以转化为三个不等式组: 化≤一 1(1) 21 即 2 1-(2x+1)-(3x-2)≥5; -5x+1≥54 1 1-2 r≥一 (2) 即 2 (2c+1)-(3c-2)≥5; 3 一x十3≥6, ·.91●. ==========第102页========== 2 (3) 3 即 3 (2-1)+(3一2)≥5; 5w-1≥5, 解不等式组(1)得 4 ≤一5 不等式组(2)元解;解不等式组(3)得 6 5 原不等式的解集应是列出的各个不等式组的解集的并集。所以原不等式的解集是 (-∞,-]u[g+). 注 划分区间时,交界处的一点可以刘入相邻的任一区间,也可以同时划入相邻的两个区间, 习题 1.解下列不等式: 214 (1)2i2c-1>.1; (2)41-3ac|-1≤0; の끊~ 国|会 2.解下列不等式: )小5 (2x2-1>x+2. 3.求能使下列不等式成立的最小正整数: (2) 2122 1 382+1-7<1000 4,解下列不等式: (1)12x-5이-14x+0(2)x-2一+1}+3>0, 2。含有绝对值的不等式的证明 证明-个含有绝对值的不等式是绝对不等式,除掉要应用一般不等式的那些性质以外,经常还要用到关于绝对值的和、差、积、商的性质: ◆820 ==========第103页========== 性质1 a+1이≥1a+1 性质2 lal-1|a+이 (3)如果和b中有一个是零,或者两个都是零,这时很明显地有 la+=ll+1이。 就是 a이|+|1=la+1. 综合上面这三种情况,就得到 a|+|b|>la+이。 (证毕) ·93◆ ==========第104页========== 根据性质1,有 a+이+|->la+b-이, 就是 la++|이|]。∴。lal-ba+(-b)1. 但是|-이=|,所以lal+||>la-이,就是 la-이<+|。 (1) 其次,证明|이|이0,1+a+b>0, 两边除以(1+a+b)(1+a+b),得 1ーTa+6 (3) 但是 a+이c>0,求证 5.已列1a|<1,b|<1,求证 a+b I-ab<1, 本章提要 1.等式和不等式 名称 等 式 不等式 用等号“=”把两个代数式连接起用不等号“>”或“<”把两个代数定义来所构成的式子做等式. 式连接起来所构成的式子叫做不等式. (1)如果a=b,那求 (1)如果a>b,那末 飞=。 bb,b>c,那求 a-C. a>c. (》如果=b,那米 (3)如果a>b,那末 4十c=b+C, a+c>6+c, (4)如果:=b,那末 (4)如果a>b,那末 ac=bc. >bc(c>0);ac=ba (c=0);{b,c>d,那米 a+c=b中d。 a+c>b+@ 质 (6)如果4=b,c立风,那术 (⑥)如果a>b,cb-d。 ·97· ==========第108页========== (续表) 名称 等 式 不等式 (7)如果a=乃,c=d,那米 (7)如果a>b,c>d,a,b,c,d uc=bd. 都是正数,那末 性 ac>bd. (8)如果u二乃≠0,那术 (8)如果a>b,a,b都是正数,那 11 未。 ab· 合合B 9)如果a=b,c=风+0,那末 (9)如果a>b,G (10)如果a=,%为自然数,那末(10)如果a>b,a,b都是正数,% an=n 为自然数,那末 an>tn (11)如果a=,&,b都是正数,%(11)如果a>b,a,b都是正数,”质 为大于1的整数,那末 为大于1的整数,那末 a=万, /a>"6. *用符号“≤”或“≥”把两个代数式连接起来所构成的式子,也称不等式.这种不等式具有等式和不等式所共同具有的性质. 2.不等式的证明 (1)应用比较法则 >0, >b, a-b=0,{=b, <0 b(a≠0). 1°如果a>0,解集是{i>名},闻这间(会,+c)片2如果a<0,解集是{z<},即区间(-心,合)*在一般情况下,只需求出解的限制条件x>名(或者 <8),即作为问题的答案。 (2)一元一次不等式组只需求出不等式组中各个不等式的解集的交集 元一次不等式组的解集的几种基本情况(α [ma f比9 la9} {xr<} {xa<助<} e (3)一元二次不等式 1一元二次不等式(w-a)(筋-)>0或(=a)× (心一B)<0的解法 ●89● ==========第110页========== 类型 (c-a)(-B)>0(a<) (-a)(B-β)<0 (a0 fa-a<0 x>0 和 化-a<0 等式组 (x-B>0 lxーβ<0 i-8<0和-B>0 解巢 {xGB} Yoa0)的解集 (a>0)的解集 1=-b-√万 .2a >0 {x<1或x>} {x<花<物} 2=二b+√☑ 2a 1=0 6 b 饥心2=2a <0 没有实根 实数集R (4)一元高次不等式一般可利用列表法来獬(⑤)一元分式不等式可以转化为一元不等式组来解,也可以转化为一元整式不等式来解. 类型 P(>0 P(x)<0 (x) Q(x) 归结成不 P()>0 rP()<0 rP()>0 「P()<0 和 和等式组 2()>0 l2()<0 IQ(x)<0 Q()>0 结成整 P(x)Q(w)>0 P(x)()<0 式不等式 表中P(c),Q()分别表示心的某一多项式. (6)一元无理不等式一般可根据算术根的意义并应用不等式的性质11,转化为一元整式不等式组来解. 0100◆ ==========第111页========== 儿个简单无理不等式的解法 类型 √P()ンa(a>0) √F(c)>√/Q() 归结成不 íP()≥0 P()0 等:北组 ()≥0 P(a)>a P()>e() ()含有绝对值的不等式简单的问题可以直接根据绝对值的意义来解. 类型 jwl>a x& 就-bl0优<-6或B>化 -ab十a -0 {,x*0} 0 {优+b} 0 集a0,b>0.呢?(②)如果ab>0.把下列各式依照从小到大的顺序用不等号“<”连接起来(要说明理由): ab,,va西,/a2+b3 2 4.设a+b,求证: (1)a2+3b2>2b(a+b); (2)a4+62b2+b4>4ab(a2+b2). 5.设a,b,c是互不相等的正数,求证: (a++a)(능++)> (2)(ab+x+b+1)(ab+4c+bc+c2)>16abc, 6.已知a>b>0,求证 (1)Va-V/万중-a(>0,6>0 ·102· ==========第113页========== (2)a+r<-b+2(a,b同号). 8.解下列各不等式组: (1)(x-3)(-4)<(x中1)(x+2), x(+1)+(c+2)>(2r-1)(c+3):5+2>1-9- (2) 1 2-(47-四)》8 9.解下列各不等式: (1)(6-3)(x-7)<5(-3);(2)x2-5x>3c+4; (3)(心-V/2)(2x+V2)>1; (4)(x-V②)2<1; (5)(x-1)3>(x-1)3, (G)(1+V2)(1-V√2x)>4x.10,解下列各不等式: (1)+1 +4>1; (2)+1 92+4>0: ()是0: (4) x+W/2 2+40; (2)(2x+1)(3x+1)(4x+1)<0 12.解下列各不等式: (1)(3xc+2)(2+3》>0; (2)+10, (x+2)2>1, (1) (2) l(x+2)(x-3)<0: l(x-2)<1, 14.解下列各不等式: (1)3x+5+V5x-3+3<0: (2)V3x+5V5c-27; @品*是是6闭1 2.解下列不等式(答案只需写出c应满足的条件): ()3r+2>2-5-6 92-58-623x4-2 (2)(c1)(x-+3)(2÷5)(x+7)+15>0: (3)vx2-3玩-2->3; (4)c+1{÷x+2!>2:+31. 3.解下列关于x的不等式(解答要进行讨论): (1)2a2+(4级+6)x+(22+3+9)<0: ·1040 ==========第115页========== (2)(u+2)x2+2(a+1)x+(a-1)>0.4,求下列各不等式组的整数解的集合: 4 2 59m-1 fx2-1<3, (1)十3r-3 2-9’ (2) 5-3 lv3a:-9-√z>0; 4+.73 5.逆明下列不等式: (1)(a4b2+bc2+c4a2)(2b4.↓-b2c4+c2a4)≥9a464c4; e,(g-会8启+云)小8 (40,b>0,且a÷b) 6.(1)已知1a2+b2=1,x2+y2=1,求证:|ax+by|≤1. (2)已知41,ag,,…,an都是正数,且a1a2ag…an=1,求证 (1+a1)(1+a2)(1+g)…(1+n)≥2n 7.从火车上下来两个旅客,他们沿着同一方向走到同一地点.第…个旅容一半的时间以速度a来行走,另一半的时间以速度b来行走.第二个旅客一半的路程以速度a来行走,另一半的路程以速度b来行走(a≠b).问哪一个旅客先到达终点. 8.已知扇形的周长是40cm,求这个扇形的最大面积.这时扇形的半径长多少? 9.设△4C的三条边为a,b,,求证 b+bc+caxa2+624-02<2(ab+bc+ca). [提示:不失一般性,可设a≥b≥]. 10.证明: (1)在给定周长为2p的三角形中,以等边三角形的面积为最大: (2;在给定面积为S的三角形中,以等边三角形的周长为最小; (3)如果三角形和正方形的面积相等,那末三角形的周长一定大子正方形的周长. 第二章测验题 1.解下列不等式: x2.-20-15 (1)30-11x-2x2<0; (2) 必-2 ≥0; •105· ==========第116页========== (312-a1<7 (4)1Vx-3-21>1. 2.已知a、五、C,都是实数, (1)求证:方程x2-(+b)a:+a6一c2=0一定有实根; (2)求这一方程有等根的条件, 3.求下列不等式或不等式组的正整数解: (2)2x+31. c2一x-6产0, 2一优 2)2-s-42<0. 4.(1)已知a>1,求证:a3+1>a+; (2)已知a,b,C,d都是正数,且有 ェ=√a2+b2,y=√c2+aP 求证: cy>√(ac+ba)(ad+br. 5。有两块布料,第一块的长度恰巧是第二块的子.如果每一块布 料各剪去8尺,那末第二块的长度比第一块长度的5倍还要多 一点。求这两块布料各长多少。 ·1ag4 ==========第117页========== 3 函数的初步知识 函数是数学里一个重要的概念.应用数学知识来解决各种问题,经常要用到函数的知识.在本书里,我们将学习关于函数的一些基本知识.这一章先学习关于函数的一些初步知识,包括什么叫做函数,怎样确定一个函数;同时还将学习两种常见的最简单的函数一正比例函数和反比例函数的图象和性质。 §3·1常量和变量 1。量和数 在日常的生活、生产以及科学技术的研究中,我们经常会遇到各种各样的量.例如:重量、时间、长度、面积、体积、温度、速度、产值等等都是量. 各种不同的量有着一个共同的性质,就是它们都可以用一个取定的同类的量作为度量单位来量它的大小.例如,重量可以取1克、1公斤、1吨等作为度量单位;长度可以取1厘米、1米等作为度量单位;时间可以取1秒、1分、1小时等作为度量单位, 表示一个量和度量单位的比的数目,叫做这个量的数值,或者叫做量数,它就是度量的结果.例如,用1厘米作为度量单位,量得一条线段的长度是5厘米,这里数5就是这条线段的长度这个量的数值, ·107· ==========第118页========== 2.常量和变量 我们所研究的量,虽然都可以用数来表达它,但是在问题的研究过程中,它们却可以有不同的性态。在问题的研究过程中,有的量始终保持着同一个数值,但是有的量却可以取某一范围里的不同的数值. 我们来看下面的一些例子 (1)铁的比重①是7.8克/厘米8.5立方厘米的铁重 儿克?10立方厘米呢?V立方厘米呢? 这个问题可以利用下面的公式来解: 物体的重量=单位体积的重量×体积. 如果用字母W表示物体重量的克数,那末有 5立方厘米的铁重是W=7.8×5=39(克), 10立方厘米的铁重是W=7.8×10=78(克), V立方厘米的铁重是W=7.8×T':=7.8V(克). 这里我们可以看到,在问题的研究过程中,铁的比重是不变的,它只能取数值7.8,但是铁的体积和重量却是在变化着的,它们可以取正数集里某些不同的数值 (②)一列火车从甲站开往乙站.在离开甲站2公里 后,用每分钟号公里的速度前进,那末从这时起经过1,之,…,10分钟后火车离开甲站的距离是多少? 设从这时起,经过t分钟后,火车离开甲站的距离是§公里,那末有公式 2 -2+중 ①一个物体单位体积的重量叫做它的比重.在物理学里,比重也是一 个置,用“克/厘米”做单位.例如铁的比重风记做风=7.8克/厘米,意思就是1立方厘米的铁重是7.8克, ·108· ==========第119页========== 这样就容易算出: 1分钟后,8=2+号×1-2号(公).2 2分钟后,8=2+号×2-8号(公里).2 10分钟后,8=2+号×10-8号(公里.在这个问题里,火车行驶的速度号公里/分以及原来 离开甲站的距离2公里这两个量是不变的,但是火车行驶的时闾t分和最后离开甲站的距离8公里这两个量是在变化着的,专可以取集合{1,2,3,…,10)里的不同的数值,而8则可取正有理数集里的某些不同的数值 为着区别这两种性态不同的量,我们把在问题的研究过程中,始终取同一个数值的量叫做常量,在问题的研究过程中,可以取不同数值的量明做变量. 从上面的例子中可以看到,变量虽然在问题的研究过程中,可以取不同的数值,但是它可以取的数值,往往有着 一定的范围,变量可以取的数值范围叫做这个变量的可取值范围,或者容许值范围.例 在下面的这些代数式中,字母x的可取值范围是什么? (1)x2-1: (2)3-1 1 (3)√x+1; (4)7 +1 [解] (1)因为x不论取什么实数值,代数式x2-1都有意义,所以x可以取全体实数的值. (②)因为在分式中,分母的值不能是苓,所以2一1≠0,就是心≠士1,这也就是说心可以取一切不等于士1的实数: ●10ee ==========第120页========== (3)因为在根式中,被开方数必须是正数或者零,所以 x+1≥0,就是≥-1. 这也就是说,心可以取不小于一1的一切实数. (4)这里,分母的值必须是正数,所以√c十1>0,由此可知 x+1>0,就是x>-1. 这也就是说,x可以取大于一1的一切实数, 习题 1.举出常量和变量的一些实例. 31 2.在下列这些公式中,娜些是常量?哪些是变量? (1)圆周长公式: 0=2tT, 其中C表示圆的周长,”表示圆的半径。 (2)球体积公式: v=4 其中V表示球的体积,?表示球的半径。 (3)匀速运动公式: 8t, 其中s表示距离,y表示速度,表示时间, (i)用来计算物体以同一速度运动,不同时间内所行的距离 时; ()用来计算在同一时间内,按不同速度运动的距离时, 3.下面这些代数式中,字母的可取值范围各是什么? (1)x8+2x2+3ax+4; (2)x+1 (3)V1-; (41- §32函 数 1.函数的意义 我们再来研究上节举过的那些例子。在例()里,两个 01100 ==========第121页========== 变量W和V之间存在着一定的关系,就是,当变量V在 正数集里取定一个确定的值的时候(例如5),按照计算物体重量的公式,变量型也随着在正数集里有一个确定的值和它对应(图3·1). 乘7.8 同样的,在例(2) 39 中,当变量t在集合 D{1,2,3,…,10} 78 里,取定一个确定的值的时候,按照匀速 w(=7.8V) 运动的距离计算公式变量8也随着在正数集里有一个确定的值 正数集 正数集 和它对应(图32) 图31 在数学里,我们把两个变量间这种依从关系,叫做函数关系。 仔细观察这些例 23倍后再加2 子,还可以看到,在同 3 一个问题里的两个变 2 3 量,在问题的研究过程中,所处的地位是不同的.例如在例(1) 6 中,我们首先考虑的 3 是变量V的变化,然 D={1,2,8,…,10} 正数集 后根据它所取的值来决定这时另一变量 图32 W的值;在例(②)中,我们首先考虑的是变量t的变化,然后根据它所取的值来决定这时另一个变量$的值.我们把问题研究过程中处在前一种地位的变量,叫做自变量,而把处在后一种地位的变量,叫做因变量,通常也说这一变量是前 0111● ==========第122页========== 一变量的函数.例如,花: 例(1)中,V是自变量,W是因变量.通常也说W是 V的函数; 例(2)中,t是自变量,s是因变量.通常也说8是t的函数. 一般的,设在某一变化过程中有两个变量心和y,如果对于心在它可取值的范围里的每一个确定的值,按照某一对应法则,都有唯一的值和它对应,那么就把叫做自变量,y叫做因变量,通常也说是心的函数 例如,在等式y=心2一1里,字母心和y表示两个变量,(这种字母,我们把它叫做变量字母),当心在实数集里取定任何一个值时,按照等号右边这个代数式所指出的计算法则(平方后再减去1),也就有唯一确定的值和它对应,如下表: 3 -1 -圣 所以这个等式也就确定了变量y和x间的一种函数关系,这里心是自变量,y是因变量.通常我们也把这个关系说成y是心的函数 例 已知在匀速运动中,物体经过的路程8(公里),速度(公里/小时)和时间t(小时)之间有下面的关系: s=vt (1)如果速度不变,这个式子表示什么?哪一个字母表示自变量?哪一个字母表示因变量? (2)如果时间不变呢? (3)如果路程不变,怎样表示速度是时间的函数?怎样表示时间是速度的函数?式中的字母各表示哪一类量? p1120 ==========第123页========== [解] (①)表示路程8是时间t的函数,t是自变量,8是因变量, (2)表示路程s是速度心的函数,w是自变量,s是因变量. (③)速度是时间的函数,可以表示成 光’ 其中t表示自变量,表示因变量,s表示常量。 时间是速度的函数,可以表示成 t=8 其中)表示自变量,t表示因变量,8表示常量. 习题 1.轮子每分钟旋转60转,写出轮子旋转的转数n和时间之间的 32 函数关系: (1)把时间t作为自变量;(②)把转数%作为自变量. 2.两个变量x和y是用下面的关系式联系起来的.试用c的代数式来表示y,在这些代数式中字母如可取值的范围有什么限制? (1)3x-4y=12; (2)y=6; (3)y一x3=0; (4)"=0; (⑤)x=y+1. y-1 (6)(+2)(y-3)=6, [解法举例:(1)3x-4y=12,4=3xc-12。 g=3-12 4 心的可取值范围是一切实数,即实数集R.] 2.函数的符号 在研究两个变量y和c间的函数关系时,语句“y是的函数”通常可以用式子 y=f()或者则=F(c) 等等来表示,这里字母∫或者F等等叫做函数符号,它不 •1180 ==========第124页========== 代表任何数,面只代表把变量y和龙联系起来的对应法则,也就是怎样根据自变量心的值来求出和它所对应的因变量y的值的法则 例如,我们知道,圆的周长C,面积S和它的半径?之 间有下面的关系: 0=2rr,S=元ra. 当半径?在正数集里取定任何一个值时,圆的周长0和面 积S分别有唯一的值和它对应,所以圆的周长O、面积S 都是圆的周长”的函数.说明这一事实,我们可以把它们分别表示成 C=f(r),S=F(r), 这里∫就表示“乘2π”这一对应法则,面F则表示“平方后 再乘π”这一对应法则 从上面的例子,可以看到式子y=∫(花)只指出了y是x的函数这一事实,但是这两个变量究竟是怎样联系起来的,只从这个式子是看不出来的;另外,当我们同时研究几个函数时,如果变量间的对应法则不同,那末就要用不同的字母来表示对应法侧 过去我们在学习代数式时,就已知道代数式的值是由它所含字母的值确定的.代数式里字母的值,只要不使这个代数式失去意义,可以任意选取,当取定以后,通过计算就可以求出这个代数式的唯一的一个值和它对应.所以每 一个含有字母的代数式都可以说成是它所含字母的函数,并用函数的符号把它表示出来,例如 1 c-2 是a的函数,可以记作了)-g w一1是t的函数,可以记作g(t)=W√-工. 3。函数的定义域 在一个函数中,自变量虽然可以取各个不同的数值,但 01140 ==========第125页========== 是它可以取值的范围是有一定限制的.例如上面提到的这两个代数式所确定的函数,自变量就有不同的取值范围. (①四)对丁函数1似-之来说,自变后之只能敢一切不等2的实数:当=2时,分式习没有意义). (2)对于函数g()=√一1来说,自变量t只能取 一切绝对值不小于1的实数(.‘当<1时,根式W一1 没有意义). 函数中自变量可取值的范围,叫做函数的定义域 一般来说,函数的定义域是由我们所研究的这个具体问题来确定的,在给出这个函数时,应该把函数的定义域指出.例如,在研究圆的面积$和它的半径?间的函数关系时,因为”只能取一切正数,所以这个函数应该表示成: 8=πr2(r>0)或者8=匹m,r∈(0,+∞),这里>0或者”∈(0,十∞)就表示了这一函数的定义域 但是,在一般地研究一个由数学式子所表示的函数 y=f(a) 时,如果没有特别的声明,我们总认为函数的定义蜮就是使∫()有意义的一切实数的全体,为了简便,就不一定要把它的定义域写出 例 求下列函数的定义域: 1 (1)y=c2-4+3; (2)则=2-4+35 (3)y=/25-x2, [解] (1)因为心不论取什么实数,一4c+3都有意义,所 以这个函数的定义域是实数集R 1 (2)要使分式-十名有意义.必须而且只须c24x3≠0,就是(知一1)(x-3)≠0.由此可翔,一1和c一3这两个因式都不能等于0。·.x≠上,中3. 年b15e ==========第126页========== 这个函数的定义域是除去1、3以外的所有实数组成的 集合,即(-∞,1)U(1,3)U(3,+∞). (3)要使根式√25-2有意义,必须而且只须25-x2≥0,解这个不等式得一5≤x≤5. 所以这个函数的定义域是{|一5≤≤5]. 注 从上面的解答中,可以看到函数的定义域,象不等式的解集一样,也可以用不同的形式来表示.例如,(①)的答案也可以说成函数的定义域是区间(一∞,+∞)或用不等式表示成一∞f(x2); (2)当0<1<2时,f(x1)0的情况. 例如我们取台=是,=1,=2等值,在同一直角坐 标系里作出下面这三个正比例函数 (1)y-1 , (2)y=x, (3)y=2的图象(图3·15). 从图象中就可以看出: (①)这些直线都在第一、第三象限内,并且都是从左到右向上伸展的、这-·事实,就告诉我们,正比例 y=2x 函数y=太(k>(D)的值是随着自变量的值的增大而增大的 ·(2)这些直线关于¢轴的倾斜程度是随着比例系数无的变化面变化着的,越小,直线越 图3.15 靠近化轴;飞越大,直线越离开轴.换句话说,也就是飞 ◆186m ==========第147页========== 越大,直线向上伸展的速度也越快。这一事实就告诉我们,飞越大,函数y=kx(>0)的值增加的速度也越快. 再来看k<0的情况. 在同一直角坐标系里,作出下面这三个正比例函数 1 (4)则=-克, (5)y=-x, (6)y=-2x的图象(图3·16). 1 y=:-20 图316 从图象中可以看出: (1)这些直线都在第二、第四象限内,并且都是从左到右向下伸展的.这一事实,就告诉我们,正比例函数y=飞x(k<0)的值是随着自变量的值的增大而减小的 (2)这些直线关于x轴的倾斜程度也是随着比例系数的变化而变化着的,|k越小,直线越靠近x轴;越大,直线越离开x轴.换句话说,也就是k越大,直线向下伸展的速度也越快.这一事实,就告诉我们越大,函数y=kx(k0的时候,的数值增加的速度和飞的值的大小有什么 关系? [提示:从图象上研究,当自变量增加同一数值的树候,函 数兰:租=“所对应的值,哪一个增加得 多一些.增加得多的,增加的速度也就越快.] (G)当飞<0的时俠,函数值减小的速度和k的绝对值的大小有 }么关系? 2.求上题中这四条直线的正方向与x轴的正方向所成的角(精确到1). §3·6反比例函数 1.反比例关系 我们知道,物体作匀速运动时,距离$,速度”和时间t这三个量之间存在着以下的关系: s=ut 如果&是一个常量,那末行驶这段距离所需的时间t就与速度w有关,并可利用下面的公式计算出来: t=8 例如从A到B的距离是120米,运动的速度是v米/秒,运动的时间是秒,那末由公式 ·1890 ==========第150页========== t120 (m>0) (1①) 就可以计算出,当=3,4,6,8,10,12,…时t的对应值如下表: )(米/秒) 6 10 12 t(秒) 40 10 这就指出,公式1=120(v>0)确定了变量t和v之间的 2 一种函数关系. 仔细考察一下这个例子中自变量的各个值与对应的函数值之间的关系,可以看出一个重要的事实,这就是:当变量v的值扩大(或者缩小)若干倍的时候,变量t的值随着缩小(或者扩大)相同的倍数.例如,当心从3扩大到4即 书大青倍,t就相应地从0缩小到0,即缩小亭倍.4 我们还可以看到,两个量v和专的每一对对应值的积都是一个常数(例如3×40=4×30=…=120,即vt=120),也就是说,变量t和变量)的倒数的比是一个常数,就是 t 1=120. 少 在算术里,我们已经知道变量间的这种关系,叫做反比例关系.现在象正比例关系一样,我们需要把算术里的这种反比例关系的定义加以推广,我们说: 变量y和之间的函数关系,如果能用公式 ,(飞是不等于零的一个常数) 来表示,那末这两个变量间的关系叫做反比例关系,飞叫做 ●140● ==========第151页========== 比例系数,出等式y=点(化≠0)确定的函数叫做反比例孟数. 例 圆柱的高是h(厘米),底面的半径是?(厘米),体积是 V(立方厘米),底面的面积是S'(平方厘米),如果体积一定 时,%和S是不是成反比例关系?h和r呢?[解) 根据圆柱体积的公式 '=hS, 可以知道h和S成反比例关系,这个函数关系就是 h-名 又根据圆的面积公式S=匹2,可以推出 V=πkr2, 即 h=I 亿r含, 这里'和π都是常数。如果令”一,上式可以写成 n3· 这个公式指出h和存在反比例关系,所以b和r不成反比例关系. 2.反比例函数的图象 要作出一个由解析式给出的函数则=()的图象,一般有三个步骤: 第一步,先取心的一些值,求出它们所对应的函数值y,列成一个表. 第二步,在直角坐标系里,画出每一对实数(,y)所对应的点 第三步,用一条平滑的线,根据自变量由小到大的顺序把这些点连接起来。 •141· ==========第152页========== 下面我们以=6为例,来作出反比例函数g-8的图象 (1)列表: :C 方 …4 -3-2 2 4 -…1.2-1.52-3 -t6 3 2 1.5 1.2 (2)根据表里的每一对实数(x,y)在坐标系里画出它们所对应的点,依照自变量从小到大的顺序用平滑的线把它们连接起来,就得到下图中的两条曲线(图3·18). 2 49 图3.18 注 因为当女=0的时候,没有意义。所以圈象不能和)轴相 交.图象是由两条曲线所组成的,这个图象叫做双曲线. 上面这个例子里,画出图象的方法叫做描点法.用描点法所画出的函数的图象,一般只能是近似的,但是图象上的点作得愈多,那末画出的图象也就越精确 〔注意〕 用描点法画函数的图象的时候,要特别注意顺次连接 ·142· ==========第153页========== 图3.19 图320 各点的线应该是平滑的线,面不能是折线.例如在例1中如果画成图3·19那样就是错误的. 用同样的方法,我们可以作出反比例函数y=二当为取某一个确定的值时的图象.画出来的图象都是双曲线,只是根据的值的不同,曲线有不同的位置 例如图20中国出的是-士2士4时函数g一冬的图象. 3。反比例函数的性质 利用图3·20所画出的反比例函数的图象,我们可以发现反比例函数则=在(飞≠0)有下面的一些性质: (1)当>0时,函数图象的两个分支分别分布在第 一、第三象限内,在每一象限内,函数值y总是随着自变量的增大而减小的;当k<0时,函数的图象的两个分支分别分布在第二、第四象限,在每一象限中,函数值y总是随着自变量x的值的增大而增大的。 ●148◆ ==========第154页========== (②)函数图象的两个分支都无限接近x轴和y轴,但永远不能达到轴和y轴.这一事实也就指出,当}心充 分大时,函数)-左的值可以无限接近0但不能是0。当|无限接近0时,函数y=左的值的绝对值g就无限增大. 习题 1、下面这些关系中娜些是反比例关系?为什么? 36 (1)完成一定工作的时间t和人数2(假定每人的工作能力相 同); (2)被除数一定的时候,除数d和商q; (3)两数的和·定的时候,加数a与加数b; (4)重量一定的时候,物体的体积V和此重d; (⑤)车行骏距离一定的时候,车轮的直径风和车轮旋转的周数 2. 2.已知y和x成反比例关系,比例系数是8.写出y和x间的函数关系,指出其定义域,并用猫点法作出图象。 3.在:同一坐标系甩,作函数 =5和=一5 的图象,这两个图象有哪些相同的地方?哪些不同的地方? 4.已知y和间的关系可以用下等式来表示,求y和x间的函数关系y=f(),并作H出图象: (1)y-10=0; (2)2cy+7=0. 本章提要 1.常量和变置 始终取同一数值的量一常量. 在问题的研究过程中【可以取不同数值的量一变量. 2.函数 (1)意义一两个变量x和y,如果对于x在它可取 ·144• ==========第155页========== 值范围内的每一个确定的值,按照某种对应法则,y都有一个确定的值和它对应,这时就称心为自变量,y为因变量,通常也说是x的函数. (2)符号-y=∫(),其中R表示自变量,y表示因变量,表示某种对应法则 (3)定义域一自变量的取值范围(这个集合一般可 用字母D表示) (4)函数的值--函数f(a)当花=a时所对应的值,以(a)表示. (5)函数的值域一函数值的全体(这个集合一般可用字母E表示,E={f()x∈D}). 3.作函数y=∫(x)的图象的方法(描点法) (1)选择x的一些值求出y的对应值,列成表格; (2)以每一对实数作为点的坐标(心,),画出各点; (3)依自变量从小到大的顺序,用一条或几条平滑的线把各点连接起来 4.正比例函数 (1)一般形式:y=k(k≠0). (2)定义域:D={实数}=(-∞,+∞). (3)图象:过原点的一条直线. (4)性质:>0时,y随的增大而增大;飞越大,增大速度越快; <0时,y随的增大而减小k越大,减小速度越快 5.反比例函数 一般形式g-生 (1) (≠0). (2)定义域:D={,x÷0}=(-∞,0)U(0,+∞). (3)图象:双曲线(k>0时在一、三象限,<0时在 ·145◆ ==========第156页========== 二、四象限) (4)性质:>0时,y随x的增大而减小; 无<0时,y随心的增大而增大。 复习题三A 1、闻答下面的问题: (1)什么,叫做常量?变量?举一个例子; (2)什么叫做自变量、因变量?举一个例子; (3)记号C=F(t)表示什么意思?(这里字母C表示温度的度 数,表示时间的小时数.) (4)平面上的直角坐标系是怎样构成的? 2.(1)已知-个用解析法表示的函数y=(x),怎样求自变量 x=a时的函数值? 创妇果)号,求0,f(合》-1 (3)在上题中能不能求出(②)或者(-2)?为什么? 3.(1)如果f(x)√9-,求f(0),(1),f(2),f(3); (2)在上题中求(),这里可以取的值有什么限制?4,求下列各函数的定义域: (1)y=gV-2, (2)y=ー3x ()V (4)y=√c-1. 5.求下列函数的定义域: (1)y=V9-2, (2)y=V3-z/3+ 3-x (3)y=√-:+V3+x (4)y=V3+ 1 1 (⑤)y=79-2i (G)y=3- N3+ 6.求下列函数的定义域:()y=1 31i (2)y=V3-L; (3)9=c2-x+1氵 (4)y=Vx2-g+1, ●香4内0 ==========第157页========== 7.用描点法作下列函数的图象: y-专g=0,主,主2,8,士 ②-是(红±好,±分±1,士2,士4月 (8)4=V化(x=0,1,2,3,4,6,9) [提示:在(2)中,可以用方格纸上的2个方格的边长作为 坐标轴的长度单位,在(3)中,可以利用平方根表查出√心有一位小数的近似值,如果需要,还可以在自变量指定的这些值之间,适当插入几个值,求出它们所对应的函数值,多作出几个点再来描图.] 8.(1)对于下表里x的值,查表求y=无的对应值(精确到 0.1) 2 -6 4 -2 0 2 4 (2)利用表更x和y的各组对应值,作出函数y=√的图象; (3)根据作出的图象,求/3,/,一7(精确到0.1)并且用立方根表中的结果加以核对. 9.已知x和y有下面的关系,把它们改写成y=f(x)的形式,然后 作出它的图象: (1)y=4; (2)2c+3y-6=0; (3)2y3-8.c=0; (4)2c-√y=0. 10.作出下列函数的图象: (1)y=8/+1; (2)y=+1, [提示:设法利用第8题中已填过的表.] 11.回答下面的问题: (1)怎样判断两个量之间的关系是正比例关系?反比例关系? (列举你所知道的各种方法.) (2)什么叫做比系数?比例系数可以取的值有没有限制? 12.设从管口流出的水量(升)和时间t(秒)间的关系是=t,这 里是常数: ·147● ==========第158页========== (①)和闻间的关系是什么关系? (②)已知3秒钟内流出的水量是72升,求比例系数飞(升/秒); (3)用上面的比例系数,求4.5秒钟内管口流出的水量; (4)用上:面的比例系数,求管口流出的水流是100升所需的时间。 复习题三B 1.下面的这些等式是不是能确定y是x的函数或者x是y的函数?如果可能,把它表示成y=f(x)或者x=g(y)的形式,并写出它的定义域: (1)x-22=(0; (2)82-y3=1; (3)心2+y-1=0; (4)x2+y2+1=0. 2.求下列函数的定义域(答案用区间符号表示): (1)y=/x2-5+6+-xi V3+2x-02 (2)y=23r+・ 3.当自变数为指定的数值时,求下列函数的值: (1)f(x)=3-3/9x+8当x=1-/9时; 的 (2)f()=2aV1+2当rー(음-),a>0时 4.求下列函数的定义域和值域,并用最简便的方法画出下列函数的图象? (1)y=3x; (2)y=3到xL 5.已知y=十2,y1与c成正比例,y2与x2成反比例,并且当c=2和=3时,y的值都等于19,求y和x之间的函数关系式 [提示设h==号] 6.设有三个变量,y,: (1)知果名与y成正比例,y又与x成正比例,证明名与x也成正比例: ·18• ==========第159页========== (2)如果2与y成反比例,y又与x成反比例,那末与x之间 有什么关系? (3)如果名与y成反比例,而y与x成正比例,那末与x之间有什么关系? 第三章测验题 1.已知变量y和x间有下面的关系,试用x的代数式表示y,并指 出在这些关系式中,哪些可以把它说成y是x的函数,哪些不能 (03:4y-3 (2)2=4x; (3)y2-4a+1=0(y≤0);(4)(x+3)(y-2)=y-6 2.水下列函数的定义域(答案用区间符号表示): (1)y=W+1 a2ーx+1 (2)y=Vx+2+1 V2-z (3)y=1+V4-0; x2+1 (4)y=-3c+1· 3.已知为整数,求下列函数的定义域和值域: (1)y=V3-x; (2)y=ー y 8-2c-x2· 4.已知圆锥体的体积V和底面积S可以用下面的公式来计算: V-寻8=a 这里是圆锥体的高,?是圆锥体底面的半径: (1)如果是常数,V与h之间的关系是什么关系? (②)如果V是常数,S与之间的关系是什么关系? (3)当V=3c时,写出r与之间的关系式(以h为自变毫). (4)当V=2时,画出S=f(h)的图象, 5.已知y=1十,1与x成正比例,比例系数为;与x成反比例,比例系数为2: (1)写出y和c之间的函数关系式:y=f(); (2)如果f(1)=5,f(2)=7,求(3). ·140e ==========第160页========== 4 一次函数和二次函数 一…次函数和二次函数是两种常见的重要函数.这一章里我!将研究这两种函数的图象和性质.另外,还将把以前学过的元一次方程、方程组,一元二次方程、一元二次不等式与一次函数、二次函数的图象联系起来,研究它们之间的关系,以及一些应用. §4·1一次函数的图象和性质 1。一次函数 在第三章里我们曾见到过象下面这样的函数: :2+兰t(匀速运动中路程s与时间专之间的关系 C.2r(圆的周长C与半径r之间的关系);y=kx(k≠0)(正比例函数). 这些函数有一个共同的特征,就是:在这些表达式里,等号右边都是自变量的一次式 象这种由自变量的一次式给出的函数叫做一次函数 一次函数,一般地可表示为以下的形式: y=kx+b(k≠0). 这里k和乃都是常数.注 因为允=0时,x+b就不是次式,所以在给出这个函数的表达式时,要把这个条件写出 因为不论x取什么数值,x的…次式+b都有意义,所以一次函数y=c+b(飞≠O)的定义域是实数集R. ●150· ==========第161页========== 在一次函数y=kx+b(k÷0).中,如果b=0,这个函数就是正比例函数y一x(化≠0),所以正比例函数是一次函数的一个特例. 2。一次函数的图象 我们知道:正比例函数y=kx,(k≠0)的图象是经过原点斜浓为的一条直线.现在我们就利用这个特殊的一次函数的图象来研究一般的一次函数 y=x+b(k≠0) 的图象 例如,我们在同一直角坐标系里,作下面这三个一次函数 (1)g=2.x, (2)y=2x+3, (3)y2.x-3的图象,并进行比较. 首先,我们作出如下的表: 2 0 (1)y=2x 0 2 4 6 (2)y=2+3 -3 3 5 9 (3) 1=2x一3 -9 -5 ー3 3 从表里可以看到,对于自变量x的同一个值,函数y=2x+3所对应的值比函数y=2所对应的值都大3个单位.所以在函数y=2x+3和函数y=2x的图象上如果各有-一点,它们的横坐标相同,那末第一个点的纵坐标应该比第二个点的纵坐标多3个单位.这样,就容易看到要作出函数y=2+3的图象,只要把函数y=2x的图象向上方平行移动3个单位,就可以了(图4·1). 这里我们看到,函数y=2心+3的图象是和直线y=2x平行的一条直线,它和y轴的交点是点(0,3). ●1510 ==========第162页========== 同样,要作出函数y=2一3的图象,只要把函数y=2的图象向下方平移3个单位就可得到(图4·1).这里我们看到,函数则=2c一3的图象也是和直线y=2x平行的一条直线,它和y轴的交点是点(0,一3). 之y=2.+3 y=2x-8 图41 一般地,我们可以知道: 一次函数y=心十b的图象是一条直线,这条直线平行于直线=,它和y轴的交点是(0,). 一次函数y=飞x十飞的图象,以后简称直线 9=k+8 要作出一次函数y=:kx十b的图象,只要先找出图象上的任意两个点,再过这两点作出一条直线就可以了。例1 作西数9子4叶是的图象 ·152· ==========第163页========== [解] 列表: 作图: 1 图4.2 3.直线=k十b的斜率和截距 因为直线y=形心十b平行于直线y=k心,所以直线y:k.x十乃的正方向与c轴的正方向所成的角和直线y=k心的正方向与心轴的正方向所成的角是相等的.这个角a的大小与:的值有关,就是 tg a=k, 我们把飞叫做直线y=kx十b的斜率,s叫做该直线的倾角. 因为b可看作直线y=kx+b和y轴的交点离开原点的距离再附以适当的符号,我们把它叫做直线y=x+b在y轴上的截距. 例2 求直线y=√3一1的斜率,倾角和它在y轴上的截距, [解] 这里k=√3,b=-1,所以直线y=√3x一1的斜率是√3,它在y轴上的截距是一1. 设心是它的倾角,那末 tg=√3,&=60° 所以直线y=√3一1的倾角是60°。 ◆153◆ ==========第164页========== 4。一次函数的性质 因为直线y=c+b是和直线y=飞x平行的,所以从函数y=kx的性质,可以推得函数y=kx十b也具有同样的性质,就是 (1)如果k>0,郑末当自变量逐浙增大的时侯,函数的值也逐渐增大; (2)如果k<0,那末当自变量逐浙增大的时侯,函数的值则逐渐减小. 例如,图43是函数 (1)y=3x+2和(2)=-3x+2 的图象,从图中可以看出,函数W=3十2的值是随心的增 加而增加的;而函数y=一3十2的值是随心的增加而减少的 y-3x+2 y=-3x+2 图43 从上面的图中,我们还可以看出一次函数的一个重要性质 ●1540 ==========第165页========== 在函数y=3+2中,如果自变量心从一2开始每次增加同一个值1,那末对应的y的值也就从-4开始,每次增加同一值3 2 2 2 5 8 对这种现象,通常我说函数的值是均匀增加的.可以验证函数y=3x十2增加的值和自变量x增加的值的比是个常数,这个常数也就是直线y=3x+2的斜率3.例如 (-1)-(-4)3 (-1)-(-2) 2-(-1)= =3, 3 0-(-1) 5-23 1-0 =3, 等等. 同样可以看出函数y=一3x十2,当自变量每次增加同 一个值的时候,对应的函数值就每次减少同一个值 -2 5 -1 ー4 对这种现象,通常我们说函数的值是均匀减少的.可以验证,函数y=一3x+2减少的值和自变量增加的值的比是一个常数,这个常数也就是直线则=一3心十2的斜率一3的绝对值. 一般地,我们可以证明: 函数y=ka心十b当>0的时候函数值是随自变量的增 ·155● ==========第166页========== 加而均匀增加的,函数所增加的值与自变量增加的值的比等于;当<0的时候函数值是随自变量的值的增加而均匀减少的,函数所减少的值与自变量增加的值的比等于1%. [证明] 设心=红,2=2(1<2)是自变量的任意两个值,利它们对应的两个值分别记做y1和y2,那未 y1=kx1十b, y=k.心gb. 两式相减,得 2-1k(c2-), 2一1=k, dーC! ..当k>0时,-y1>0,则2>y1,函数值是均匀增加的;增加的比等于. 当飞<0时,一y1<0,yg<1,函数值是均匀减少的,减少的比等丁k: 习题 1.火车出A站以,以每小时40公里的速度匀速前进: 41 (1)求t小时以后,火车离开A站的距离s公里(②)由下表里(的值,求s的值; 0.10.2 0.) 0.4 0.5 0.6 0.7 0.8 0.9 .0 (小时) (公里) (3)画出s和t间函数关系的图象; [提示:为了画图的方便,表示t的横轴和表示§的纵轴, 要取不同长度的线段做长度单位.] (4)这个图象种函数s=40t的图象有哪些相同的地方?哪些 不同的地方? 2、:一坐标系里作下列这些函数的图象: )y号出 (2)y=3如-出 ·156· ==========第167页========== (8)y=3x+2 比较这些图象有哪些相同的地方?哪些不同的地方? 3.在上题中,求 (1)这些直线的率,和它们在y轴上的截距;(2,这些直线的颅角(情确到1°)「是:利用三角函数表.] 4.(1)在同-坐标系里作承数 3y=2x-3和y=3a-2 的图象; (2:从象上找出当x取什么值的时候,这两个函数有相同的 值.这个值是什么? y=2x-3 (3)用鲜方程组1y-3-2的方法来验证(2)里得出的结果 5.已知变x和y间有下的关系: (1)2x+3y-4=0; (2)3x-2y-5=0 写:y和x间的数关系y=f(x),然后作出它们的图象. §4·2根据已知条件确定一次函数 在:一次函数y=kx+b里,如果我们知道了k和b的值,那末这个函数就完全确定了.要确定和的值就需要有两个条件.下面我们举例说明怎样根据已知条件来确定一个一次函数.例1 己知函数y=x十b的图象通过两点 A(1,2)和B(一1,1). 求这个函数,并画出它的图象 [审题] 这里就是要求出和b的值.因为A点在直线上,所以它的坐标=1,y=2应当适合y=k+b.这样,把这个值代入等式可以得到关于和b的一个方程.同样把花=一1,则=1代入等式可以得出另一个关于和b的方 ·1574 ==========第168页========== 程,解这两个方程所组成的方程组就可以了,[解] 因为A(1,2)在直线上,所以 2=k.1+b, (1) 因为B(-1,1)在直线上,所以 1=k(-1)+b. (2) 解这两个方程所组成的方程组,得 所以所求的直线是 1 则=2t大9 . 图象如图44。 ≈1メ3 图44 例2 按照下面的条件确定一个一次函数y=(x): (1)f(0)=3,f(1)=-1; (2)函数的图象是过点(一1,1)并且倾角是135°的 一条直线 [解] 设f(c)=kc十b. (1)根据已知条件,可得方程组 3=花.0+b, 1-1=k1+b. ·158· ==========第169页========== 解这个方程组得b=3,k=一4. 所以所求的函数是则=一4心+8。 (2)根据已知条件,有 k=tg135°=-1. 所以所求的函数有 则一亿十b 的形式.因为点(-1,1)在这条直线上,所以 1=-(-1)+b。 解这个方程,得b=0 所以所求的函数是y=一x. 习题 1.已知函数y=x+b的图象通过A,B两点.确定这个函数,并 42 且画出它的图象: 四4(层》(1,2)4(1,-1),B(3,2月 (3)4(0,),():(4)a(1,)(ラ,) 2.按照下面的条件确定直线y=x十b: (①)斜率是-3,y轴上的截距是: (2)倾角是45°,y轴上的截距是一1; (3)斜率是受并且过点(3,0):(④斜率是子并且过点(0,3)9 (5)倾角是60°,并且过点(0,2); (6)顷角是60°,并且过点(2,0). §4·3二元一次方程的图象 1.方程ac+by+c=0的图象 在§4·1里,我们己经知道函数y=k心+b(飞≠0)的图象是-条直线。事实上,函数y=kx十b也可以看做是关于 ·159· ==========第170页========== 心和y的一次方程 k-y十b=0(飞≠0). 这样,我们也就可以知道二元一次方程 kx-y+b=0(k≠0) 的图象是一条直线 现在,我们来研究一般的情况,证明任何一个二元一次方程① ax+byc=0(a,b不同时为零) 的图象都是直线 根据,b可取值的条件,可以看出要证明这个结论应该分成三种情况,就是: (1)≠0,b≠0; (2)a=0,b≠0; (3)a≠0,b=0, (1)《≠0,b≠0.这时方程可以化为 (1) 这是x的一次函数,我们已经知道它的图象是一条直线.这条直线的斜率是一名,在y朝上的截距是一云如果c=0,那末这条直线就经过原点. (2)u=0,b≠0.这时方程可以化为 y=0x-云 (2) 这里可以看到,不论变量心取什么实数值,和它对应的y的值总等于一云,所以它的图象是平行于轴,并且和它朝的距离等于一合的直线。当-号>0时,宜线在如 ①当a,b中有一为零时,例如0:x十飞y+c:=0(≠0)实际上是y的一次 方程,但是在研究二元一次方程组时,可把它看成是关于x,y的二元次方程。 0180· ==========第171页========== 轴上方,当-分<0时,直线在心轴下方;特别,当0=0时, 一云0,所以它的图象就是在轴即y=0(图4. (3)《≠0,b=0.这时方程可以化为 (3) 可以看到,不论变量则取什么实数值,和它对应的的值总等于-合所以它的图象是平行于y轴,并且和y轴的距离是一的一条直线.当一&>0时,直线在轴的右边:当一8<0时,直线在y轴的左边:特别,当c=0时,一日0,所以它的图象就是y轴即如=0(图4⑥). =-分0 <0 x=0 .0 0 0 y=ー 0 图4.5 图46 总结上:面这三种情况,我们得到 方程a+by十c=0(a,b不同时等于零)的图象是一条直线. 以后我有」把这个图象,筒称为直线a十y十c=0. 。161. ==========第172页========== 注 由于二元一次方程的图象是直线,所以常把一次方程叫做线性方程. 我们已经道,画出一条直线,只要找出直线上的两个点.为了计算上的方便,如果方程ax+by十c=0中,c≠0,我们可以分别使x=0和y=0,找出和它们对应的y和x,从而来得到这两个点.但是有时为了使所求出的点尽可能用整数来表示,以便于作图,也可以通过观察,适当地选择x的两个整数值,求出和它们对应的y的整数值,从而来得到这两个点 例1 画出方程2x+3y=:17的图象,并利用图象求出它的正整数解. [解] 列表: 作图: U 5 2 1234667s -1 -21 图47 从图中可以看出直线在第一象限内经过的横坐标和纵坐标都是整数的点有3个,即(1,5),(4,3),(7,1). 所以本题的正整数解是 x=1, ∫龙=4,∫=7, 0y=5;【y=3;【y=1. ·182· ==========第173页========== 〔注意) 1.本题也可采用下面的解法:由原方程得 0= 17-22=6-1+2℃ 3 3 因为,9都是正整数,所以1吉2必领是小于6的正整 数.由此可知x只可能是1,4,7.从而可求得本题的正 2.二元一次方程ax+y=c(u≠0,b≠0).一般都有无穷多个解.这种方程叫做二元一次不定方程.上面这个问题,通常也把它叫做求二元一次不定方程的正整数解。 2。二元一次方程组的图象解法 如果在同一直角坐标系里画出二元一次方程组里两个方程的图象,就可以根据图象求出方程组的解。这种解法叫做二元一次方程组的图象解法.举例说明如下:例2 用图象解法解方程组 ∫化-y+2=0, (1) 17+2则+2=0. (2) [解] 列表: 作图: (1) 2-+2=0 2 0 -2 2 0 (2) 23 -1 1 图48 ·163◆ ==========第174页========== 直线和(2的交点是(-,1号),所以方程组的 解是 2 3 1 3。二元一次方程组的解的讨论 现在我们利用二元一次方程的图象,来讨论二元一次方程组 (I)f (1) (a1,a2,b1,bg都不等于0) luex十b2y=('g (2) 的獬的各种情况 设方程(1)和(2)所表示的直线,分别是L1和L,那术 直线4的斜率是-一云,在9轴上的截距是直线1的斜率是-器,在y轴上的裁距是号,平面上两条直线的位置关系有而且只有相交、平行和重合这三种可能情况 (1)如果L1和L相交,那末它们只能有个公共点, 相应地方程组(I)有唯一的解、这时两条直线的斜率应该 不同,从而可以推出: *않>+a2 6a (2)如果L1和L?平行,那末它们就没有公共点,相应 地方程组(I)也就没有解.这时两条直线的斜率应该相同. 但是在y轴上的截距不同.从而可以推出 -1=-9→41=01 02b2 C.+은a → aa 9 ◆1640 ==========第175页========== (3)如果工1和L2重合,它们就是同一条直线,因此也 就有无数多个公共点,相应地方程组(I)就有无穷多个解. 这时两条直线的斜率、在y轴上的截距都应该相同.从而可以推出 b 61 2 b= 品→1 Ca 上面的讨论,反过来讲也对.这样我们就可得到以下的结论 (1)如果卡6,那未方程组①有唯一的解 (2) 如果6≠生,那末方程组(①)没有解; (3)如果-。-生,那未方程组(④有无穷多个 襄3 62 C2 解. 应用二元一次方程组的解的公式,上面这个结论就是:设方程组 1b=c1,(a,e,b,b都不等于0) L asa-1-bay=ca中, D= aib3-4:ba:D:co o=C10-Cab Ci D =1Cg-201, (1)如果D÷0,那末方程组有唯一的解 D D (2)如果D=0,D≠0(这时Dy≠0),那末方程组没 ●165· ==========第176页========== 有解。 (3)如果D=Dx=D=0.那末方程组有无穷多个解. 例3 解下列方程组,并进行讨论: 2-3y=k十1,(k≠0).しkx+6y=3 D/2 -3 [解] =12+3k=3(4+k), 6 k+1-3 D =6(k十1)+9=6k+15=3(2k+5), 3 6 2k+11 3=6-k(张十1)=6-龙-k2 =(3+)(2-). (1)如果D≠0,即12+3k≠0,k≠一4.方程组有唯 一的解: 2k+5 4十k’ 6-龙-k2 y=12+3k· (2)如果D=0,即k=一4,这时D:=-9≠0,方程组无解. 1. 习题 作下列各方程的图象,并戈前面三条直线的斜率和在y轴上的 43 截距 ()풀+号=1 (2)4x-5y-20=0; (3)y-4=0; (4)g+4=0. 2.(1)作出方程4c一3y=8的图象; (2)求这条直线的斜率,y轴上的截距; (③)这条直线是不是和x轴相交?如果相交,交点离开原点的 距离是什么? (④从图象上找出当女=宁,一是的时民,适合于方程的y的 值; ·166● ==========第177页========== (5)从图象上找出当y=4,一3的时候,适合于方程的的值. 3.用图象法解下列各方程组,并且用代数解法(加减法或者代入 法)来检验 xy17, 〔x+y一6=0, (1) (2) (:+y=1; 2x-4y+9=0; f龙头=1, 23 (3) 1劣-3y十4=0, (4)(2x+y+1=0. 4.已知方程组: fy=kx十m1, y=kax-+m2 试讨论在什么情况下,这个方程组有唯一的解,有无穷多个解,或者没有解。 5.解下列关于x,y的方程组,并进行讨论: (1)「nx+y=3, c+my=州+3(m+0);(2②){y=x十, y=2r-1 6.求下列各二元一次方程的正整数解的集合: (1)3x+5y=33; (2)10x+13y=184 §44二次函数的图象和性质 我们来看变量之间下面这些函数关系: 设正方形的一边是c厘米,它的面积是y平方厘米,那末变量y和x间有下面的函数关系 y=x2. (1) 设圆的半径是厘米,又设它 的面积是A平方厘米,那末变量A 和r间有下面的函数关系: A=元ra (2) 图49 设在半径是20厘米的圆面上,从中心挖去一个半径为 ◆187● ==========第178页========== x厘米的圆面(图4·⑨),剩下的圆环面积是y平方厘米,那末变量y和心间有下面的函数关系: y=400x-x.2 (3) 从上面这些例子可以看出,它们行一个共同的特点,那就是每一个函数关系中,等号右边都是自变量的二次式 象这种由自变量的二次式给出的函数叫做二次函数 二次函数,一般地可表示为以下的形式 y=am2+bx+c(a≠0), 这里a,b,c都是常数 因为不论e是什么实数,x的二次式a2+bx十c都有意义,所以二次函数y=a22+bx+c(a≠0)的定义域是实数 集R. 下面我们来研究最简单的二次函数y=2的图象和性质,然后以它为基础来研究一般的二次函数y=a2+bx+G(a≠0)的图象和性质 1.函数)=2的图象和性质 我们先用描点法,画出二次函数y=22的图象(图410). 列表: 作图: D -3 -1 增 0 加 1 2 3 增训 4 16 图410 c16e· ==========第179页========== 观察这个图象的特征,就可以发现二次函数y=x的 一些重要性质,对照如下: 图象的特 征 函数y=f()=2的性质 1.过甄点,目在轴的上 1.函数的值y>0. 分问无限按. 当=0时,y=0.函数的值域是{yy≥0}. 2.!对所:于y轴① 2.取互为相反数的两个值时,对应的函数值相等, f(-x)=f(e). 3.在铂的左侧,图象自上而 3.当x<0时,函数值y随自交 下逐渐下降、 量x的增大逐渐减小. 在y轴:的右侧,图象自下而上 10时,函数值y随自变量的增大逐渐增大 0于(01). 4。象有最低的一点,即原点, 4.函数当x=0有最小值y=0, 但无最高点, 但无最大值. 上面画出的二次函数y=x的图象叫做抛物线;y轴叫做抛物线的对称轴(简称轴);对称轴与抛物线的交点叫做抛物线的顶点,这个抛物线的顶点是原点 二次函数y=的图象,以后简称抛物线y=2 2.函数y=(a≠0)的图象 这里a可以是正数或负数,我们分两种情况来研究(①)设《是正数.例如我们在同一坐标系中,来画下面这三个函数的图象: y=22,y=,则=2. ①果线段PP'被一条直线垂直平分,那就说P点和P点是关于 这条直线为对称的, ●169● ==========第180页========== 我们先作出下面的表: 3 化 -2 -1 0 2 2 2 y=2:x2 9 8 2 2 0 2 8 2 型=3 9 0 4 4 4 、1 1 9 0 2 8 2 8 从这个表可以看到,对于同一个心的值,函数y=22所对应的值是函数则=心所对应的值的2倍(例如,当¢=1的时候,函数y=2所对应的值是2,函数y=2所对应的值是1,2是1的2倍,等等).所以要画出函数y=2x的图象,可以用函数y=2的图象为基础,把图象上每一点的纵坐标扩大到原来的2倍,得到一些新的点,然后把这些点用平滑的线连接起来. 1 同理,要作出函数y=克的图象,也可以用函数y=2的图象为基础,把图象上每一个点的纵坐标缩小到 y= 原来的子,有到一些新的 y=2c2 点,然后用平滑的线把它们连接起来 这样,就得到这三个函数的图象如图411, 这里,我们看到函数 2和话数ー~受的 图411 图象仍旧是抛物线,它们仍都在心轴的上方,并且仍以则轴 01700 ==========第181页========== 和原点为对称轴和顶点。只是抛物线y=2x的开口比抛 物线y=的开口小一些,而g-受的开口,比=心的开口大一些 (2)设%是负数.例如,我们要画函数 y=ーx3 的图象,也可以在函数y=2的图象的基础上来研究 3 3 0 2 y=2x2 9 9 4 1 0 4 9 9 y=-22 0 4 4 从这个表可以看到,对于同一个x的值,函数y=一公所对应的值,恰巧是函数y=x所对应的值的相反的数.所 y=22 一2x2 三-2 图4.12 图4.13 。1710 ==========第182页========== 以只要把函数y=心的图象上的每一个点的纵坐标改为它的相反的数(就是取这个点关于心轴的对称点,或把图象沿心轴折转过来),把新得到的这些点用平滑的线连接起来,就得到函数y=一x2的图象(图4·12). 这个图象仍旧是抛物线,它的对称轴还是y轴,顶点还是原点,不过图象是在轴的下方. 同样,从函数)=2和g名的图象可得出困数g=-2和y=一是的图象,它们也是以y轴为对称轴,·原点为顶点的抛物线,但是图象在轴的下方(图413). 总结上面这两种情况,我们知道: 函数y=Q22的图象是一条抛物线,它的对称轴是y轴,顶点是原点.当a>0的时候,抛物线y=2在轴的上方,在y轴的左右两侧同时向上无限伸展;当a<0的时候,抛物线y=ax在心轴的下方,在y轴的左右两侧同时向下无限伸展 习题 1.石块从40米的高处自由落下,已知计算高度的公式是 4•4 7=40-4.92, 这里是所求高度的米数,t是所经过的时间的秒数,求:(①)经过2.0秒钟,石块的高度是多少? (2)经过多少时间石块落到地面(精确到0.1秒)? 2.在坐标纸上画出函数-x2的图象 (1)根据图象,求当化=1.5,x=2.3,心=-1.4时,y的值(精确 到0.1): (②)根据图象,求当y=2,=8,y=4.5时,对应的心的值(精确到0.1); (3)利用图象求√万,√7的值(精确到0.1). 3.在同一坐标系里作函数 y=3心,y=3.c2 •1720 ==========第183页========== y=-3c2, =- 的图象,这些图象有哪些相同的地方?哪些不同的地方? 3、函数y=ac3÷bc+c(a≠0)的图象和性质 现在我们来研究怎样利用函数y=2的图象来作出 一般的二次函数y=ax+bx+c的图象,并由此来看出它所具有的一些重要性质. 在研究一次函数的图象时,我们曾经利用直线y=就上的点的平行移动来作出一次函数y=c十b(飞≠0,b≠0)的图象;并且知道只要把直线y=飞x上的各点沿y轴的方向向上(或向下)平行移动{个单位,画出的那条直线也就是一次函数y=+b的图象.现在我们也用平移的方法,把二次函数 y=ax (1) 的图象作平行移动,看会发生什么结果 设抛物线(1)上的任意一点是P(,1).点P沿x触方向问右移动p个单位, y=a(化-p)2+q 沿y轴的方向向上移动g个单位所得到的点是(,y). 那末从图中可以看到点Q的 坐标(a,y)与点P的坐标 Q(x,9) (1,y1)之间有如下的关系: {=+2, P(x1,y1) 0 y=254+9: 也就是 图414 U1=优一p, (2) 01=y-9. 因为P(1,y)是图象上的点,它的坐标应该满足(1),所以 •173● ==========第184页========== 1=a.c. (3) 把(2)代入(3)得 y-9=a(x-p)2, 即 y=a(-p)2+q. (4) 这就是点Q(x,)的坐标应该满足的方程.这样也就说明了点Q在二次函数y=a(心一p)”+q的图象上移动. 由此我们也就可以知道: 二次函数y=a(x-p)+q的图象是函数y=aa2的图象沿c轴方向移动p个单位(p>0时向右,p<0时向左),沿y轴方向移动q个单位(q>0时向上,9<0时向下)所得到的抛物线,它的顶点是(p,q),对称轴的方程是化=p. 例如,因为 -受+x-营-日+2+1)-3 -是e+12-3, 由此就可知道,二次函数 5 5 的图象是和二次函数 3 1 1 形状、大小开口方向都是 .3 2·3 一样的一条抛物线,只是 y= 它们的位置不同. 3 1 抛物线=立心2的 图4.15 点是(0,0),对称轴是y轴即直线=0. 抛物线是”+一名的顶点是(-1,一),对称 ●174● ==========第185页========== 轴是直线c=一1(图4.15). 一般地,因为 +ar-e-이+3会+()+(-2) Aa 所以我们得到: 函数g=a心2-+bx十c(a≠0)的图象是一条抛物线,它的对称轴是直线=一 顶点的 坐标是 64ac-62 2a' Aa (1)当>0时,抛物线从顶点起在直线4ac-b° 的上方,在其对称轴的两侧同时 la 句上无限伸展; (2)当a<0时,抛物线从顶点起在直线 4ae-62的下方,在其对称轴的两侧同时 4a 向下无限伸展 由此我们也就可以从二次函数y=ax2+bc+c(a≠0)的图象中看出它所具有的一些重要性质: 函数y=ac2+飞ax十c(a>0)的性质 函数y=ax2+bc+c(a<0)的性质 1.当<名叶,函数值随 ,当<-云时,函数道脑” 的值!增式逐渐减小 值的增大,逐渐增大。 当e>ー时,函数值随 当公品防,函数植随值2a 的值的增大,逐渐增大 的增大,逐新减小 2.当双=时, 2a 函数有最 2.当4=-8时,函数有最大 小值=4一2, 值y=4ac-b2 4a 4a ·175· ==========第186页========== 题 1.以函数y=x2的图象为基础,你怎样可以不通过计算就画出下 4·4 面一些函数的图象(不作出图象)? (2) (1)y=x2上2; (2)y=2”-2; (3)y=(c41)2; (4)y=(x一1); (5)y=(+1)2+2; (6)y=(x-1)°-2; (7)y=(g1)-2; (8)y=(8-1)2+2. 2.把下面这些函数,改写成y=a(父十)+k的形式,然后指出把函数y=x2作怎祥的移动,就可得到这个函数的图象: (1)yニ2-x-1; (2)y=x2+2x; (3)y=2+1; (4)y=(+2)(x+3). 3.六下面这些抛物线的对称轴、顶,点,和开口的方向(不作出出图象): (1)2y=2x2-4x+2; (2)y=2x2+4x-6; (3)y=3-5x; (国gw-号-2-。 §4·5二次函数图象的作法 在§4·4里,为了要说明二次图数y=a:x2+b十c(a≠0)的图象是一条抛物线,所以先作出二次函数y=的图象,在这基础上一步…步地推出一般的二次函数 =a2+bx千c 的图象.但是,在实际作图的时候,却可以不必套用这种方法.我们只需找出抛物线的对称轴和顶点,然后再找出抛物线上的几个点,就可以相当精确地把抛物线画出. 例如,作函数y=2一心十2的图象,我们可以采取以下的步骤: 1°首先把等号右边的二次三项式配方得 y-ー2(-)+1 由此根据§4·4里的结论,我们知道这条抛物线的对称轴 是直线号,顶点是4(侵,1星) ·176· ==========第187页========== 2列表: 1. 1 出 1 3 2 21 0 1 2 ●● 2 12 2、 0 9 25 4 1 4 4 13 2 3 63 8 3把表里的每一对实数(花,)作为点的坐标,作出这些点,并且用平滑的线连接起来,就得到抛物线y22-2在对称轴右边的“个部分. 4°因为抛物线 22-2是对称门对称轴的,利用 一y= 这个性质就可以画出抛物线在对称轴:边的部分(图4·16). 注 作海时也可以不先把二 图416 ーー3用方法改写威ー(一)+的形式, 而直接应用第175页里的结论.这里 《=1,b=一1,c=2 4a0-62=13 4a 因此抛物线的对称轴是直线=是,顶点是4(合,1星) 从=号起,取心的-些适当值,算出函数的对应值,列成下 表: ●177• ==========第188页========== 1 1 3 2 52 3 2 23 6 4 8 4 以下就按照上面解法中的步骤来做。 例 作函数y=3一40一22的图象. [解] (1)这里a=一2,b=-4,c=3. 2a2(-2)=-1, 48-39=4(-2)3-(-4)2=5. Aa 4(-2) 所以抛物线的对称轴是直线x=一1,顶点是(一1,5). (2)列表: (3)作图: 2 0 1 2 1 图4.17 习题 1.画出下面这些函数的图象,并求出它的顶点的坐标和对称轴: 45 (1)y=x2-6:c+4; (2)y=-2x2-4x+6; (3)y=x(x-2); (0y=1+2-司x. 2.按照自变量x可取值的范围,画出下面这些函数的图象: 。178。 ==========第189页========== (1)y=22+3(0≤x<+o)②1-号0c4: (8)=ーaー():(4)yニ4ー(0ε16) §46根据已知条件确定二次函数 上一节里我们学习了已知一个二次函数画出它的图象的方法.但有时我们也会遇到另一类问题:已经知道一个函数是二次函数,并且还知道它具有某些特点,要求找出这个二次函数.下面我们来看几个例子:例1 已经知道函数y=∫()是一个二次函数,并且知道它 的图象通过A(0,1),B(1,3),C(-1,1)三点,写出这个 二次函数. [审题] 二次函数的一般形式是 y=ax2+b0+c. 要确定这个函数,必须知道二次三项式里三个系数a,b,0 的值.现在已知A,B,C三点在图象上,它们的坐标适合 上面的方程,因此可以列出关于a,b,c的三元一次方程组: f1=a02+b0+c (因为A在图象上), 3=a12+b1+G (因为B在图象上), 1=(一1)+b(-1)+c(因为C在图象上).解这个三元一次方程组就可以确定a,b,c.[解] 设所求的二次函数是 y=:ax2+6x+c. (1) 因为A(0,1)在函数的图象上,所以把c=0,y=1代入(1),这个等式应当成立,因此有 1=a08+b.0+c, 就是 ·179● ==========第190页========== 1=C, (2) 同理,由B,C的坐标适合方程(1)得到 3=a+b十C, (3) 1=a-b十0. (4) 解由方程(2),(3),(4)所组成的关于a,b,c的方程组得 a=1,b=1,c=1. 所以所求的二次函数是 y=22+G+1. 例2 己知函数y=ax2+b:x十c的图象是以点(2,3)为顶点的抛物线,并且这图象通过点(3,1).写出这个函数,并且画出它的图象 [审题] 抛物线y=u+b.x十c的顶点的坐标是 (-a, 根据已知条件,可以得到两个关于a,b,c的方程 =之,100- b =3, 2a da 另外,根据抛物线通过点(3,),可以得到另一个方程 1.=9a+3b+C, 解这三个方程所组成的方程组就可以确定%,b,C. [解] 根据已知条件,有 、6=2, 2a (1) 4a0-b3Aa =8, (2) 9a+3b+c=1. (3) 从().得 b=-4a, (4) 代入(2)和(3)得 ●180· ==========第191页========== 4ae-16a2 4a =8, 就是厂c0=3,(5) 9a-12a-+c=1, lc-3=1. (6) 解(6)和(6)所组成的方程组得 a-2,心-5 把2代入(4)得 b=:8. 所以所求的二次函数是 y=-2x2+8.-5利用下面的数值表: 图4.18 0 2 3 5 3 1 可以画出函数的图象如图4·18所示.注 事实上,(化,)做顶点的抛物线的方程,可写成 y=(c-)2+飞 这样根据已知条件,就可以写出所求的二次函数是 y=1c-2)2+3 (7) 因为点(3,1)图象上,所以x=3,y一1代入(7),等式成立,就是 1a(3-2)2+3, 由此得一2,把它代入(7),就得 y=-2x-2)243 =-2x28x-5. 所以所求的次函数是 y=-2c2+8x-5 这个解法要比前面那个獬法更方便一些。 0181● ==========第192页========== 习题 1.设有二次函数y=x2+x+9,按照下列条件,求出p和q的值, 46 然后写出这个二次函数: (1)在=2的时筷,y=12;左x=一3的时侯,=2; (2)函数的图象和x轴的交点的坐标是(一4,0)和(一1,0); (3)面数的图象是以点(5,-2)为顶点的抛物线 2.设有二泸函数y-x2+b心÷(,按照下列条件,求出a,b,c的 值.然后写这个二次函数: (1) 2 3 0 0 (2)函数的图象是以,点A(一1,一8)为顶点的抛物线,并且和 y轴交于点B(0,-6). §4.7二次函数的极值 1。二次函数y=ac3+bc+C的极大值和极小值 我们来观察下面这两个二次函数的图象: (1)y=2-x十2; (2)y=3-4x-22, =5 y=x2-g十2 7 =3-4x-22 图4.19 图4.20 ●182· ==========第193页========== 从图4·19中可以看出,二次函数划=心2一心十2的图象 都在过地物线的顶点(经,?)所作平行于轴的直线y一子的上方,并且在它的对称轴云-是的左边是从左到右逐渐下降的,在对称轴”=立的右边是从左到右逐渐上 升的.这也就告诉我们: 当。<号的时候,函数的值随着:的增大而逐有减小心>的时候,函数的值随着的增大而逐渐增 y 当知=的时候,函数的对应值y一在7 比:略小于 和略大于是时,函数的对应值都要小,通常我们说:数y=ー+2当ー时有小子记作=7 很明显的,二次函数y=x2-心十2的极小值也就是它的最小值. 同样的,通过对图4.20的观察,我们可以知道:当<一1的时候,函数的值随着心的增大而逐渐#大: 当心>一1的时候,函数的值随着心的增大而逐渐诚小 当:=1的时侯,函数的值y=5,比化略小于一1和略大于一1时函数的对应值都大。通常我们说:函数y=3一4一在xx一1时有极大值y=5。记作y欧=5(或ymax=5). 很明显的,二次函数y=2+4x-的极大值也就是它的最大值。 01880 ==========第194页========== 在研究一般的函数y=f()的极小值和极大值的时候,我们是就函数在心=a附近的情况来考察的,就是如果在心=a的时候,函数f(心)的值,比略小于和略大于a的时候函数f()的值都小,我们说函数f(心)在化=a时有极小值f(a). 如果在心=a的时候,函数f(心)的值,比心略小于和略大于a的时候函数∫()的值都大,我们说函数f(心)在=a的时候有极大值f(a). 函数f(c)的极小值和极大值,统称函数f()的极值. [注意) 函数的极值是在一点近旁来考察的,所以是局部性的,它与从某一范围来考察的函数最大值和最小值是有区别的、但对于二次函数来说,它的极大(小)值就是它的最大(小)值,所以我们可不加区分了. 从上面的例子可以看出,求二次函数的极值,只需找出它的图象的顶点的坐标( b4ac-b2),根据a是正数2a 或负数,就能知道: 对于二次函数y=ax2+bx十c, ()如果>0,那未,当x=-2名的时 侯,函数有极小值 3袋小4a0-63 4a (2)如果a<0,那末,当x=一 的时 侯,函数有极大值 大4ac-b3 Aa 注 在实际解题时,可不必记忆这个结论,只需把函数 y=x2+br十c ·184◆ ==========第195页========== 应用配方法写成 4a— 这样就可以直接判断x在取什么值的时候,函数有极小值或者极大值。 例1 求函数 (1)y=x2+4x+7, (2)y=6+4-x2 的极大值或者极小值,[解] (1)'y=x2+4w+7=(x+2)2+3. 函数在x=一2时有极小值3. (2).·y=6+4c-c2=-(2-4c-6) ー(c-2)3+10 .。函数在x=2时有极大值10. 习题 1.画出二次函数y=x2-6元+4的图象,并且从图象上找出: 47 (1)x取什么值的时侯,函数值随x值的增大而增大?随:值 的增大而减小? (②)函数在x取什么值的时候有极大值或极小值? 2.仿照上题那样,画出函数y=一x2+6x一12的图象,并且从图象上研究这个函数的性质 3.下列函数在x取什么值的时候有极大值或者极小值?函数的极大值或者极小值是什么? (1彩=2-+是: (2)y=2x2-4x+3; (3)y=2-5c-3x2; (4)y=5+50x-5c2。 2.二次函数极值的应用 求二次函数的极值有着许多实际的应用,下面我们苹几个例子 例2 在墙旁的一块空地上,推备靠墙用36米长的篱笆围一块矩形的空地,种植蔬菜.问怎样围法,才能使所围成的园地的面积最大?这时面积是多少平方米? .1得· ==========第196页========== [审题] 如图(图4·21),折线ABCD表示篱笆,如果AB- CD=x(米),那末BC=AD=36-2c(米).矩形的面积是 y=(36-2x)(平方米) 所以本题只要求函数y三c(36一2c)的极大值.[解] 设篱笆的宽是x米,那么它的长就是36-2米.又设矩形的面积是y平方米,那么有 y=c(36-2x)=36x-2x2 =ー2(a3-18x)=-2(c-9)2+162 当c=9的时候,二次函数y=一2(x-9)2+162有极大值162. B 36-2x 2 B 图4.21 图422 这就是说,当篱笆的宽是9米时,可以围成最大面积的园地,这块园地的面积是162平方米例8 窗的形状是矩形上面加一个半圆(图4·22),它的周长等于6米.要使窗能够透过最多的光线,它的尺寸应该怎样设计? [审题] 这个问题就是要求出半圆的半径是多少米的时候,图形有最大的面积.因此要先求出图形的面积y(平方米)和半圆的半径心(米)间的函数关系 ◆.1860 ==========第197页========== [解] 设半圆的半径是心米,那末半圆的长就是π米,矩形 的底B0就是2米,而矩形的高4B和CD就是822 米. 设图形的总面积是y平方米,那末 y=8-g22+是,2 就是 y=6-(受+2) -[+(门+ 历+4 一。(-2+18 6 2 匹+4/ π十4 由此可知,当龙=π十4的时候,y有极大值。 6 所以尺寸应该这样来设计:半圆的半径是_6+4≈0.84 米,或者说矩形钓底边长是号≈1.6s米 例4快艇和轮船分别从A地和C地同时开出,各沿着精头 所指的方向航行(图4·17).快艇和轮船的速度分别是40 公里/小时和16公里/小时.已知A0=145公里,经过多空 时间以后快艇和轮船之间的距离最短?(图中AC LOD.) 一145公里 406 D 图423 [解] 设经过t小时以后,快艇的位置在B,轮船的位置在 D。这时, AB=40t公里, ●87● ==========第198页========== 0D=16公里, B0=(145-40)公里. 根据勾股定理得 BD-:WB09+CD2-:(145-40)+(16)3, 现在要使BD最短,就只须使被开方数 (145一40t)(16ò2 有最小的值. 令 y=(145-40t)2+(16)9=185612-11600t+21025. 这个二次函数在 t、1600=31 3712 的时候有极小值,所以,快艇和轮船分别从A地和C地开 出3召小时的附候,它们间的距离最短. 可质 1.已知一个正方形ABCD.现在从它的四个顶点A,B,C,D分 47 别向B,C,D,A的方向截取粗等的线段AP,B,CR,DS. ②) 连接PQ,R,RS,SP,成一个正方形PQRS.要使这个正方 形的面积最小,这四条线段应该怎样截取? R ◆t080 ==========第199页========== 2.在半径是20厘米的圆内作一个内接矩形。这个矩形的面积最大可以是多少平方厘米? B 20厘米 D 3,有长200米的篱笆,要把它围一块矩形园地,问围成的园地面积最大是多少? 4.…个窗户的形状是矩形再加上一个等边三角形,如果窗户的周长是6米, (1)求窗户面积y(平方米)与等边三角形边长(米)间的函数关系; (②)怎样才能使透射进室内来的光线最充足? §48二次函数图象的应用 在前面几节里,我们学习了二次函数图象的作法,并且利用图象,研究了二次函数的一些性质。利用二次函囊前图象,我们还可以方便地讨论一元二次方程和一元二次不等式的解的各种情况,有时还可以利用它来求一元二次方程(不等式)的解的近似值. 1。一元二次方程和一元二次不等式的解的讨论 我们已经知道,二次函数 y=aax+bx+c(a≠0) 的图象是一条抛物线,它的顶点的坐标是 b Aac-b2a14g ·189● ==========第200页========== 现在我们在a>0的条件下,根据顶点的纵坐标4ac-79 4a 是正数,是零,或者是负数分别画出他们的图象(图4·24). y 0 (1)4ac-b9 >0 (2)4ac-b2=0 4a 4a 龙 0 4c-b2<0 4a 图4·24 从图象中,我们可以清楚地看出: (1)当4ac-b3>0,也就是b2-4ac<0时,抛物线 4a ·1900 ==========第201页========== 都在直线y=4c-6 Aa 的上方,不论心取什么实数值,对应 的函数值y都大于零,所以 方程a2+bx十c=0(a≠0)无解, 不等式a+bc十c>0(a>0)l的解集是实数集,不等式a2+bx+c<0(a>0)无解. (2)当4ac--=0,也就是b2-4ac=0时,抛物线和女轴相交于点(-名,0),并在a抽的上方,由此可知当x=-时对应的丽数位y=0.而当*一品时, 2a 对应的函数值y都大于零.所以 方程a2+b十c=0(a≠0)有两个相等的根 b =ー2a 不等式a2+bx+c>0(a>0)的解集是x≠-不等式ax+bx+c<0无解 (8)当4ac- Aa一<0,也就是2-4ac>0时,抛物线 和化轴有两个交点,设这两个交点的坐标是A(,0)和 B(B,O),这里B时,抛物线上的点在轴的上方,对应的函数值y>0;当&<心0(a>0)的解集是 优<&或C>B, 不等式2+bx+c<0(a>0)的解集是a&<花0时,函数是均匀增加的;(ⅱ)当k<0时,函数是均匀减少的. 2.方程x十by十c=0的图象 (四a40,6≠0.图象是直线y-名z- b (2②)a0,6=0.图象是直线=-分 特例:x=0(y轴)。 (③)a=0,6≠0.图象是直线y=-分, 特例:y=0(x轴). 3.用图象法解二元一次方程组的步骤 (1)先在同一直角坐标系里作出方程组的两个方程的图象; (2)如果这两条直线: (1)相交,交点的坐标就是所求的解;(i)重合,方程组有无数个解;(ii)平行,方程组没有解。 4.二次函数 (1)-一般形式:y=ax+bx+c(a≠0). (2)图象:抛物线; 对称辅:直线2名顶点:4(易。 开口:当>0的时候,向上; 当a<0的时候,向下。 01840 ==========第205页========== 5.二次函数划=ac2+bc+C的极值 ()如果a>0,当c=一多时,函数有极小值 小二4ac-3 4a 如果a<0,当龙=-时,函数有极大值 4ac-63 )极大产_Aa 6.一元二次方程心2+bc十c=0的图象解法 (1)先把方程变形成=a2-ca (2)i出函数y=a和y=ーb-。的图象 (3)读出它们的交点(如果有的话)的横坐标,即得所求。 复习题四A 1.(1)在同一直角坐标系里,作下列一次函数的图象: y=x+2;y=20+2;y=-3x十2 (2)在一次函数y=x+b(k≠0)中,如果b是一个定值,问当|l 的值逐渐增大时,函数图象的位置将会发生怎样的变化?这些图象有什么共同的特点? 2.(1)在同一直角坐标系里,作下列一次函数的图象: 방=+3,공-2 (2)在一次函数y=x+b(k+0)中,如果是一定值,问当Ib1 的值逐渐增大时,函数图象的位置将会发生怎样的变化?这些图象有什么共同的特点? 3.设y=f(c)是一次函数,f(2)=3,f(-1)=0. (1)画出这个一次函数的图象;(②)求f(x); ·195● ==========第206页========== (3)写出变量y与x间的函数关系式; (4)求直线y一(x)=0的斜率,倾角和在y轴上的截距. 4.(1)在同-直角坐标系里作出下面三条直线: x+y=5;2.c-y=7;x-y+1=0; (2)找出这三条直线所围成的三角形的三个顶点的坐标; (3)用解j程组的方法,验证上面得到的结果是不是正确。 5.按照下列条件,求直线的方程; (1)过A(&,0)和B(0,b)两点(a≠0:b≠0); (2)斜是%,并且过点(a,b). 6.弹簧的伸长和下面所挂砝码的重量成正比.现在设挂10克的砝码,弹簧的长度是12厘米;挂30克的砝码,弹簧的长度是16厘米.果以x克表示所挂砝码的重量,y厘米表示它所对应的弹簧长度,求变量y与x之间的函数关系.弹簧不挂砝码时长度是多少??。求下列函数的值或 1)y=号,当-2<<)(2②)-3名,已知这一函数的定义城是区间[-2,31. 8.设地面上的气温是15心时,km(千米)高空的气温是y℃,与心间的关系,可以近以地用关系式 y=:-16c+15 来表示。问 (1)每升高1km,气温下降几度? (2)离地面2km的高空处气温是多少度? (3)在离地面高度什么范围内,气温是在-10℃和10℃之间(可以等于一10℃或10°℃). 9.回答下面的问题: (1)在二次函数y=aa2中,a是正数或者a是负数,对于图象 的位置有什么关系? (2)在二次函数y=ax2中,a的绝对值的大小,和函数值增加 或减小的速度有什么关系? (3)在同一坐标系里,二次函数 y=ax2,y=ax2+k,y=a(c+m)2,y=a(ac+m)2+ 0106●: ==========第207页========== 的图象的形状间有什么关系?图象的位置间有什么关系?(以函数y=ac2的图象形状和位置为.比较的标准,) (4)二次函数y=a(+m)2+在什么情况下有极小值?在什 么情况下有极大值? (⑤)同一个二次函数能不能既有极小值又有极大值?能不能既 没有极小值,又没有极大值? 10.已知二次函数y=a+4r-5, (1)把它改写成y=Q(x十)2+的形式; (②)当取什么值的时候,函数的值是零?是正数?是负数? (3)当心取什么值的时候,函数有极大值或者极小值?这个极 大或极小值是什么? (4)当:取什么值的时候,函数的值随着x值的增大而增大?随 着x值的增大而减小? (5)画出这个函数的图象, 11.按照上题同样的步骤,研究下面的二次函数,并且画出它们的图象: (1)y=x2ト4x+5; (2)y=x2+4+4; (3)y=6-4x-2a2; (倒y=-子2+8-1; のーー-ー1 12,x取什么值的时候,二次三项式x2-5心+6的值是正数?是负数?等于零?有极小值?这个极小值是多少? [提示:只要令y=x2-5c十6,这个问题就变成研究在x取什 么值的时候,y>0,y<0,y=0,y有极小值,并求出这个极小值.] 13。用上题同样的方法,讨论下面这些二次三项式: (1)2x2-3x+4; (2)24c2+5x-3; (3)-x2+7-12; (4)‘-42+12x-9 14.已知二次函数y=ac2+br+c,按照下面的条件,确定a,b,c的值,并写出这个二次函数: (1)优=6时,y=0;x=4时,y极小=一8. (2)父=克州,则很大=25;如=0时,=24。 ●197· ==========第208页========== 15.已知抛物线y=a2+bx+c满足下面的条件,求,b,c的值,并 写出这个方程: (1)顶点是(6,-12),开口向上,并且和x轴交点之一是(8,0)(②)顶点是(2,一7),开口向下,并且和y轴有一个交点 (0,-15). 16,利用函数y=2的图象解下列二次方程(精确到0.1): (1)x2-2m2=0; (2)22-3x-4=0; (3)x2-2x-8=05 (4)5x2-2x-1=0. 17.解下列不等式: (1)(x+3)2<1; (2)(x+2)(3-x)<1; (3)x2+6x4-10>0; (4)22-3x十4<1. 18.矩形的周长是40米, (1)用x(米)表示矩形的长,y(平方米)表示矩形的面积,写出 y与x间的函数关系y=f(x); (2)矩形的面积在什么情况下最大?这时矩形的面积是多少?矩形有怎样的特殊形状? (3)画出函数y=f(x)的图象来加以验证, 复习题四B 1,一次函数y=ax+b(a≠0)的图象是一条直线,反过来能不能说平面直角坐标系里的任何一条直线都能确定变量y和x间的函数关系?如果能够,这种函数是不是都是一次函数?为什么? 2.已知f(x)=2x-6: (1)画出系数y=f(x)的图象; (②)从图象上观察,当x取什么值时,函数的值(1)等于零,(i)大于零,(i)小于零; (3)由此你能发现一元一次方程、一元一次不等式和一次函数 之间有什么联系吗? 3.(1)设(,y)表示平面直角坐标系里点P的坐标,下列集合 (i){(,y)12-y=3},(i){(如,y)|2x一y>3卧,(i){(x,y)12xr-y<3}的几何意义各是什么? 、0198。 ==========第209页========== (2)由此你能发现我]可以怎样应用二元一次方程的图象来解 二元一次不等式 x+bu+c>0或ax+by+c<0 吗?这里z,b,部是常数, (3)应用陶家法末至+号<1的正整数解(x,)的矣合. 注:含有两个末知数的一次不等式叫做二元一次不等式, 4.解下列方程组并根据参数:的值进行讨论: (k-1)x+2y=-2, (1) 2c+(e-1)y≈k一1;5x+3y=30, (2) 2x+-6y=30, 8x十ky9, 5.已!二次函数y=ux2+b:+c(x>0)的图象在第一象限的内部, (1)参数4、b、c应该满足什么条件? (2)如果另一二次函数y=g(x)的图象,和它有相同的顶点且形 状完金相同,只是开口方向相反,求9(x) 6.从53.9米高处以9米/秒的初速度垂直向上射出一枝箭, (1)利用公式H=53.9+49t-4.9,列出箭从射出到落到地面这一段时间里它的高度H(米)与时间t(秒)间的函数关系的表; (②)作出高度与时间关系的图象: (3)根据图象回答下面问题,再用计算来检验: (主)从射出到落地-一共经过几秒钟?(ⅱ)经过几秒钟达到最高处?()最高处离开地面多少米?(题中数据看成是准确数.) 7,画出下列二函数在指定的区间里的图象,然后,求函数在这区间里的最大值、最小值、极大值、极小值.自变量x在取什么值时,函数才能取得这些值: (1)y=f()=a2-4+1,x∈[0,3] (2)y=f(c)=x2-4红+1,x∈[2,3); (3)y=9()=-x2+一1,x∈[0,3]; ·199● ==========第210页========== (4)y=g()=一c2+4x-1,x∈(2,3). 由此你能发现定义在区间上的二次函数有哪些特性鸿? 8.已知二次函数刘=ax2bc+c(a≠0)当=1时有极大值9,方程x+bx+=0两根的平方和是20,求这个二次函数的解折式. 9。甲船以每小时15浬的速度,从A地向正东方向的B地航行;在 甲船出发的同时,乙船以每小时12裡的速度,从B地向正南方 向的C地航行.已知1,B两地相距20浬.问几分钟后两船 距离最近,这时的距离是多少? 10.挖一条截面是等腰梯形的水渠,斜面的倾角为45度,渠底和两腰总长为4米.问渠底的究度为多少时,水渠的流水量最大?这时水渠的高是多少? [提示:截面面积最大时,流水量也最大,] 第四章测验题 1.巴知y=()=2+bx+c,问什么条件下: (1)y与2成正比例; (2)y是x的-·次函数: (3)y是x的二次函数; (4)y=f(x)的图象是顶点在原点且开口问上的抛物线;(⑤)y∫()的图象是顶点在第三象限且开口向下的抛物线 2.已知…次函数y*(如)的图象平行于直线y=2x,二次函数y=(x)的图象是以A(3,一2)为顶点的抛物线,而且它们的图 象都经过点B(~1,6),求: (1)f(a)和F(): (②)直线y=(x)与抛物线F()=0的另一交点的坐标: (3)不等式(x)>F(2)的正整数解的集合. 3.已知二次函数y=()在x=2时有极大值3,方程(x)=0的 两根的刺为字,求这个二次函效的表达式。 4。无取什么值时,二次函数()=一十(心一1)的极大值为 (1)零;(2)i正数;(3)负数 5.取什么值时,二次涵数y=222-5x-12的图象 (1)与x轴相交: (2)在x轴的上方; 0200. ==========第211页========== (3)在x轴的下方; (4)是上升的; (5)是下降的. 6.三角形的两边的和是4cm,其夹角为60°: (1)试以其中一边为自变量,分别写出这个三角形的周长(cu)及面积S(cm)与它的函数关系式: (2)求这个三角形的最小周长; (3)求这个三角形的最大面积。 ·201● ==========第212页========== 5 有理数指数的幂函数 在前面二章里,我们学习了函数的-一些初步知识,并对几种常见的简单函数一一正比例函数、反比例函数、一次函数和二次函数的图象和性质,作了初步研究.在这基础上,这一章里,我们将应用集合的概念,进一步探讨一下函数这 一概念的意义,学习应用初等数学的知识研究函数性质的 方法,然后对一类以自变量的有理数指数的幂心(∈Q)给 出的函数,即有理数指数的幂函数的性质和图象作初步的研究. §5·1对应和函数 1。从集合到集合的对应 在第一章里,我们曾经学习过两个集合间的包含关系、相等关系以及两个集合间的几种运算(交、并、补),现在我们再来考察两个集合间的另一种重要关系.先看一个例子 设有两个集合: A={中国、美国、日本、英国、法国}, B一{北京、上海、天津、纽约、华盛顿、东京、伦敦、巴黎) 容易看出,这两个集合的元素间,存在着一定的关系。例如: (1)对属于集合A的各个元素(国家),按照 “是该国的首都” ·202● ==========第213页========== 这一关系来选择集合B中的元素(城市)并用箭头“→”连 接,就可得到图5.1所示. 北 京 国 北 京 上 海 上 海 冷 津 萸 国 天 津 纽 约 纽 约 华盛顿 日本 华盛领 东 京 东丹 英 国 英 国 伦 敦 伦 法 黎 法 国 巴 梨 图5.1 图5.2 2)对属于集合A中的元素,按照 “是该国的城市” 这一关系来选择集合B中的元素,并用箭头连接,就可得 到图5.2所示 象这样,已知两个集合A与B,用某种确定的法则、对 集合A的每一个元素,使集合B的元素与它搭配,这种关 系叫做从集合A到集合B的对应.这时,如果对于集合A 的元素a,选择了集合B的元素B与之搭配,就称b对应于a,并把它表示成 a-→b 观察上面的例子中画出的图形,可以看到这二种对应 有不同的特点.在图5·1中,从集合A中的每一个元素所 画出的箭头都只射到集合B中的一个元素,但是在图5·2 中,从集合A中某些元素所画出的箭头,却可以射到集合 B中不上一个元素.为了区别,我们把前面这种对应,叫做 单值对应. 一般地,设给定了两个集合A和B,如果按照某种对 ·203● ==========第214页========== 应法则,使A中的任何一个元素,在B中都有唯一的元素 和它对应,这种对应关系,就叫做从集合A到集合B的单 值对应.单值对应一般可用字母f,g,…来表示例1 判断下面各种对应关系,哪些是从集合A到集合B的 单值对应?哪些不是(说明理由,并用图形表示出来)。 (1)A={0,±1,土2,±3,B={0,1,4,9}, 对应法则:“平方”; (2)A={0,1,4,9},B={0,+1,±2,±3}, 对应法则:“开平方”; (3)A={0,士1,+2,土},B={整数}, 对应法则:“2倍后再加上1”; ④4=1,2,到,B={,是}, 对应法则:“取倒数”. [解] (①).·02=0,(±1)9=1,(±2)2=4,(±3)29. A中的每一个元素,B中都有唯一的元素和它对应,所以 这是从集合A到集合B的单值对应. 3-2 图53 图54 (2)因为除去0以外的正数,都有2个平方根,它们互为相反数, W0=0,士WI=士1,士W4=±2,土W9=土3 A中的元素1,4,9都有B中两个不同的元素和它对应, 所以这不是从集合A到集合B的单值对应. (3)设∈A,y∈B,那末对应法则可以表示成 2c+1=y. ·204· ==========第215页========== A的每一元素都有B中的唯一元素与它对应,所以这是从 集合A到集合B的单值对应. -3 -2 6 图5.5 (4)设x∈A,y∈B,那末对应法则,可以表示成 =y. A的每一元素都有B中的唯一元素和它对应,所以这是从 集合A到集合B的单值对应. 任细考架图5.3、图55和图56,可以发现这三个单值对应,也有一些区别 1°在图53中,A中的不同元素,可 图56 以和B中的同一个元素对应(如A中的2和一2,都和B 的4对应). 2°在图55中,A中的不同元素虽然都是和B中的 不同元素对应,但是B中的许多元素,在A中找不到它所 对应的元素(如B中的一4,一2,0,2,4,6在A中都找 不到和它对应的元素). 8°在图56中,A的每一个元素,都有B中的唯一元 素和它对应,A中不同的元素在B中所对应的元素也不 同,而且在B中的每一个元素都能在A中找到和它对应的 元素. 为了区别,我们把最后这种对应关系,叫做从集合A 到集合B上的一一对应.这也就是说: 设有两个集合A和B,∫是从集合A到集合B的单 ●208· ==========第216页========== 值对应,如果对于集合A的不同元素,在B中有不同的元 素和它对应,而且B中的每一个元素,在A中都有它所对 应的元素,那末这个单值对应就叫做从集合A到集合B上 的一一对应. 很明显的,如果手是从集合A到集合B上的一一对应, 那末,从集合B到集合A的对应g也是单值对应,并且是 一一对应 例如,f:心-→2x=y是从A到B上的一一对应,那末g →y=就是从g到4上的-一对应(图5. f:→2cーy 29 3 图5.7 一般地,设于是从集合A到集合B上的“一对应,它 使集合A中的每一个元素x与集合B中的元素y相对应, ●208◆ ==========第217页========== f:x→y(x∈A,y∈B), 那末也就存在一个对应关系9,能使集合B中的每一个元 素y与集合A中原来和它相对应的元素心相对应. g:y→(则∈B,x∈A), 这个从集合B到集合A上的一一对应g,叫做对应手的逆对应 对应∫的逆对应,通常也用符号1来表示. 很明显,f-1是于的逆对应,那末于也是f1的逆对应.也就是说,它们是互逆的.注 本书中只是在f是一一对应时,才研究它的逆对应。 例2 下列各题中的对应关系是不是有逆对应?如果有,把它找出来. (1)f:R→2x-3=y(c∈R,y∈R); (2)g:→2=y(a∈R,y∈{yy≥0}); (3)h.→x2=g(x∈{cx≥0},y∈{gly≥0}). [解] (①)对应f是从R到R上的一一对应,它有逆对应。 由g=20-3,得x=y十82 fg8-8gE尾,. (2)根据题设条件,从y=可求得 化=士Ny. 这说明对应g不是从R到{yy≥0}上的一一对应,所以它不存在逆对应 (3)根据题设条件,从y=2可求得 a-vy. 这说明对应h是从{xx≥0仍到{yy≥O}上的一一对应,它有逆对应 b-1:y→N则=c(y∈{uy≥0仍,∈{c|≥0}). ●2070 ==========第218页========== 习题 1.下面各图所示的对应,是不是从集合A到集合B的单值对应? 51 为什么? (1) (2) (3) (4) 2.已知集合Y是由集合X={一2,-1,0,1,2}中的元素,按照 对应法则平方后再减去1”所得到的元素所组成的 (1)用列举法写出这个集合; (②)用图形把X到Y的对应关系表示出来; (3)这个对应关系是不是单值对应? (4)反过来,从Y到X的对应关系是不是单值对应?为什么? 3.下面的对应关系是不是从集合A到集合B上的一一对应?为 什么(题中x表集合A中的元素,y表集合B中的元素)? (1)A={自然数},B={实数},对应关系:x→2=y; (②)A={c≠0,B={实数},对应关系:→ 少 3)4={实数,B=实数},对应关系,→x=劣(④4=正实数,B={正实数,对应关系:x量=. 4.下面的单值对应是不是从集合X到集合Y上的一一对应?如 果是,求出它的逆对应. (1)f:→2+1=,X={x-o1 ◆208· ==========第219页========== (2)头g→是-,x=41z*引,了g*》 2。函数 理解了上面所讲的这些概念,也就可以体会到在前二章里所学习过的那些函数: (1)正比例函数y=x(≠0); (2)一次函数y=kc+b(≠0); (3)二次函数y=ax2+bc十c(a≠0) 都是从集合R到集合R的单值对应,而反比例函数 (k≠0) 则是从集合X={≠O}到集合R的单值对应. 由此也就可以理解到,在函数的概念中,涉及到三个因素,即自变量心所在的集合X,,与之对应的因变量y所在的集合Y,以及这两个集合的元素和y间的对应法测于。只有这二者完全确定了,我们才能说这个函数是确定了. 这里我们需要再说明儿点: (①)现在我们所研究的函数,集合x和Y都是限于 由一些实数所组成的集合,对应关系都是从X到Y的 单值对应. (2)在一般地研究某一函数y=f(c)时,y所在的集 合Y,认为就是实数集R;所以给出一个函数,可以不把它 指出,简单地写成 y=f(c),c∈D或f(),∈D 这里D二R表示函数的定义域。特别的,如果函数的表达 式能用一个解析式来写出,还可以把函数的定义域略去不写,简单地写成 y=∫()或f(c. 这时就认为函数的定义域D,就是能使这个解析式有意义 ·209● ==========第220页========== 的一切的值所组成的集合. (3)函数y=f(x),x∈D的值域E,一般可以表示成 E={yy=f(x),x∈D}或E={f(x)x∈D}. 这个集合可能就是实数集R,也可能只是实数集的一个真 子集,即 ECR. 例组,反比例面数y=至化+0是从巢合D-≠ 到集合R的单值对应,这里函数的值域 E-{g多&0,eD} ={yly≠O}cR, 是实数集R的一个真子集. (4)表达式相同而定义域不同的两个函数是不同的函数.例如函数 y=22,花∈R与y=a2,x∈(0,+o∞) 虽然表达式相同,但是定义域不同,它们是不同的函数.定义域相同,表达式不同,但是对于定义域里自变量的一切值所对应的函数值都分别相同的两个函数,则认为是相同的函数.例如函数 4一1 2+1与=x2-1 是相同的函数 ()我们目前所研究的函数,f()都可以用x的一个解析式来表示,但是也应注意,并不是所有的函数都能用这种方法表示出来, 例如,寄信时,信重不超过20克付邮资8分,超过20克而不超过40克付螂资1角6分,那末信重x克与邮资y分之间的关系是 ●210● ==========第221页========== 9-E 这个函数的定义域是(0,40],但是在两个区间(0,20]和(20,40]上,y与c间的对应法则就不相同.这个函数的图象,如图5.8所示. 16 o이10203040 图5.8 注 这种函数叫做分段函数, (6)通常我们说函数y=f(c),x表示的是自变量,y表示的是因变量.自变量:是在函数的定义域里变化着的,但是它所对应的值,有时也可能是一个常数.例如上面这个例子中,我们如果研究信重心(克)在区间(0,20]上变化时,应付邮资y(分)的变化情况,那末就有 y=f(e)=8,x∈(0,20]. 1. 习题 设f:-→x=y是集合孔到集合R的单值对应: 51 (1)能不能说y是的函数?为什么? 2) (2)求f(c)[不用绝对值符号]. (3)画出y=f(x)的图象. 2.茶杯每只0.6元,身边有钱4元. (1)如果至少要买一只,可有几种不同的买法? (2)如果把所买茶杯的只数记作x,所付的钱数记作y(元)列出算式 y=0.6x, 并说这就表示了本题中y与x之间的函数关系,是不是正确?为什么?应该怎样改正? (3)求本题列出的函数的值域。 0211◆ ==========第222页========== (4)画出函数的图象.它与我们过去所学过的正比例函数的图象有什么区别?为什么有这种区别? §5·2函数的一些重要性质 在第四章里,我们曾经应用描点法,画过最简单的二次函数y=x2的图象,并从画出的这条抛物线的特征,讨论过这种函数的一些重要性质 用描点法我们可以作出最简单的三次函数则一3的图象(图5.9),这个图象叫做三次抛物线.利用这个图象,我们就可以发现这种函数的一些重要性质,对比如下: 函 数 图象的特征 y=f(c)=3 的性质 1.图象通过 1.当x=0时, 原点,并且从这 y=0.函数的值域 点起向左下方和 是实数集B. P(x,y) 右上方无限伸展. 2.图象对称 2.x取互为相 于原点① 反数的值时,对应的函数值也互为相反数 f(-x)=-f(). P(一x,一 3.图象自左 3.函数值y随 至右始终上升 自变量x值的增大而增大。 1<3→ f()m(或者f(x)≥m) (2) 成立,那末函数f(x)叫做有下界的函数;如果这两个不等式都能成立,就是 mf(x), (2) 那末就说函数f(x)在这个区间上是递减的,函数f(x)叫做这个区问上.的减函数. 某-一区间上的增函数和减函数,统称为这个区间上的单调函数,而这个区间就叫做函数的单调性区间.注 当然上面所说的区间,也可能就是函数f(x)的整个定义域,这时我们就说函数f(x)在:整个定义域上是增函数或者减函数 例3 证明:函数f(x)=2在区间一o0就可以了 [证明] 设化1i是在区间一∞<化<0上自变量x的两个值,且化1<<0,那末 f(1)一f()=ai-2=(1-xg)(1+xg). ●217◆ ==========第228页========== ”.'x10, 就是 f(x1)-f(c)>0. ,.f(x1)>f(g). 这就是说,函数f(x)在区间一∞<化<0上是减函数.在研究函数的增减性时,我们常常可以利用函数的奇偶性,从函数在一个区间上的增减性来推导出函数在另一区间上的增减性.例如,在上题中,我们知道函数y=心2是偶函数,它的图象对称于y轴,从而也就可以从函数y=心2在区间(一o,0)上是减函数,推出它在区间(0,+∞)上必然是增函数. 例4 证明:函数f(c)=x3在整个定义域上是增函数. [审题] 因为f(一心)=一f(c),函数f(x)是奇函数,所以只需证明在区间[0,+∞)上是增函数,就可推出它在整个定义域上是增函数 [证明] 函数的定义域是一00,从而有 f(1)一f(2)<0,f(c1)0,但无上界 值 域 {yy=0} fyiy>o 极 值 无 无 这些性质,也可以根据函数的解析式,直接推导出来,举例如下 例1 讨论函数y=化8的性质,并画出它的图象。 [解] ()婴使有意义,≠0.所以函数的定义域是 {x≠0},即两个开区间(-∞,0)U(0,+∞). .221· ==========第232页========== (2)设四-是因为 f(一x)= =1 -f(), 所以函数f(x)是偶函数. (3)设c1,g是自变量的任意两个值,并且x1<,那末 f()-f()=1-1=(=)(+ (U13)3 如果心1,都属于区间(0,+∞),则有 2+x1>0,9-i1>0,(c1心2)3>0.∴.f()-f(2)>0,即f(x)>f().由此可知,函数在区间(0,十∞)上是减函数. 因为()是偶函数,从而可知函数在区间(一∞,0)上是增函数 (④因为不论是正数或负数,子总是正数,当的绝对值愈来®增大时,子的值就愈来愈接近0;当心的笔对值愈来愈接近0时,是的值就愈来愈增大,面没有 限制.所以函数有下界y>0,但无上界、函数的值域是{yy>0}. 描出下表中各点,并利用图象的对称性可作出函数y=x-9的近似图象(图5·13). 1 3 2 2 9 0 图513 0222◆ ==========第233页========== 3。有理函数 由有理代数式确定的函数,叫做有理函数;其中由整式确定的,叫做有理整函数(或多项式函数),由分式确定的,叫做有理分函数.例如函数 y=kxfb(k≠0),y=a22十bx十c(a≠0),y=等等都是有理整函数,函数 ー(+の),=8,y=t+ cx+d(c≠0) 等等都是有理分函数 很明显,有理函数都是由一些整数指数的幂函数与常数,通过四则运算构成的.因此,我们也可以利用已知的整数指数幂函数的性质,来对他们进行研究或画出它们的图象.下面举一个简单的例子。例2 已知函数y=3-4 Eー2 (1)求函数的定义域和值域; (2)画出它的图象; (3)讨论函数的增减性, 3x-42 [解] -2 t之+3, 所以这一函数就是 2 2-9下3. 图5.14 (1)函数的定义域是{xx≠2},值域是{yy≠3}. (2)函数的图象可以利用函数y一2的图象画出.即把函数y=的图象沿心轴方向向右移动2个单位,沿y轴的方向,向上移动3个单位画出,它也是两支双曲线: ·228◆ ==========第234页========== 直线=2和y=3是它的两条渐近线. ③)象函效-品一祥,这一西数在定义城的两个区2 间里都是减函数. 习“题 1.已知通数y=3+1 53 (1)求函数的定义域和值域: (2)讨论它的奇偶性、增减性、有界性: (3)画出它的图象 2.以国数划=二的图象为基础,面出下列函数的图象: (1)y=- (2)y=2 x-Ti 2 (3)y= 2+1-3(4)=一 3x小-12+1 §5·4分数指数的幂函数、无理函数 1.分数指数的幂函数 由等式 -f y=x9或y=x 给出的函数,叫做分数指数的幂函数,这里”是一个概约分数,p、9都是自然数,且q>1. 根据分数指数幂的意义,这类函数也可以应用方根表示成 y=又/P或 例如, 割=2√r(≥0;y=是-n(rE:gy-(ER3g=-1(x>0).2 下面我们来研究几种简单的分数指数的幂函数的性质 ·224· ==========第235页========== 和图象. (1)函数=c2因为当≥0的时候幂(就是√)都有意义,所以函数y=2的定义域是全体非负实数,即区间0,∞). 从数的解折式f()一五可以看出,它具有以下的-一些性质 (1)当x=0时,∫(w)0,所以函数的图象通过原点. (2)当x取负值的时候,∫()没有意义,所以函数y=x不具有奇偶性.函数的图象位于y轴的右边,既不对称于轴,也不对称于原点 (3)当x取非负的值的时候,√x的值总是非负的数,就是√心≥0;并且当0≤1). 这里我们可以看到为了使方根与有理数指数幂一致起来,当底数 a<0,指数是一个既约分数,且加为奇数时,我们还应规定它的意 义就表示j根/am. 但是,在幂ai中如果是一个偶数,那末当a<0时,它在实数集里就没有意义,例如(一2)云,(一2)音在实数集里都没有意义.应该注藏,(-2)8=/一2是有意义的,所以(-2)÷(-2)3 (3)函数y=x导因不论心是什么实数,3即8 都有意义,所以函数J=如的定义域是实数集R 因为(ーの3ー-[(ー)=(の)一,所以函数y~3是偶函数 因为y==/,为了便于讨论,我们可以先令=u,把y值的计算分成两步: u=a3, y=シau= 这样根据幂函数u=x2与y=u3(u作为自变量)在区间[0,十∞)上都是增函数的性质,就可知道y=3在区间[0,十∞)上也是增函数. 应用描点法,先作出它在第一象限内的图象,然后再利用对称性,就容易作出它的图象如图5·17所示 3 U=U2 0 y 4 =2 0 1.62.1 图5.17 ·227● ==========第238页========== (4)函数yーェ因当>の时,(是)才有意义,所以函数y=x三的定义域是正数的集合,即区间(0,+∞). 仿照研究负整数指数的幂函数那样,我们利用已知的正分数指数的幂函数y-od(即yY) 的性质和图象,容易道:函数y=$不具有奇偶性;它是一个减函数;它的图象是 图5.18 在第一象限内的一条曲线,这条曲线也经过点(1,1),在这点向左与y轴的正方向无限制地靠近,在这点向右与x轴的正方向无限制地靠近(图5·18). 2,无理函数 一切由代数式给出的函数,都叫做代数函数,其中对法则不能用有理代数式给出的代数函数,叫做无理函数.例如函数 y25-, y=그 +3, 等等都是无理函数,但是因为/(x+1)=(c十1)①,所以函数y=()就不是个无理函数.例 求下列函数的定义域: (1)y=(2-3x+2): (2)y=(1-)+(2+041)克. [解] (1)函数的定义域由不等式 ①“=”是恒等的符号、 ◆228● ==========第239页========== a2-34-2>0,即(x-1)(-2)>0 所确定.解得x<1,>2.所以函数的定义域是{<1或x>2},即两个开区间(-°,1)U(2,+∞). (2)函数的定义域由不等式组「120, (-1)(x+1)≤0, 即 しx2÷-+1≥0, x2++10 所确定.因为对于任意的实数值x,总有 心++1-(e-》+星>0, 所以这个不等式组的解集是一1≤≤1.函数的定义域是闭区间[-1,1]. 习题 1.利用函数y=√x的图象,求下列方程的近似根(精确到0.1), 5•4 并查表进行验算: (1)2\'x-3x+6=0; (2)10Vx+4x-40=0 [提示:们照-一元二次方程的第二种图象解法。] 2.利用函数y=/无的图象,求下列方程的近似根(精确到0.1)。并查表进行验算: (1)28/E-=0; (2)G+43-8=0. 3.以函数y=√的图象为基础,在同一坐标系里画出下列函数的图象: (1)y=2/c; (2)y=2Vx-1; (3)y=2Vx-1+3. 4.已知函数y=: (1)求函数的定义域和值域: (2)讨论这…函数有哪些重要性质; (3)画出它的图象. 5.求下列函数的定义域: (1)y=V6+1 V (2)y=(2-3x+2)+(3+2m-x) ●220● ==========第240页========== §55反函数 1。反函数的意义 在研究变量间的函数关系时,究竟把哪…个变量看做是自变量,需要根据问题的性:质来决定.例如,在前面我们 研究过的铁的重量W和它的体积V之间的关系,如果我 们考察当体积V变化时,重量W的变化情况,那末应该把 ”看做自变量,列出函数关系式 W=7.8V, (1) 如果我们要考察重量W变化时,体积'的变化情况,那未 就要把W看做自变量,列出函数关系式 V-isw. (2) 容易看出,在(2)中的对应关系 W7.8W-V 是(1)中的对应关系 V->7.8V=W 的逆对应,所以如果把表达式(①)一般地表示成 W=f(V), 那末表达式(②)就可以一般地表示成 V=f-(W). 我们把函数V=f-1(W)叫做函数W=f(V)的反函数 我们再来看一个例子:设已知的函数是 y=x2+1(x≥0). (8) 这个函数的定义域是x∈[0,+∞),值域是yE[1,+∞),对应关系 f:→x+1=y ◆280· ==========第241页========== 是从集合[0,十∞)到集合[1,+∞)上的一一对应,因此它有逆对应 f-1:y>yー1=0, 这个逆对应就确定了一个以y为自变量的函数 =yー1。 这个函数的定义域是y∈[L,+∞),值域是x∈[0,+∞). 函数x=√y一1是由函数y=:1(x≥0)导出的,我们把它叫做函数y=2+1(x≥0)的反函数 从这个例-子可以看出,函数y=2十1(c≥0)的定义域利和值域,分别是它的反函数x=√y一1的值域和定义域. 一般地,如果给定的函数y=(x)的对应关系于是从 它的定义域X到值域'上:的-一一对应,那末由∫的逆对 应于1所确定的函数=∫1()就叫做函数y=∫(x)的反函数.函数y=∫(x)的定义域和值域,分别是它的反函数x=f-1(y)的值域和定义域, 〔注意〕 上面这个函数如果改为y=2+1,那末它的定义域是x∈(-∞,+∞),值域是g∈[1,+∞),对应关系虽然仍旧是心-241=y,但不是从集合(-∞,+∞)到[1,+∞)上的一一对应.所以函数y=c2+1在它的整个定义域(一∞,∞)上就没有反函数.事实上,从y=心+1可导出=士/y-1,→±√y-1=.这不是单值对应.例1 求下列函数的定义域和值域. (1)9=20-3 x+1 (②)=11, [审题] 函数y=f(x)的定义域,可以直接从解析式f()看出,但值域不易看出.·因为函数y=∫()的值域,就是它的反函数c二∫-1()的定义域,所以可以先找出它的反函数的表达式,这样一切能使解析式∫(y)有意义的y的值的全体,就是所求的值域。 ·231● ==========第242页========== [解] (1)函数y=2x-3 +1的定义域是{≠一1}.2x-3 +1 =(a+1)=2m-3→(9-2)x=-(y+3)=→a=+3 2-则 函数=号+8的定义域是侧y≠2. 2ーy .函数y=2x一3的值域是{y≠2头. x+1 (2)函数y=一1的定义域是{>0.y=1-1→Nx-1 +ig+1>0) 1 g+1){w>-1), 今化= .函数y=,-1的值域是>-. 〔注意〕 在(2)中推出√元1十以后,应该及时根据算术根 的意义,把y的取值范围指出,否则就会得出错误的结论{yly÷1}.其实做到这一步以后,原来这个函数的值域已能找到,所以下一步求反函数也可省略. 在研究一般的函数时,我们所要讨论的只是确定两个集合的变量间的对应法则,用什么字母来表示自变量,什么字母来表示因变量,是无关紧要的.因为习惯上,我们总是用字母x表示自变量,y表示因变量,所以求函数y一∫()的反函数、在导出了x=∫-1(y)这一关系式以后,通常还要把表示变量的两个字母对调,把它写成y=∫-1(心),并说函数y=f(w)的反函数是y=f-(心).例如 (1)函数y=2+1(w≥0)的反函数=Ny-1(y≥1)通常表示为y=W/-1(c≥1)的形式; ●282· ==========第243页========== (2)函数y=2-3(x≠一1)的反函数 x+1 +3 2-y(y÷2) 通常表示为y=六3 2一x(x≠2)的形式. 例2 求函数y=√:+1的反函数.并指出它们的定义域和值域(用不等式表示).[解] 函数y√心+1的定义域是0≤心<十o,值域是 1≤y<十∞ 用y的代数式表示c,得到以y为自变量的反函数x=(y-1)2.定义域1≤y<+∞,值域0≤<+∞.交换字母得以心为自变量的反函数 y=(x-1).定义域1≤<+o,值域0≤y<+∞从上面这个例子,可以看到求函数y=∫(x)的反函数 一般可采用以下的步骤: 以y为自变量 原函数 的反函数 反函数 ①解出 ②交换字母yf(a) →x=f-1(y) y=f-1(a) 定义域2∈X Xy∈Y ∈Y 值域y∈' x∈X y∈X 习题 求下列函数的反函数(表示为y=(x)的形式),并指出反函数的定义 55 域和值域: 1) 1.y=-上 -Ti 2.y=x3+1 3.y2+3 -i 4.y=x2-1,(x<0)5 5.y=/c1.-…2 6.y=x3-1. 2.反函数的图象 很明显的,函数y=∫(x)的图象和它的反函数x=f-( ◆233· ==========第244页========== 的图象是同-一个图象,只是前者以:轴上的点所对应的数作为自变量,而后者则以y轴上的点所对应的数作为自变量.现在我们来研究,在变换了字母以后,在同一直角坐标系里函数y=于(x)的图象与它的反函数y=-1(心)的图象间有怎样的关系 先来看一个例子. 例3 在同一直角坐标系里,作出下列函数及其反函数的图象 (1)2y=3-2; (2)y=. [解] (1)由y=3x-2,得 x=2 3 交换字母,得函数则-3.a一2的反函数是 化2 y=8 他们的图象如图5·19所示. y 2=32-2 x中3 L-v7 1 3 2 图5.19 图5,20 (2)山y=,得x二3y.交换字母,得函数y=3的反函数是 ●2340 ==========第245页========== 它们的图象如图5·20所示, 从上面例子中所画出的图象,可以发现一个重要的事实 在同一平面直角坐标系里作出的函数y=∫()的图象和它的反函数y=f1(w)的图象,关于第一、第三象限的角乎分线,即直线y=为对称. 注 这一事实,可以这样来证明: 设I(,物)是y=(x)的图象上的任意一点,即当x=时,f(ao)=20. ∵リ=ず1()是y=(x)的反函数, .当c=0时,于-1(6)=心o, 即点M'(o,)在y=-1(x)钓图象上. 1°如果=0,那末1M(,)和M'(0,)是同一个点,而且 在直线y=c上, ly=f(a) y=8 Bey=1() N 图521 ·2359 ==========第246页========== 2°如果+0,那末由图5·21所示,可以证出 △INO≌M'N'O, .∠NOM=∠N'OM',MO=I'O 由此可知,△1M'O为等腰三角形,直线y=x平分∠M0M',且是线段IM的垂直平分线,从而也就可以知道点M(o,)和点 M'(o,o)关于直线y=x对称. 因为点是y=f(x)的图象上的任意一点,所以y一(x)的图象和它的反函数y=f-1(c)的图象,关于直线y≈x对称. 利用这个关系,我们就可以从函数y=f()的图象作出它的反函数y=1(x)的图象,并且从函数y一f()所具有的性质推出它的反函数所具有的性质.例如从图5·20中可以看出,增函数y=3的反函数y=/就仍旧是一个增函数,等等 习题 1.在坐标系里,作出直线y=x,然后画出下面这些点关于直线 55 y=x的对称,点: A(2,3); B(1,0); C(-2,-1);D(0,-1). [提示一点A(x,y)关于直线y=x的对称点就是A'(,).] 2.(1)在坐标系里先画出函数y=2a3的图象,然后利用这个图象 3 画出函数y=√艺的图象; 3第 (2)从图象上可以发现函数y=2x和y=V之有哪些相同的 性质? 3.以函数y子的图象为,说明在同一坐标系里: (1)y=(x)的图象和x=f-1(y)的图象有什么关系? (2)x=f1(y)的图象和y=f-(x)的图象有什么关系? (3)y=(心)的图象和y=f-1(x)的图象有什么关系? 4.以函数y=2的图象为例说明 (1)函数y=f(c)如果在它的定义域里不是单调函数,那末在整 个定义城里就没有反丞数: (2)把函数y=(x)的定义域,划分成几个单调区间以后,那宋 在每一个单调区间上都有反函数 ◆280· ==========第247页========== [提示:先画出函数的图象,再作一条与y轴平行的直线, 如果这条直线和图象最多只能有一个交点,那末就有反函数,交点不止1个,那末就没有反函数,] *§561单值函数和多值函数 一个实数x的平方y,可以用等式 y=x3 (1) 来表示,这里,当心取任何一个实数值的时候,y都有一个非负的值和它对应,所以y是:的函数 反过来,如果我们问,什么数的平方等于一个非负的实数x呢?把所求的数记做y,那末y可以由等式 y=士√x(≥0) (2) 来确定.这里,当心取任何一个非负的值的时候,y就有两个确定的数和它对应. 这种对应关系不是单值对应,所以根据我们前面所给 ·出的函数的定义,不能就把它说成y是心的函数. 为此,在数学里还引进了多值函数的概念: 如果两个变量x和y由一个确定的法则联系着,当变量x在它可取值的范围里,取每一个确定的值时,变量侧有不止一个确定的值和它对应,就说变量y是变量的多值函数,变量龙叫做自变量 与之区别,我们也把前面所研究的那种对于自变量的每一个值只对应着变量则的唯一的值的函数叫做单值函数 从等式(②),可以看到这个多值函数,可以分拆成两个单值函数 y=√x,(≥0,y≥0),y=-Nx,(>0,y<0). ●237· ==========第248页========== 研究了每一个单值函数之后,就可以知道原来这个多值函数的性质,本书中,以后所提到的函数,如果没有特别说明, 一般都是指单值函数.例1 已知变量y和心间的关系是 x2+y=25, (1)以作自变量,写出y和x的单值的函数关系; (2)画出它的图象. [解] (1)由 2+y2=25, (1) 得 y=士25-, (2) 等式(②)可以分成两个式子: y=W√25-2 y=-√25-x2 这就是所求的两个单值函数, (2)函数的图象如图5·22.在x轴上边是函数 y=V25-0 1 则=√5-x ya-'25 图5·22 ·238· ==========第249页========== 的图象,在心轴下边是函数 y=ー√252 的图象, 例2 求函数y=x2+1的单值的反函数. [解] 从 y=2+1, (1) 得 c=士√yー1。 (2) 为了求单值的反函数,我们把(②)分成两个式子: C =y-1 (y≥1,c≥0), x=-√y-1(y>1,花<0). 这就是说,函数y=22+1的区间0≤x<十∞上的反函数是 =√yー1, 把字母心和y对调,就是 y/c-1 而在区间一○<<0上的反函数是 =-√y-1, 把字母x和y对调,就是 g=-√c-1. 从这个例子可以看到,果函数()不是一个单调函数,要求出函数y=∫()的单值的反函数,可以先把函数的定义域划分成儿个单调区间,再找出它在各个单调区间上的反函数.例如在例2中函数y=心2+1在区间 一心<≤0上是单调递减的,在区间0≤化<+∞上是单调递增的.我们就可以在这两个单调区间上找出它的单值的反函数 注 因为x=0既可以属于区间一∞m一有下界的函数; (iv)f()m中至少有一不成 立一无界函数. (3)增减性:设1和是自变量在定义域的某一个区间上的任意两个值,而且心1f(2)-减函数。 3.反函数 (1)求法 原函数 反函数 逆对应 c=f-1(y)交换字母 y--f(a) y=f(知) 定义域∈Xy∈Y x∈Y 值域y∈Y X x∈X y∈X (2)图象y=f(c)与心=f-1(y)是同一图象, y=f(c)与y=f-()的图象对称于直线 y=化 4.有理数指数的幂函数由等式y=心所确定的函数,式中”是有理数。 ●241· ==========第252页========== 儿种常见的有理指数的幂函数的图象和唑质 性 质 函数定义性值域 图 象 奇偶性有界性:增减性 8+yV818+y=V8 奇 无界 递 增 在:区间 -∞o 奇无界递增 ●242· ==========第253页========== (续表) 性 质 函数定义域值域 图 象 奇偶性有界性增减性 在区倒 8 -0<<0 有下界 递 减 1=-a2> /20 偶 y=0在区间 8 0≤w<十o 递 增 有下界 彩e℃营20≥0 非 递 增 U=0 复习题五A 1.回答下的问题 (1)能不能说一个函数如果不是偶函数,就一定是奇函数?为什么? (2)能不能说一个函数如果不是增函数,就一定是减函数?为作么? (3)能不能说一个有界函数,一定也是有上界的函数,并且也是有下界函数?为什么? (4)能不能说一个无界函数,一定不是有上界的函数,而且也不是有下界的函数?为什么? 2.明: (1)两个偶函数的和或者差所组成的函数仍旧是偶函数; ·243● ==========第254页========== (2)两个奇函数的和或者差所组成的函数仍旧是奇函数。 [解法举例:(1)设f(x)Ip(x)是偶函数,那末 f(-)=f(x), p(一x)=p() 设把(x)与p(x)的和组成的函数是'(),就是 I'(E)=f(x)+p(x), 那宋 F'(-)=j(-x)十p(x) =f()中()=() 所以(a)是偶函数.] 3,确定: (1)两个偶函数的积所组成的函数是不是具有奇偶性? (2)两个奇函数的积所组成的函数是不是具有奇偶性? 4.判断下列函数,分别属代数函数中的哪一类型,并求出它们的定义域和值城: (1)y=22+-1; (2)y=+; (3)y=1+V-I; (4)y=x-1 5,研究下面这些函数(定义域,值域,奇偶性,有界性,增减性)并作出它们的图象: 1 (1)y-+1; (2)y=+I 6.(1)在什么条件下,函效y=f()在它的整个定义域里才能存在 (单值的)反函数? (2)求下列函数的反函数,并写出反函数的定义域和值域: (i)y=4+1(x<0); (ii)y=可+T' (3)反函数的定义域、值域与原函数的定义域和值域之间有· 么关系? (4)函数y=f(x)的图象与函数y=f-1(x)的图象之间有什么 关系?根据这一结论,利用上题画出的图象,画出求得的这两个反函数的图象 7.求下列数的定义域和值域 四f“群古士+2 (2)f(x)=W√4-x-1, 。244 ==========第255页========== 8.证明: (1)一次函数y=x+b(a≠0)的反函数仍是-次证数: (2)有理分函数y=x+”(c≠0)的反函数,仍是有理分函数; cx+d (3)有理数指数的幂函数,如果有反函数,它的反函数也是有理 数指数的幂函数. 复习题五B 1.把下列关系式表示成y=F(x)的形式(不化简),并求函数F()的定义域 (1)y-1+ー,w-1+,-고+(②)y=1+w,u=1-5,u=4r+2. 2.已知变量x和y间有下面的关系: (1)22+3y2-2x+2y-14=0; (2)x2-y2+2+2y-25=0 写出以x为自变量的函数关系式(限于单值函数),并指出其定义域 3,证明下列各函数是有界函数: 6x (1)f(c)=1+r-3c-4(2)f()=4주4,讨论下列各函数的主要特性,然后作出它们的图象: (1)y=V2x-x; (2)y=4-6a2+8. [提示:根据作图的需要来进行讨论.] 5.已知函数y=f(x),∈(a,b) (1)如果a<11). 4.已知形=(x=V3-44 a (1)求函数f(x)的起义域; (2)如果不用绝对值符号,这-一函数应怎样表示? (3)画出函数=(x)的图象,并指出函数的一些特性。 5.已知函数y=1+①+ Vi (1)对这一函数的定义域、值域、奇偶性、单调区间、极值等特性 作出出讨论(要成应用已知幂函数的性质). (2)根据函数的特性,用猫点法画出它的图象 h 甲 乙 丙 ●49● ==========第257页========== 6 指数和对数 以前我]学过的运算加法和减法,乘法和除法,乘方和开方(包括求有理数指数的幂)都是代数运算.这一章我们将学习数学里另一种重要的运算一对数运算,并引进对数的概念.首先我们要把代数第二册里学过的有理数指数幂的概念扩展到实数指数幂, §6·1幂的概念的扩展 在代数第二册里,我们曾经引进了有理数指数幂的概念,分别规定了有理数指数幂的意义: n个 a”=aua (n是正整数,a是任意实数), 1 G (n是正整数,a≠0), aa (m是正整数,%是大于1的整数,a≥0), (m是正整数,z是大于1的整数,a>0). 我们还知道,对于有理数指数幂的运算,有以下的法测: Q"i.Q-=ar, (w)=aw, (ab)"=ar.br 上面的?,T1,2都表示有理数;a,b都表示正数 利用这些关系,可以使某些计算得到简化.但是在实标 ●247● ==========第258页========== 计算中,只有有理数指数的幂还不够,我们还有必要引进无理数指数的幂的概念.这就是说,我们需要把幂的概念推广到指数是任意实数的情形. 现在,我们看应该怎样来规定无理数指数的幂a的意义 让我们先从一个具体的例子102着手. 我创知道,任何-个无理数,如果取它的特确到品 的不足近似值或者过剩近似值,就可以得到一串有理数.例如√Σ的不足近似值和过剩近似值是-一串有理数: 精确度 1 1 10 102 1G03 不足近似值 1.4 1.41 1.414 过剩近似值 1.5 1.42 1.415 我们应用√2的这些近似值,写出相应的幂: 101.4,101.4红,101.4红4 (1) 101.5,101.42,101.45,… (2) 因为上面这些幂的指数都是有理数,所以每一个幂都表示着一个确定的数. 因为当底数大于1的时候,指数愈大,这个幂的值也愈大,所以在第一串数(1)里有 101.4<101.4红<101.幻4<…, (3) 在第二串数(2)里有 101.5>101.42>101.415>… (4) 并且第一串数里的每一个数,都小于第二串数里的每-一个数,就是 101.4<102.41<101.4红4.…<101.415<101,42<101.5 ·248● ==========第259页========== 当2的不足近似值和过剩近似值的精确度愈来愈高的时候,这两个近似值也愈来愈接近;相应的用10做底数,并且用这些近似值做指数的幂也就愈来愈接近, 这就是说,夹在(3)和(4)这两串数之间,一定有并且只有-一个实数存在①.我们就把这个实数规定是10②。这也就是说,我们规定10②是满足条件 101.4<101.41<101.414<…<10Vg<… <101.416<10.49<1015 的唯一实数 同样,我们规定(品)是满足条件 ()“(品)>(品)》>>(品)…()>()">() 的唯一实数。 般地,我们规定:以任何正数a(a≠1)为底、以正无理数a为指数的幂2是夹在a以a的任何精确度的不足近似值和过剩近似值做指数的幂中间的唯一实数. 因为1的任何有理数指数的幂都是1。所以1以正无理数α,的任何精确度的不足近似值和过剩近似值做指数的幂总是1.我们规定: 1a=1(a>0) 象规定负整指数的幂的意义一样,我们规定: 正数w以负无理数一ax做指数的幂aa是幂aa的倒数.就是 1 2g, 这样,当底数a是正数的时候,它的无理数指数的幂 ①证明超出了本的范围,这里我们略去了。 ·2490 ==========第260页========== a,不管a是正数还是负数都有着确定的意义,它都表示某一个实数 可以验证①,这样规定以后:有理数指数的幂的运算法则对于任意实数指数的幂都能够适用例 已知10=4,109=3,求下列各式的值: (1)10+8,(2)10x-B,(3)10-3a,(4)102, [解] (1)10+8=100.109=4×3=12. (2)10-8=10=4.1号 10=33· (3)10x=(10)-2=4-3=1=1 4316 (4)10=(102=42=√4=2. 可题1.化简下列各式,然后根据已知条件求这些式子的值:61 ((+:).已я,-y0.75,ー d是-b益 2.比铰下列这些幂的大小 (1)0.1和0.1; (2)3和5; (3)产和a(a>0): (4)a立和u(u>0). [提示:在最后两题中要分01这三种情况帝究.] 3.已知10=2,10=3,把下面的数写成底数是10的幂的形式: 2 (1) (2)1.5;(3)G; (4)48;(5)/3: (6)42;(7)5:(8)0.125:(9)0.15;(10)√4.5. 解法举列)5-碧--16i.] ①验证较烦,这里我把它路去了。 ·2504 ==========第261页========== §62.对数 1。对数的意义 我们知道2的3次幂等子8,可以记作 23=8, (1) 这里2是底数,3是指数.8是2的3次幂.这种运算是乘方运算. 反过来,如果我们要问什么数的3次幂等于8,那末也就可以利用乘方运算的逆运算一开方,求得 38=2, (2) 这里8是被开方数,3是根指数.2是8的3次根. 现在,我们]把问题改变一下,问2的几次幂等于8?要解决这个问题就得从表达式 2z=8 (3) 中,找出使这个等式能够成立的指数心.这是一种新的运算,我们把它叫做对数运算,并把(3)式改写成 10g28=", 由(1)式,可知x=3,所以 10g28=3. (4) 在(④)式中,2仍旧做底数,8叫做真数,3叫做以2为底8的对数,“10g”①是对数符号 一般的,可以证明②:当a>0,a≠1,V>0的时候, 一定有一个唯一的实数b,满足等式 a--N. 在幂中的这个指数b,就叫做以%为底真数N的对数, og'是拉丁字logar.thm的缩写。 ②证明迢出本书范围,从略。 ●2510 ==========第262页========== 并把它记作 loga N=6, 这里a叫做底数,N叫做真数. 这就是说:如果不等于1的正数a的b次幂等于N,那末幂指数b就叫做真数N的对数 在上一节里,我们已经引进了无理数指数的幂,当a>0时,对于任何实数(正数、负数或者零)幂a都有意义,所以以不等于1的正数a为底的真数N的对数b可以是正数,可以是负数,也可以是零例1 证明对数具有以下的性质: (1)负数和零都没有对数; (2)1的对数等于0(10g。1=0,a>0,a≠1); (3)底的对数等于1(10gaa=1,a>0,a≠1). 一对数的性质 [证明] (1)根据幂的意义,当a>0且a≠1时,对于任意实数 五,总有a°=N>0.这就指出了真数必须是正数,也就是说负数和零没有对数. (2)‘.·a°=1,.∴.10ga1=0 (3).a2=a,∴.1ogaa=1. 例2 求以10为底,下面这些数的对数: 1000,100,10,1,0.1,0.01. [审题] 先求出10的几次幂等于这些数就可以了. [解] .°103=1000, ∴.10g101000=3. 102=100, ∴10g1o100=2. 101=.10; 10g1o10=1. .·100=1, .10g101=0. 01-品-0.山,1g01-1 02520 ==========第263页========== :10=0-0.0,.go0.01=-2.100 2.式子=N和1ogaN=b的互换 从对数的定义,对于a>0,a≠1,N>0,我们知道: a=N (1) 和 loga N=b, (2) 这两个式子所表示的,b和N三个数之间的关系是一样的,只是表达的形式,有所不同.所以我们可以根据需要,把写成指数形式的等式(①)和写成对数形式的等式(2)互相变换。 指数 对 数 真数… a的a:N ←→ loga N=6 …底欲… (a>0,a≠1;N>0) 把(2)的b代入(1),就得到 alogax=N (a0,at1,N>0). 这是一个很重要的恒等式。利用这个恒等式,可以把一个、正数写成幂的形式.例3 以√Z为底,子的对数是什么? [解] 设所求的对数是x,那末 1 10gz4に0 这个等式可以写成 (N2)= ·2530 ==========第264页========== 問ーでー(写)一,代人上 (/2)”=(√2)4 。.化=-4 答:所求的对数是一4。 例4 底是什么数的时候,625的对数是4. [解] 设所求的底是化,那末log625=4,就是 4=625 '.花=625=√√625=W/25=5 答:所求的底数是5。 〔说明) 因为底只能是正数,所以只要取正的方根. 例5 底是号的时候,一个数的对数是一3。求这个数。 [解] 设所求的数是,那末 10g1优=-3。 就是 (》- ∴.c=8. 答:这个数是8 例6 求下列各式的值: (1)28152; (2)218:5. [解] (1)281n:5=(2e)3. 。'2oei=5,(28)3=53 ∴.28185=53=125, 22-(), z-[()」ー[()*]==言 ·254· ==========第265页========== 匀题 1.把下列等式写成对数等式的形式: 62 (1)24=16;(2)23=8;(3)22=4; (4)21=2 (5)20=1;)2=:(の- 2.把下列等式写成指数等式的形式来进行检险: (1)1og1=0; ②1log青=-2 (3)1ogs32=2; (4)1o8:3=-1; 1og是- (6)1og魁27=3. B.用对数的形式来表示下列各式里的x: (1)10c=5; (2)3=2; (3)0.5x=2: (9W区=字 4.求下列各式的值: (1)21g:7: (2)31g; (4)4g,5. 5.求: (1)0g636; 1 (2)1og2 1 1 (3)wg (4)1og8江· 6.如果: (1)10g2x=-3; (2)0g:0x=-2; (3)0g3x=2; (4)10g5x=-1, 求 7.如果: (1)logz3=1; (2)loga16=4; 8o8房-子 (④g.V3=, 求瓶沁 §6·3关于对数的定理 1.积、商、幂的对数 在§6·1里引进了无理数指数幂以后,对于以任意实 ·255· ==========第266页========== 数&,B为指数的同底数幂有下列的运算法则: u.a=-a+: Qx=a-; (a)8=a3 幂的运算法则 在这基础上,我们可以导出计算积、商、幂的对数的法则. 法则1 两个正数的积的对数等子这两个正数的对数的和. 法则2 两个正数的商的对数,等于被除数的对数减去除数的对数 法则3 正数的幂的对数,等于幂的指数乘以底数的对数.用式子来表示,就是: 1.log MN-log:M+loga N (M>0,N>0). 2.1g.-1g-1gN(>0.W>-0. 3.log Ma -alog..M (Ar>0). 对数的运算法则 在法则3中,如果令=1(饥是大于1的整数),那末 23 就得到了方根的对数计算法则: logeM-1og.M"-1loga M (M0). m 这些法则可以这样来证明:根据§6·2里的恒等式,命 M-alogaM N=alogaN 那末:(1) MN-alug 1.alsa x=alog 1+10g aN 改写成对数等式的形式,就得 loga MN loga M+loga N, ●256· ==========第267页========== (2) Maloga M 7=agor=0a黑-1e,” 改写成对数等式的形式,就得 M loga=log:M-loga N. (3) Ma=(alga 1)a=aa logaH 改写成对数等式的形式,就得 logu Ma=a loga M 〔注意) 应用上面这些法则的时候,必须注意真数M,V是正 数这一条件.例如 10g1o[(-10)(-10)]=10g10100=2; ogo二l8-1g0l1-0-10 是有意义的,但是10g1o(一10)是没有意义的,所以下面这些等式都不成立: 10g1o[(-10)(-10)]=10g1o(-10)+10go(-10), 10g0- -10=1og10(-10)-1og1o(-10), 10g10(-10)2=210g10(-10). 例1 已知1og1o2=0.3010,10g1o3-0.4771,求: (1)10g1o6;(2)1og1o5;(3)1g1o√z [解] (1)1og1o6=10g1o(2×3)=10g102-+10g23 =0.3010+0.4771=0.7781. (2)g0g-10ga =1-0.3010=0.6990,(8)10g0V2-1g02-号1cg12 -7×0.3010=0.1505. ●267◆ ==========第268页========== 2取式子的对数 应用上面的法测,我们可以把几个正数的幂或算术根相乘除所得的式子,用式子里的各个数的对数来表示。这叫做取式子的对数.下面举例说明.例2 已知,y,名都是正数,且 8=vy 求1ogaS. [解] ton.8-oglog10g. =2 l0glog8 10g2. 例3 设二23,末 [审题] 因为等号右边这些幂的底数和根式的被开方数中有负数,不能直接取对数.所以要先把这个等式进行变换[解] x(-8)(-2)3.32 42 24 3.82 ∴.10ga=10ga24 =10gu3+1ngu8/z-10gu24 -51g.8+号1og.2-41og.2 -5 loga 3--loga 2. 例4 如果三角形ABC的二条边分别是a,b,c,那末它的 面积A可以用公式 A=√8(s-a)(s-b)(8-c) 来表示,这里s=之(a+6+).求1og04,[审题] 只要取式子√s(8-a)(s-)(一c)的对数,但是为了 ·258● ==========第269页========== 要应用定理,首先要证明,$,8一a,8一b,8一c都是正数.[解] 因为三角形中任意两边的和大于第三边,所以b+c-a>0,c+a-i>0,a+b-c>0.又 受(a나ώ+o)>0,a=受(あ+0a-(+ο-の>0。 同理,$>0,s-0.因此我们可以取式子 √s(s-a)(s-b)(8-c) 的对数,得 10g10A=10g1wVss-u)(s-b)(8-c) 1 =立1gn83〔8-a)(s-)(8-c) ·是ago8+1lgwg-) +10g1o(s-b)+1og1o(s-c)]. 〔注意) 1og1o(s一a)等是不能再化简的,把1ogio(s一a)写成等于10g1o8-一10g1oa是错误的. 习题1.己知110g02=0.3010,1og103=0.4771.求63 (1)log18; (2)1og1o0.125; (3)1og1o1.5; (4)log3); (5)l0g100.3; 1 (6)log10 (7)1og1o&5; (8)1og1o2000, 2.求下列各式以b为底的对数,式中a>0,b>0,c>0,且a>b. ①)x=: (2)G=21-26; (3)x=2-b2, (4)3x=e3-b; (5)=Vb /5a+56 (⑥)x=V3u-D .下列各式以10为底的对数: (1)C=2; (圆周长公式) ·2590 ==========第270页========== (2)A=rr3; (圆面积公式) (⑧)V-告 (球体积公式) (4)A=2rr1; (直圆柱侧面积公式) (5)V=xr3. (直圆柱:体积公式) 4.在上题中,分别用C,A,V,的对数来表示的对数,以10为 底。 5.利用恒等式N=agaw证明 loga NiN2Ns=log Nr+log,N2 +loga Na. 这里真数N1,N2,N都是正数. 3.从式子的对数求原式 把第256页的法则反过来,就是 logo M+l0ga N=loga MN,loga M-logo N=loga M/N, a loga M=logs Ma 利用这些关系,我们可以从某一个式子的对数求出这个式子,这种方法叫做从式子的对数求原式.举例如下: 例5 已1og.z-는(1og.(m-+)+1o.(m-m)],求.[解] g[og.(m)og.(m-n)] -是10g,(m+nm-) 、z10g(m2-2)-10g4√m2-. .x=√n2-n, 例6 已知 kgy-3ngwz-b)+710g6-号1gn(a+6),求, ·280· ==========第271页========== [解] gw8-31hgma-d)+号gw6-号iogm6+0 =10g1n(u-b)8+10g1c-10g1o(a+b)510g10(a-b)3N6 /(a+b)· y=(a-)V0 8 (a+b)・ 习题 1.求证: 63 (1)log1o(22+2+1)÷log1o(n-1)=1ogo(83-1): ②) (2)logo(a+Va2-1)=-loga(a-Va-) 「提示+√-丁=(+V-ICαー-。 x-Vx21 2.在下列各式中,求r(答案中不用分指数或负指数):(少log,g-5hg,+号iog-l图g2,3 .(2)lg0a=-lgan(a+8)-星(lcg0c-2gn. 3.在下列各式中求如: (1)logx=51gm±是 "2 ×[log(m+)+号1g(m-)-lgn-lcgn小} (2)lgx=- ×[lgb-是lg4+号1g(a-b)-÷1g(a+b)小、 这里1ogx表示以任意不等于1的正数为底的对数。 §6·4常用对数 利用积、商、幂的对数的性质,可以分别把乘法、除法、乘方和开方的运算改成加法、减法、乘法和除法的运算.所以利用对数来进行数值计算,常可使计算的手续得到简化,这是对数的-一种重要应用。 ◆261● ==========第272页========== 我们常用的记数制是十进位记数制.应用10做底数的对数来进行数值计算也最为方便.这种以10为底的对数叫做常用对数.为了书写的方便,我们约定把1og1oN的底10略去不写,并用专门的记号“g”代替“10g10”,例如: 10g10100-2简写成1g100=2. 以后,如果没有说明对数的底,就认为所说的对数是常用对数.例如,说“2的对数是0.3010”,就是指2的常用对数是0.3010,也就是1g2=0.3010. 常用对数除去具有§6·2例1中所指出的一般对数都具有的那些性质以外,还具有一些特殊的性质.下面我们提出其中最重要的儿个, 1.10的整数次幂的对数 在§6·2例2里,我们曾经计算过10的一些整数次幂以10为底的对数,得到下表的结果: N 1000=103 100=10210=101 1=100 0.1=10-110.01=10-2 lg N 3 2 1 0 -1 -2 从这里可以看出,10的整数次幂的对数,恰巧等于幂的次数 一般地,我们有 g10”=n1g10=外(n为整数). 利用这个性质,在写出10的整数次幂的对数时我们有下面的法则: (1)L后边带有若干个零的整数的对数是一个正整 数,它等于真数中零的个数 (2)1前边带有若干个零的纯小数的对数是一个负整数,它的绝对值等于真数中零的个数(包括小数点前面表示整数的一个零). 0262● ==========第273页========== 例如 g100000=5, 1g0.00001=-5 从上面列出的表中,我们可以看到一个重要的事实,即:真数愈大,它的常用对数也愈大注 在下一弦里,我们将要证明对于任何大于1的数4为底的对数,都有这样的柱质 2.从1到10的数的对数 设飞是一个比1大而此10小的数,就是 14914 (3)同样,只要查出300的对数尾数,就可以知道3的对数尾数是0.4771, N 0 … 30 →4771 2。真数有4个有效数字 例如,要求真数3126的对数尾数,可以先求出真数左 ●.288· ==========第279页========== 边3个数字312的对数蓖赉详042?,然后向右横着看到 修正值里第一横行标有“6”的那一直行相交叉的地方,找到数“8”,这表示修正值是0.0008 N 0 2 …6… …↓ …… 31 4942 …8… 把这个修正值加到312的对数尾数上,得到 0.4942+0.0008=0.4950, 这就是3126的对数尾数. 3。真数有4个以上有效数字 例如要求真数31257或者31264的对数尾数.在<四位数学用表》的对数尾数表里,我们找不到这样的数,这时可先用四舍五入法把它们截成只具有四位有效数字的数31260,再从表中查出3126的对数尾数,就可以看做是31257或者31264的对数尾数的近似值.如果需要求出较高精确度的近似值,那末要用五位或者五位以上的对数尾数表. 例 求下面这些数的对数尾数: 83.06;346000;0.033;3.4835, [审题] 这些数的对数尾数分别和 3306;346;33;34835 的对数尾数一样,所以可仿照上面所说的方法来找。[解] 33.06的对数尾数是0.5185+0.0008=0.5193;346000的对数尾数是0.5391; 0.033的对数尾数是0.5185; 3.4835四舍五入截成3.484,·它的对数尾数是 0.5416+0.0011=0.5427. ·269● ==========第280页========== 注 在实际解题时,只要直接写出对数就行,这时如果要用到修正值,可以把表中的尾数先看成是整数,用心算求出结果,再记上小数点 习题填写下面的表:65 声数小 2 74.6848.30.14 0.1687 135455.55564.713 入数 打数 [提j::式0.1687的对数数时,修正值要用和表中0.2253同-一横 里J数0.0018.] §6·6常用对数的求法 1。求一个数的常用对数 从§6·4和§6.5知道,求一个数N的常用对数,可以 分成两个步骤:第一步是先通过观察来写出它的首数;第二步是利用对数表找出它的尾数。 例1 求下面这些数的对数: 280.9:33.46;0.8317;0.0548 [解] 1g280.9=2.4486; 1g33.46=1.5245; 1g0.8317=1.9200; 1g0.0548=2.7388 〔说明) 同时求几个数的对数的时候,不妨先写下它们的首数,然后再一起查表.在查表时,也可根据真数的第一个有效数字的大小顺序来进行,以免忙乱.例如在解这个题目时,不妨先写下首数2,1,工,2,然后再按照2809,3346,548,8317的顺序来查表求尾数.注 因为要表示1g0.8317中I.9200是精确到0.0001的近似值,所 以小数部分最后两个“O”不要别去, ·70· ==========第281页========== 习题 求下面这些数的对数白更口 66 1.(1)5; (2); (3)13; ) (4)78; (5)125; (6)240: (7749; (8)989 2,(1)0.7; (2)0.015; (3)0.497; (4)0.0567; (5)4.09; (6)1.02; (7)0.0084; (8)0.000809 3.(])543.6; (2)7132; (3)15.08; (4)2.197; (5)0.04536; (6)0.9032; (7)167400; (8)638400 4.(1)24868: (2)1.7384; (3)32.1435: (4)286.74; (5)0.06835; (6)3.1416; (7)1.4142; (8)152.95 5.(1)1.34×105; (2)2.035×10; (3)7.×10-3:(4)4.1356×10-4 2求积、商、幂的对数 我们来看下面的例子: 例2 求125×2.87的对数. [审题] 因为1gMN=gM41gN,所以这个积的对数,就是1g125与1g2.87的对数的和.[解] 设0=:125×2.87,那末 1gx=1g(125×2.87)=1g125+1g2.87. 1g125=2.0969 1g2.87=0.4579(+ 1g:2.5548 答:所求的对数是2.5548, 例8 求12.5÷2.87的对数. [审题] M 因为g为-gM-gV,所以这个商的对数就是1g12.6利和1g2.87的差.[解] 设cx12.5÷2.87,那末 0271● ==========第282页========== 1255 1gx=1g2.87=1g12.5-lg2.87. 1g12.5=1.0969 1g2.87=0.4579(- 1gx=0.6390 答:所求的对数是0.6390. 例4 求(12.5)3和/12.5的对数. [审题] 因为1gMa=x1gM,所以这里所求的对数可以从 g2.5的对数乘以3及号求得、 [解] (1)设x=(12.5)3,那末 1gx=1g(12.5)3=31g12.5=1.0969×3=3.2907. (2)设y=/12.5,那末 1gy=1g32.5=g(12.-景g12.53×1.0969=0.3656, 答:(12.5)3的对数是3.2907; /12.5的对数是0.3656, 注 尾数部分只要求出四位小数的近似值 习题 求下列各题中这些积、商、幂及算术根的对数: 66 1.(1)24.5×1.57; (2)1.046×2.86; (3)4.134×15.48, (4)12.78×3.392 2.(1)41.34÷15.48: (2)10.46÷2.86; (3)12.78÷3.392 (4)24.5÷15.7, 3.(1)8.54, (2)2.0363; (3)/15.36; (4)V3.1416, 3.首数是负数的对数的运算 从上面的例子看到,求积、商、幂的对数,可归结为对数的加法、减法,或者对数与一个数的乘(或者除)法.当对数 ●272◆ ==========第283页========== 的首数是正数或零的时候这些运算是通常的运算,但是我们常会遇到对数的首数是负数的情形,所以我们还要学会首数是负数的对数的运算.举例如下: (1)对数的加法:例如,我们要求对数2.0969与 1.4579的和,这就是要求一2+0.0969与一1+0.4579的和.这时,我们应把加数的首数和尾数分别相加,再把结果合并起来,列成直式,就是 2.0969 +1.4579 3.5548 又如,我们要求对数2.0969,1.4579与1.6789的和,仍和上面一样,先把各个加数中首数和尾数分别相加,再把结果合并起来,但是这里各个尾数十分位上的和超过了10,要进位到首数上去,列成直式,就是 2.0969 1.4579 +1.6789 .2337(注意,2+1+1+1=1). (2)对数的减法:例如,我们要求对数2.9969减去对数1.4579的差,这就是要求 (-2+0.9969)-(一1+0.4579) 的差,根据有理数减法的性质,我们可以把首数、尾数分别做减法,再把结果合并起来,就是求 [(-2)-(-1)]+(0.9969-0.4579), 于是得出结果是-1+0.5390就是1.5390.列成直式,就是 2.9969 -1.4579 1.5390(注意,2-1=1). 又如,要求对数2.0969减去对数3.4579的差,仍照上面方法来做,但是因为这里被减数的尾数比减数的尾数小, ·273 ==========第284页========== 先要从首数退下上来再减,列成直式,就是 2.0969 -3.4579 0.6390(注意,2一1-3=0) (3)对数的乘法:例如,我们要求对数1.4579乘以3的积,这就是要求 (-1+0.4579)×3. 应用乘法对加法的分配律,得到 1.4579×3=(-1+0.4579)×3 =(-1×)÷(0.4579×3)=-3+1.3737 =(-3+1)+0.3737 =-2+0.3737=2.3737, 为了简便,我们仍把它写成直式,就是 I.1579 3 2.3737 〔注意) 尾数十分位上的数乘以3超过10要进位到首数上去, I×3+1=3+1=2. 又如,我们要求对数3.4579乘以24的积,可以这样来算。 3.4579×24=(-3+0.4579)×24 =-72+10.9896 =62.9869 (④)对数的除法:例如我们要求3.1457÷3,应用除法的运算性质,得到 3.1457÷3=(-3+0.1457)÷3 =(-3÷3)+(0.1457÷3)=-1+0.0486=1.0486。 列成直式,就是 ·274· ==========第285页========== 料东7 .0486 但是,如果遇到被除数的首数不能被除数整除的时候,那就要先把被除数化成能被除数整除的一个负数与一个正数的和的形式,然后再做.例 3.1457÷8=(-8+5.1457)÷8 =-1+0.6432=1.6432. 我们也可用直式来做,写成 83.1457 1.6432 〔注意〕 T×8=8,3-8=5,把5并到尾数部分再除. 习“题计算: 66 1.(1)1.5436+2.3892; (2)1.1546+2.9835: (3)0.7658-1.3s56; (4)I.2396-0.7459: (5)0.1548-I.2836; (6)I.1631-1.9448; (7)2.1271-3.2848; (8)1-I.2812, 2.(1)I.2432×3; (2)I.9272×2, (3)2.2817÷2; (4)T.1536÷3; (5)1.7235÷4; (6)2.6483÷3; (02152×号: (8)I.5G8×3 例5 求下列这些式子的对数: (1)e=0.1357×0.086×14.58; (2)y=0.1357 14.58 (3)2=(0.086)8, (4)t=√0.1357 [解] (1)取对数: 1gx=1g0.1357+1g0.086+1g14.58. 列出直式: 1g0.1357=1 1g0.086=2 1g14.58=1.(+ g必七 ●276e ==========第286页========== 查表,把各个对数尾数填上去,然后再做加法: 1g0.1357=1.1326 1g0.086=2.9345 1g14.58=1.1638(+ 1g心=1.2309 (2)取对数:gy=1g0.1357-1g14.58.列式,计算: 1g0.1357=1.1326 1g14.58=1.1638(- .gy=3.9688 (3)1g2=3×1g0.086 1g0.086=2.9345 3(× .1g2=4.8085 1 (4g=克1g0.1357. 21g0.1357=1.1326 ∴1gt=1.5663 注 在熟练以后,第一步取对数的横式可以不写出来,直接写成直式来计算.解题时最好象(①)那样先列出直式,把对数的首数写好,再一起查表,填上尾数,然后进行计算.这样有计刘地进行,不但可以省事,而且也不易发生错误。 例6 求下式的对数: =0.1357×√0.086 (14.58)8 [解] 第一步,先取对数,得到 0.1357×/0.086 1gx=1g (14.58)8 =(1g0.1357+1g0.086)-1g(14.58)8 -(g0.1357+号g0.06))-8g14.56, ·276· ==========第287页========== 第二步,列出直式星楼校小1g0.1357=1. 2|1g0.086=2. 21g0.086= 之g0.086= 1g14.58=1 31g14.58=(- (× g= 31g14.58= (这里虚线右边是中间步骤的计算) 第三步,查对数表,求出尾数,填到写出首数的各行中去,得到下面的式子:1g0.1357=1.1326 21g0.086=2.9345 号1g0.0e8= 立1g0.086= 1g14.58=1.1638 31g14.58= 3(× lga= 31g14.58= 第四步,做中间步骤的计算,把结果填到左边的相应地方,最后算出如下式表示的结果:]g0.13571.1326 21g0.086=2.9345 1 1g0.086=1.4673 (+ 之1g0.086=1.4673 2.5999 1g14.58-1.16 81g14.58=3.4914(- 3(× .gc=5.1085 31g14.58=3.4914答:所求的对数是5.1085 注 实际做这个题目的时候,当然只需应用第四步最后的那种式子 习题 求下列这些式子的对数: 66 1.(1)0.046×2.86; (2)4.134÷0.1548; (4) (3)8/0.1536; (4)(0.273)5 103.8×20.97 2.(1) 9.738×21.09 5.174×13.62 (2)48.72×0.8478 (3)0.156×835.74; (4)30.15322 2.74 •277● ==========第288页========== 草流: §6·7反对数表 要利用对数来进行数值计算,除掉要学会常用对数的求法之外,反过来还必须学会从常用对数求真数的方法, 知道了一个数的常用对数求这个数,在一般情况下也要分成两个步骤来做:第一步是从对数的尾数求出真数的 四位有效数字;第二步是根据对数首数的值来确定真数中小数点的位置 在:第-步,我们可以用反对数表.下面是四位数学用表》里反对数表的一个部分. m 0 2 3 5 6 8 9 23456789 ,352239224422492254225922652270227522802286 23345 ,36229122962301230721223172323232823332339 e23344 .372344235023552360286623712377238223882393 3445 ,332399240424102415242124272432243824432449 445 .3924552460246624722477248324892495250025061 这个表的构造和对数尾数表的构造正相反.标有m的直行和横行是对数的尾数,其余是真数的有效数字,最后一栏是修正值. 瑰在我们来说明,怎样应用反对数表从已知一个数的对数来求这个数例1 已知1gw=1.3860,求花. 〔解] 这里尾数是0.3860.在反对数表标有m的直行里找到“38,横行里找到“6”,从38横着看,从6直着看,交叉 甲 的地方是2432.这就是真数中的有效数字 乙 性 6 … …… .33 2432 。278。 ==========第289页========== 因为对数的首数是1,所以真数有两位整数,小数点应该点在4和3的中间,得 x=24.32 例2 已1g=I.3624,求. [解] 这里尾数是0.3621,和例1一样先根据尾数0.3620,我出对w的真数的四个有效数学“2301'”. 3 .36 2301 然后再在:这一横行里横着看到修正值栏第一横行标有4的那一格,写着2:把这个2加在2301上,就得到2303 因为对数的首数是工,真数是整数位上有一个零的纯小数,所以小数点要点在2的前面,得 x=0.2303 同样,我们可以求出§6.6例2到例6各题算出的这些对数的真数: (1)1gc=2.5548, c=358.8; (2)1gx=0.6390, x=4.355; (3)1gx=3.2907,花=1953 (4)1gy=0.3656, y=2.3205 (5)1gx=I.2309, x=0.1701; (6)1gy=3.9688, y=0.009307; (7)g≈=4.8035, %=0.0006360, (8)1gt=1.5663, t=0.3684: (9)1gx=5.1085, x=0.00001283, 注 应用四位数学用表中的反对数表时,已知的对数尾数如果是有4位小数的近似数,那未求出的真数也可以有4个有效数字,所以在(4)中,我们把y写成y=2.320. ·270 ==========第290页========== . 在写出真数的能分脚果对数首数是正数或零,可以先写出真数的四个有效数字,然后再按照首数的值来确定小数点的位置;如果首数是负数,那末可以先写出第一个不是零的数字前应有的零,并且在第一个零的后面点上小数点,再接着写真数的其他四个有效数字, 习题 1.设g公等于下列各数,求, 6.7 (1).7482; (2)0.4362; (3)I.6415; (4)2.6149; (5)3.2648; (6)2.8176; (7)3.9340; (8)4.7267; (9)0.9236; (10)T.1015; (11)2.6995; (12)1.5419 2.在下列各式中求x: (1)1gx=-0.8342; (2)1gx=-1.2345; (3)gx=-2.1486; (4)1gx=-0.0732. [提示:先把负数写成负数和正的纯小数的和.] §68利用对数进行计算 有了上面的基础,现在我们就可以利用对数来做数值计算.下面举一些例子.例1 利用对数计算: 341×0.06794×8.7 0.9832×4780 [解] 设 A=341×0.06794×8.7 0.9832×4780 那末 1gA=(1g311+1g0.06794+1g8.7) -(1g0.9832+1g4780). 1g341=2.5328 1g0.9832=1.9927 1g0.06794=2.8322 1g4780=3.6794(+ 1g8.7=0.9395(+ 3.6721 2.3045 3.6721(- 1gA=2.6324 、。A=0.04289 ◆280● ==========第291页========== 注 如果把8,7看成是只有满蜘数字的近似数,那未最后算出 的结果,也只需写出两个有效数字,就是A≈0.043 例2 已知三角形的面积可以用公式 △=Ws(8-a)(s-b)(s-c) 求出,这里a,b,0是三角形三条边的长,8=+b+c2 如果a≈15.37厘米,b≈21.42厘米,c≈13.83厘米, 求三角形ABO的面积. [解] a=15.37 8-a=9.94 b=21.42 5-b=3.89 0=13.83(+ 8-c=11.48 28=50.62 s÷25.31. g△=壹[g8+lg8-a)+lg6s-)+lg(s-c] 1g8=1.4033 1g(s-a)=0.9974 1g(8-b)=0.5899 1g&-c)=1.0599(+ 24.0505 1g△=2.0253 .'.△=106.0≈106(平方厘米). 注 这里s一,s一b都只有3个有效数字,按照近似计算的法则,最后求到的△也只需保留3个数字, 在上面的例子里,我们所遇到的数都是正数,现在我们再来看如果已知数据中有负数,应该怎样来计算.例3 利用对数计算: (-2.31)×5/72 [审题] 这里一2.31是负数,因为负数没有对数,所以不能直接用对数计算.我们需要分两步来做,第一步是求这个式子的绝对值,第二步再确定它的符号。 ◆281· ==========第292页========== [解] 设A=(-2.31)3×/72=-2.313×3/72, 那未 1A|=2.313×5/72 g4=8g2.81+后1g72. 81g2.31=3×0.3636=1.0908 51g72=方×1.8573=0.8715(十1g 4 =1.4623 A=28.99, ∴.A==28.99, 注 上面的直式,也可以按照前面讲过的方法来列,但是象这类题目,做乘法、除法都以利用心算,所以列式可以简单一些. 例4 利用对数计算: 43.05-54.752 [审题] 这个式子里,有减法运算,不适宜用对数来计算,我们先利用平方差公式,把它分解成两个因式的积.[解] 设 A=43.06-54.758 =(43.05-54.75)(43.05-+54.75)=-11、70×97.80 1gA|=1g11.70+1g97.801g11.70=1.0682 1g97.80=1.9903(+ 1gA=3.0585 A=1144, .'.A=-1144. 习 利用对数进行计算(题中各数都是近似数): 68 154.8×5.436 1.(1) 12.72 (2) 54.83×1.367 2.832 (3)103.8×20.97 5.174×13.62 (4) 9.738×21.0948.72×0.8478 2.(1) 92.172×5.143 (2) 1.8944×23.409 2.184×0.5836244.15×0.96473 ◆22◆ ==========第293页========== 81508×/14.36 :.1%④12.483×/5.760 (3)24.38×V8.734 1.842×/673.8· 3.(1)(-5.32)3×0.0294: (2) シー0.536 3.89×0.924 4.(1)求半径?≈15.G2厘米的圆的面积 (2)设圆的面积A≈0.5676平方米,求这个圆的周长; (3)直楼柱的体积T可以用公式 V=hS 来计算,这里S是底面的面积,是直楼柱的高.如果这 个直校柱的底是正方形,边长a≈0.567米,直棱柱的高h≈4.09米,求直梭柱的体积; (4)-个圆环的内直径d≈5.27米,外直径D≈7.30米.求这 圆环的面积; (5)三角形的三条边的长是a≈8.975厘米,b≈7.863厘米,c≈6.456厘米,求它的面积. §69对数的换底公式 上面我]讲了常用对数的计算,但有时我们也需要求其它不等子1的正数为底的对数.例如,我们来解这样的问题: 镭每年蜕变0.044%,究竟要经过几年,1克镭蜕变后 彩下的铺是是克: 设经过:年后所利的锯是是克,那末 99956 .1 =10g999ǎ67 1 100000, 0U003 这里,我们就需要用到以99956 100000为底的对数, 这个问题可以利用常用对数来解,但要把上面这个对数化做以10为底的对数。 ●2●g● ==========第294页========== 80r0 1.对数的换底公式 现在我们先来研究怎样把一个已知底是不等于1的正数b的对数 10gV(6>0,b≠1,N>0) (1) 变换成与它恒等的以另一个不等于1的正数(为底的对数. 象证明对数的运算法则一样,首先我们把V写成幂的 形式,得 N=6ogx(b>0,b≠1,N>0) (2) 在等式两边同时取以a(a>0,a≠1)为底的对数,得 l0go N=l0ga 61oso N 应用对数的运算法则,得 10gaN=1ogbV·loga b. (3) 。”b≠1,10gab≠0,在(3)式中两边同除以10gab,即得以b为底的对数变换成以a为底的对数的公式: log N.logaN loga b 对数换底公式 在这个公式中,如果把%换成10,就可得到以任何不等于1的正数b为底的对数变换成常用对数的公式: log.N-Ig N Ig6 (b>0,b≠1,W>0). 利用这个公式,就可以求出 1g0.5 1 元=10g999562一 1U0000 1g0.99956 从五位对数表上查出 1g0.5=1.69897=-0.30103,1g0.99956=1.99981=-0.00019. 0284· ==========第295页========== 由此得6=-0.30103 -0.00019≈1.6×108 这就是说,大约要经过1600年镭才蜕变掉…半 注 (1)因为0.99956和1很接近,利用四位对数表不能求出它的比较精确的对数,所以要利用有较高精确度的对数表 (2)在:求?的时候,要做除法,这时应该把I.69897和I.9998L 都写成普通的负数,才能计算. (3)因为原始数据里0.04%只有两个有效数字,所以最后求出的结果,我们只需保留2个数字 在科学技术中以及在高等数学中主要使用以无理数 6=2.71828… 为底的对数.这种以e为底的对数叫做自然对数,1og。N 通常记作1N.根据对数换底公式,可以得到自然对数和 常用对数之间的关系是 In N=IgN ige 因为1g8=1g2.718…≈0.4343, 1 1g日0.454g≈2.303, 所以在把自然对数化成常用对数时,经常使用下面这个近 似公式 1nN≈2.303gN. 例1 求(1)1ogo.5√2.(2)1n√2 1 [解] ()1ogo.√2=1gW② g2 1g互 g1-1g2 1 ×0.3010 0.1505 0-0.3010=0.3010- -0.5。 ·285· ==========第296页========== 1 (2) nV②-gw22g2 Ig6 Ige 立×0.3010×2.803≈0.3466.1 2.对数换底公式的应用 对数的换底公式,除去可以把一般的不等于1的正数为底的对数化成常用对数后,利用对数表进行计算以外,还可以利用它来作关于对数式的各种恒等变换,下面举例说明: 例2 求证 1ogub.10ga=1(a>0,a+1,b>0,b≠1). [审题] 把1oga化成以《为底的对数,再进行运算。 [证明] Jogaa :1o4gea=10ga6-10ga6' .∴.10gb…10ga=1. 例3 已知: 10g1s5=4,18=9, 求证 10g436=2-b a+6 [审题] 由18°=9可推得10g1s9=b,因此只需把10g4:36改用18为底的对数来表示,再进行化简. [证明] .·18=9,∴,10g1s9=b. 。六hgs2-1ogs5-1g6l8-1g9-1-d. 由此即得 10g4636=10g86-10g1818+10g182 10g1R4510g189+1og186=1+(1-)=2-b a+bg十b。 (证毕) ·年2%● ==========第297页========== 可题 1.已知:g2=0.3010,1g3=0.4771, 69 求:(1)1og23(2)og20.3. 2.求下列自然对数(不查对数表): (1)1m6;(2)1n0.15 3.在:下列各式中,求c具有4个有效数字的近似值: (1)l1g16oc=-0.9508; (②)1Dg0.1x=2.670; (3)10gc=-0.&41; (4)l0g6x=-0.8764. 4.求证: a)ee.yc双 log。M loge M氵 (2)leg N-leN; (3)gob.logoc-logod.loge a=:1; ④log方=三g.b. 本章提要 1.指数和对数 cb=N luga N=b 式 子 (a0,a≠1) :a0,a≠1,N>U) 《一冪的底数 &-对数的底数 名 称 b幂的指数 N…真数 N一幂 b-以a为底、N的对故 (1)ab>0 (1)负数和零没有对数 性 质 (2)a0=1 (2)1oga1=0 (3)ad2=a (3)1oga=1 a.9=C+5 loga MN loga M+loga N (M>0,N>0) a as,=aa-8 10g.只=1cgaM-1ogaN 运算注则 (M>0,N>0) (c")8=aa6 oga Ma-=a loga M (M>0) 基本恒等式 N=aiuge N(a>0,*1,N>0) 027● ==========第298页========== 2.常用对载的性贯 (1)1g10=n(n是整数). (2)00. (1)第一步求首数. (i)N≥1.设V中整数数位的个数是p,则 n=p-1. (i)N<1.设N中第一个不是零的数字前零(包括整数数位的一个零)的个数是g,则n=一9. (2)第二步,从对数表中查出尾数. 4.已知一个数的常用对数求这个数 (1)第一步,根据尾数,从反对数表中求出真数的有效数字 (2)第二步,根据首数,确定小数点的位置。n≥0,真数有n+1位整数; n<0,真数第一个不是零的数字前有肌个零(包括整数数位的…个零). 5.对数换底公式 l0g,N-logN loga b (a>0,a≠1;b>0,b≠L;N>0). 特例: l0g,N-Ig N 1g6 复习题六A 1.是不是对于必的一切实数值,等式 ●288· ==========第299页========== =缴拼治续.0,“+1) 都能成立?为什么? 2.下列各式是不是恒等的?(式中心,y都是正数) (1)log(x+y)和logac+1og。y;(2)log.x2和(1ogax)2; (8)0g和og号:logay (4)log。和log。/正. 12 [提示:如果要说明它们不恒等,可给x,y以任意数值,再来比 较.如,要说明(1)中两个式子不恒等,可令 =子y= 了是 g(合+)-g1-0, 1=log2-1=-10g22=-1, 10g2 .lcg(侵+)*log:3+lg受 由此即可说明log(x+y)和log。c+log.y是不桓等的] 3.下面这些式子是不是恒等?如果不恒等,那末未在什么条件下才 相等? (1)l0g.2和210ga; (2)og(x2-y2)和oga(x+y)+l1og(x-2y). 4.(1)如果两个数的常用对数具有相同首数,这两个数间有什么 关系? (②)如果两个数的常用对数具有相同尾数,这两个数间具有什么关系? 5,取什么值的时候,下列不等式才是正确的: (1)1gx>1; (2)1g(-x)>1; (3)1gx2>1; (4)(lgx)2>1; ()1g<21gx; (6)gx>2lg. 6.已!1g2和1g3,不查对数表,可以算出从1到100之间哪些整数的对数? 7.下面这个推理,错哪里? (1) 等式左边增大一倍,右边不变得 ●280● ==========第300页========== (2) 由此得 g(得)s子 (3) (4) 8.三角形内切的半径可以用公式 (s)(s-b)-) 来计,式,6,0是三角形边的说,8=10求g 9以任意不等于1的正数做底,取下面这些式子的对数 (1)x/ (,b,c,d都是正数); b"v d (2)=s(8-)(-b(g-) Aubc [,么,0为三角形的三边,9=子a+6+o)小 (3)x=C-6)-b)2 マーc (a>0,b>0,c>0); ④-告aV7m≥,0, [提示:在对数符号“og”下角,不注明底数是什么,就表示可 以用任何一个不等于1的正数为底.] 10.从下列各式求(2,m为大于1的正数):(④bgr=hga+nlgu+b)-是lga-03 (2)bgr=lngb+mlbg0-0)-是go+e: (3)log t==-log(a+b) 2ga+g0-(bg-kgb) (4)og=[ga+号1oga+b)-2loga-b)-号log0. 11.求证: (④)bga3+3a2+3a+1)-1g(a-w2+3a-l1)=3log8t是 ·20● ==========第301页========== (②log(y+T+=-1(at0).log(Va2+I-a) 12,球的体积V的公式是V=言匹,这里R是球的半径. (1)按照下列或据,求球的体积: ()K≈12.46厘米; (ii)R≈5.45厘米. (2)知果要使球的本积是200立方厘米,那未球的半径应该是 多少?(200看做精确数) 13.圆筒的体积由公式 V=풀(D2-)H 来算,这里D是圆筒底的外直径,是内直径,丑是圆筒的高, 现在:有一根铅管,量得外直径D≈27.2厘米,内直径d≈25.0厘米,长1≈4.375厘米.知铅的比重是11.3克/厘米3.求这 根铅管的重量V. [提示:把船管看成是圆简,那末它的长1就可以看是圆筒的高 H,义它的重量等丁比重乘体积.] 14.的体积!严的公式是 V=nR2H, 这里兄是圆柱底面的半径,1是它的离.设铜丝的比重是8.55克/厘米3,求直径是2毫米,长1000米的铜丝的重量(直径和长都把它看成是精确数) 15.一捆直径是2.50亳米的钢丝重15.8公斤,如果钢丝的比重是 7.96克/厘米3,求这捆钢丝的长. 16.已知g27=a,求证:kog616=4(3-a)3+4 复习题六B 1.(1)1果,y都是实数,并且 (2x-1)2-+(y-8)2=0, 求log&的值; (2)求g(√/3-V+√3+V/5)的值. 2. ()已知1g-의,。날의(a>0,a+1),求2 ·291d ==========第302页========== (2)已知方程x气-4)x+4=0有等根,求10g2m。 3.(1)已知:2x+5y=20,求lgx+1gy的最大值; (②)已知:g+gy=2,求子+号的最小值, 4.已知不等边三角形的三条边分别为,b,c,求证: k+g은+g>lga+eb+lgo 5.已知log23=a,log37=b,试用a、b的代数式表示log4g56. 6.求证: ,2,3 1og19+1og99+log9<2, 7.已知:两个正数,y满足条件: x+y=7V以, 求证:log子(V云+Vy)-子1g+10ga80(a>0,a+1). 8,(1)已知正数V的常用对数是一个有理数,怎样把N表示成 幂的形式? (2)证明1g35不是一个有理数. 第六章测验题 1计算 ④log,881V729x9, (2)(1og43+1og83)(1og32-+1og92). 2.已知lg2=0.3010,lg3=0.4771, (1)求2?×811×510是几位数? (2)求(0.0144)在小数点后面第一个非安数字前有几个零?()求g号V.西的值 3.已知方程Qc2-2bx十=0(4,b,c均为不等于1的正数)有等 1+12根,求证:1og8十1ogog如 4.已知:m=a,2=a,mmx=4,求证:y=1. 5.铜丝长28.34米,它的直径等于1.40毫米.设铜的比重是8.85克/立方厘米,求这条铜丝的重量。 ◆292◆ ==========第303页========== 指数函数和对数函数 第五章里我们研究的幂函数,其中自变量是作为某一个幂的底数出现的.但是我们经常还会遇到另一类的函数,在这类函数里,自变量作为某一个幂的指数出现.例如函数y=10:,其中自变量心就是幂10的指数.这类函数叫做指数函数 指数函数以及它的反函数一对数函数,是函数中重要的两类.这一章我们将研究这两类函数的性质和图象。 在这一章里,我们还将联系这两种函数的研究,学习一些简单的指数、对数方程,以及指数、对数不等式的解法, §7·1指数函数 1.指数函数的意义 我们来研究下面的问题: 已经知道镭在一年内有0.044%蜕变为其它元素,1克镭,经过1年,2年,…,还剩下多少?经过心年呢? 这个问题,可以这样来考虑:经过1年,剩下的镭是 1×(1-0.044%)=99956 100000(克) 经过2年,剩下的镭是 1×(1-0.044%)=99956 100000 (克)。 ·298· ==========第304页========== 一般,如果经建年,剩下的镭的克数是y,那末有 99956\ y-(00000)· 从这个式子可以道,当变量龙在它可取值的范围(≥0)里取每一个确定的值时,变量y都有一个确定的值和它对应,所以这个等式确定了变量y和间的一种函数关系.象这样的以不等于1的正数为底、自变量为指数的幂给出的函数叫做指数函数. 指数函数一般地可表示为 y=a 的形式,这里底数是大于0且不等于1的常数. 因为a>0且a≠1时,对于任意实数龙,幂都有意义,所以函数y=a(a>0,a≠1)的定义域是实数集R. 例如函数y=2兴,y=10,y=(侵)等等都是的指数函数, 注 因为当底数%=1时,1总等于1.这就是说函数y=1“中不论x取什么实数值,对应的函数值总是1.这样的函数实际上是一种常数函数,没有专门研究的必要。所以,我们在研究指数函数的时侯,规定底数a>0,且x≠1, 2.指数函数的图象 我们在同一坐标系里作下面这三个指数函数的图象, y=-2,y=():, y-10对于前面两个函数,我们可给:以下列的整数值: -3,一2,-1,0,1,2,3, 来计算y的对应值.对于第三个函数,如果仍旧给x以上面的各个值,那末所得的y的对应值就很大,画图不方便,因此,我们给以下列的一些数值: ◆2040 ==========第305页========== 子0是 来计算y的对应值. 把计算得到的对应值列成下面的三个表: y=28 -.3 -2 0 2 3 18 1 4 8 y-(》 -3 -2 -1 0 2 ● 6 2 1 1 y=10 1 3 1 4 2 4 12 34 0.1 0.2 0.3 0.6 1.8 3.2 5.6 10 第三表里y的某些值是精确到0.1的近似值,例如,在心= y10 号的时侯, y=10-1103 1010 Y元√3.16 10 10 1.78 ≈i0≈0.2. 在同一坐标系里,用每一个 图71 ·2950 ==========第306页========== 表里的各对值作为点,并且分别用一条平滑的曲线把它们依次连接起来,就得出图7·1中的三个图象 从这三个图象可以看出它们的一些特点: (1)这三个图象都在x轴的上方.这就是说不论取怎样的值,函数的值总大于零 (2)这个图象都通过点(0,1).这就是说当x=0的 时候,函数的值总是L (3)随着底数a的大于1和小于1,函数y=a的图象有不同的特点,就是: 当a>1(例如a=2或者a=10)时,图象是从左到右逐渐上升的.在y轴的左方,图象位于x轴和直线y=1之间;在y轴的右方,图象则在直线y=1的上方。这就是说,当a>1时,函数y=a2是增函数,并且 -q*<1,如果c<0, >1,如果>0 当01,如果x<0, <1,如果>0. 题 1.在坐标纸上以5毫米作为坐标轴的长度单位作出函数y=2 7.1 的图象,然后读出图象上点的坐标,填写下面的表(精确到 (1) 0.1): 1 1 3 2 2 2 ·296● ==========第307页========== 2.(1)填写下面的表: -2 -1 y=3 3 (2)利用这个表格里的数值,作出函数y=3和y=3)的图象; (③)从图象里看出这两个函数,有哪些相同的:和不同的性质? 3.利用第1题中已作出的图象,怎样用简便的方法作出函数 =(侵》和= 的图象?作出这两个函数的图象. 3.指数函数的性质 在上面,我们从指数函数y=2二,y-()”,=10的图象发现了这些指数函数的一些重要性质。事实上,一般的指数函数y=a也都有这些性质.就是: (1)不论心是什么实数,函数的值总是正数,就是 a"-0 (2)当a>1的时侯,有 >1如果x>0, -1如果无U, <1如果x<0 当00, =1如果心=0, >1如果x<0. (3)当a>1的时侯.函数y=是增函数: 当00,4≠1)的性质 ·297· ==========第308页========== *这三个性质可以证明如下: (1)第一个性质的证明这里a是底数大于0的幂,我们只需应用幂的意义来证明,因为心可以是正数,负数或零,所以要分成三种情况.当x=0时a°=1,这从零指数幂的定义就知道,所以只要考虑x>0及心<0两种情况.如果能够证明x>0时a>0,那末对于龙<0,利用负数指数 的定义是也就证明了0>0,所以间愚只在于证明: 当>0时a>0.对于G>0,又可分成c是正整数,正分数,正无理数这三种情况. 如果x是一个正整数,那末,由于a>0,所以 ar>0. 如果亚是一个正分数,记这个正分数为,这里m,” 都是正整数且n>1,那末, af-a-a 由于>0,所以m是一个正数,因此 am>0即a>0. 如果心是一个正无理数.用x1和x2分别代表的任何一个相同精确度的不足近似值和过剩近似值.这时因为 a>0,a2a>0, 根据正无理数指数幂的意义,a是夹在a1和,中间的数。就是 当a>1时, 0 a>a>0 .、a>0. 综合上面这三种情况,我们就证明了当心是正数的时 ·2980 ==========第309页========== 候,都有 a>0 现在设:是个负数,记这个负数为一p(p是正数),那宋 Q), 因为>0,p是正数,从上面证出的结果,有 >0,.1>0. 狱是 x>0. 把上面结果综合起来,就得到,不论心是什么实数,都有 a*>0(a>0). (②)第二个性质的证明我们证明性质中的第一个部分,即a>1的情况,至于第二部分00. 如果是一个正整数,那末由于a>1,所以 a>1*,即a>1. 如果心是-个正分数咒(m,都是正整数且m>1), 那末由于x>L,m是正整数,根据上面已证得的结果有 am>1,.'.am>T. 就是 a>1,即a2>1. 如果是一个正无理数,设1是它的一个不足近似值,那末,由于a>1,c1是正有理数(正整数或者正分数),根据上面已证得的结果有 a1>1. 但是因为>x1,a>1所以有 ·2990 ==========第310页========== a2>a,.'.a*>1. 综合上面这三种情况,可知当a>1时,不论x是任何正数,都有 a2>1, 设花<0.令x=-p(p是一个正数),那末 a*=a-n--12D. 因为a>1,p是正数,根据上面已证明的结果,有 是<1,即<1 综合上面结果,性质(2)的第一部分完全证明. (③)第三个性质的证明我们只证明性质中的第一部分a>1的情况,而把第二部分的证明留给读者. 要证明函数y=Q在a>1时是增函数,我们只需在它的定义域一心<心<十∞里任意取自变量x的两个值x1和c2,并且211,根据指数函数的第一个性质有 a1>0. (2) 因为10.又因u>1,根据指数函数的第二个性质有 aa-x1>1, ..1-a1<0 (3) 从(2)和(3)得 a(1-a-)<0. (4) 由此从(1)可知 a1-a:<0, 即 a11时是增函数. 注 更简便一些,也可以利用<1来证明aa, (2)a3a (3)aa子 试决定a是大于1的数还是小于1的数3,如果 (1)3.52>3.5 (2)(증)() (3)(0.3)≤(0.3)”, ④(<(y, 比较:和y的天小, *4.证明,当a>1,x1<2的时候, s1.ar1 由此证明,当a>1.时,函数y=a”是增函数。 5.证明指数函数第.二个性质的第二部分. *G.证明指数函数第三个性质的第二部分。 4.例题 下面我们来举例说明怎样研究同底数的两个指数函数1=2)和妇=)间的关系例1 有两个指数函数 1=2x1-8w+1和2=21+2-7, 问x取什么值的时候,这两个函数的值1和y2之间有下面的关系: (1)y1=y, (2)y1>y9, (3)y12.那末 22x1-3x+1>2x2+8x-5 这里底数2大于1。因为当底数大于1时,同底数幂中,值大的幂的指数较大,所以 2x2-3x+1>x9+2-5, 就是 (x-2)(x-3)>0。 解这个不等式,得 <2或x>3. (3)如果y19, (3)y1y2,那末 a2x2+1>at+9 ()当a>1的时候,有 2x2+1>x2+2, 就是 2-1>0. 解这个不等式,得 <-1或>1. ()当01的时侯,-1<<1;(i)当01. 习题 1.(1)在同一坐标系里作出指数函数 71 1=2x和2=2 3) 的图象; (②)从图象上观察x取什么值的时候: y1=y2,y1>2,h<5 (③)应用同底幂的性质来证明上面的劳论. 2.在x取什么值的时侯, (1)243>24e; ②(侵)>(侵) (3)+3>a“(a>0,a+1). 908· ==========第314页========== §7·2对数函数 1.对数函数的意义 我们来研究和上一节里所提出的那个问题性质相反的问题。 已经知道镭在一年内蜕变0.044%,那末重是1克的 馏经过多少年的蚊变,所剩下的笛是克,是克,号 克…? 在:§7·1里,我们已经知道,1克的镭经过心年以后,剩下的镭的克数划,可以用公式 999561 、100000 (1) 来表示,这个等式给出的是以为自变量的指数函数 现在的阿题是要求出变量)取某一确定的值(侵,是号函时候,对这的变受☑能值是什么?在862里,我 们已经知道,上式可以改写成 2x=10g999cy. (2) L00000 这里,当y在:它可取值的范围(y>0)里,取每-个确定的值的时候,都有心的-一个值和它对应,所以这个等式给出了以y为自变量的函数.这也就是说,等式(②)表示的函数是原来这个指数函数的反函数.因为在这个函数中,自变量出现在对数记号后面,我们把它叫做对数数. -一般地说,指数函数 y-a(a>0,a≠1) (1) 有一个反函数,就是 x=1ogwy(a>0,a≠1,y>0). (2) ●8040 ==========第315页========== 习惯上,我们把心表示自变量,y表示函数,在等式(2)中把两个字母:和y对调,就得到 y=logaa(a>0,a≠1,c>0), (3) 并且把它看做是指数函数y=a的反函数. 函数=oga(a>0,a≠1)叫做对数函数. 因为函数y=(a>0,a≠1)的值域是(0,+∞),所以函数y=10gac(a>0,a≠1)的定义域是(0,+oo).例1 求下列这些函数的反函数: (1)y=2,2②y=(),(3)y=10 [解] (1)由y=2,得 优=10g3则. 把字母x和y对调,就得 y=10g3w。 (2)由yー(),得 x=10g号4, 把字母x和y对调,就得 g=10g花, (3)由y=10-,得 =10g10y. 把字母和y对调,就得 y=10g10化. 2.对数函数的图象 假设我们要画出例1里这三个对数函数的图象.因为它们分别是指数函数 ·305· ==========第316页========== 的反函数,所以只需在直角坐标系里分别作出这些指数函数的图象,然后作出它们对称于直线=心的曲线,就分别得到对数函数 y=10g9化,y=10g1龙,y=10g1wx 的图象了.例如,图7·2就是在同一坐标系里指数函数y=22和它的反函数,对数函数y=10g2无的图象。 1=2 -y= 图7.2 把上面三个对数函数画在同一个直角坐标系里,就得到图73 从这三个图象中,可以看到:图象都在y轴的右方; ylogx 等=0g10 一y=1og8 图73 ●8000 ==========第317页========== 图象都经过点(1,); 当底大于1的时候,图象是上升的,并且这图象在<1时在轴的下方,在x>1时在轴的上方; 当底小于1的时候,图象是下降的,并且这图象在x<1时在心轴的上方,在心>1时在c轴的下方。 3.对数函数的性质 从上面三个对数函数的图象中发现的特点,我们可以归纳出对数函数y=logax(a>0,a≠1)的如下性质: (①)自变量x只能取正数值,而函数的值可以是住意实数 (2)当a>1的时候,有 >0, >1, logax=0, 如果=1, <0, <1; 当01, loga=0, 如果=1, >0, <1. (3)当a>1的时侯,函数y=10gac是增函数. 当00,a≠1)的性质 例2 求下列各函数中,自变量x可取值的范围: (1)y=10ga√x-1;(2)y=10g(ax2-7x+6). [解] (①)要使10g4c一1有意义,必须并且只须 NG-1>0. 由此得 心-1>0,e>1. ●07● ==========第318页========== (2)要使10g(22-7x+6)有意义,必须并且只须 2x2-7x+6>0, 就是 (x-1)(x-6)>0. 解这个不等式,得 2<1.或x>6. 所以x可取比1小的或者比6大的一切值.例3 已知 1=10ga(a21),y2=10ga2x, 问x取什么值的时候, (1)y1= (2)y1>y9 (3)10, (1)1=ya.这时 10ga(c3+1)=10g,2x, 由此得 x2+1=2, 就是 (a-L)30. 解这个方程得 龙=1. (2)y1>y3.这时 loge (a2+1)>loga 2x. 如果a>1,那末 x3+1>2x, 就是 (x-1)2>0. 解这个不等式,得 比中1, 因为x必须是正数,所以可取值的范围是 01, ◆308· ==========第319页========== 如果01,不论c是什么实数,都不能使y1>.如果01. 习题 1.(1)下面是x和10g1ox的对应值的表: 72 1 3 5 6 2 log.4 -0.60-0.3000.300.480.600.700.780.850.900.951 在横轴(x轴)上取长5毫米的线段作长度单位,在纵轴(y轴)}取长50毫米的线段作长度单位,作出函数 y=lOg10花 的图象; (2)利用这个图象,求下面的对数(精确到0.1): 10g103.4,1og105.6, 10g108.4,og1o9.6; (3)利用这个图象,求真数x(精确到0.1),如果 10g10x=0.4,1og1ox=0.5,1og10x=0.7,log10x=0.8. 2.(1)在同-一坐标系里,画出对数函数 y=logx和y=log:出 的图华(e取9,31,寻台京等值力 (②)写出这两个函数有哪些相同的性质和不同的性质。3,求下列这些函数的定义域: (1)y=1oga(-x);(2)y=1og.(1一x); ◆09● ==========第320页========== (3)y=loga(1+x); (4)y=lga(1.+x). 4.比较下列每一组里两个对数的大小 (1)l1og1:6和1og105; (2)log23和1og4; (3)og13和log34; (4)1og:6和og:7. 5.指出下列各对数在那两个整数的中间:(1log1:11 (2)1og10123; (3:og27; (4)1og20.2; (⑤)10g告2: (6)og10.2. 1 [解法举例:(1).·1og1010=1,og1o100=3,而 1og1010<1og)14<1og10100,,∴.1log2 n; (②)1og:m>0g:: (3)10g0.2my;1<92? §73指数方程 在代数第二册里,我们学过的那些方程,等式两边都是代数式,这类方程叫做代数方程.现在我们学过了指数函数和对数函数,就可以进一步来学习另一类方程.在这类方程里未知数出现在幂的指数里,或含在对数记号下,例如 1 2=8,10g0纺=2,10gz4=2 等等.这一节里,我们先来研究在指数里含有未知数的方 乙 程的解法. 在指数里含有未知数的方程,叫做指数方程。例如 20=5,22x4+1=2*+9,22=3x+1, 42-32+2=0, ·8t0· ==========第321页========== 等都是指数方程 指数方程只有在特殊情况下才能够用普通的方法来解。下面我们举例说明儿种简单的指数方程的解法. 1.最简指数方程的解法 我们来看下面的例子: 例1 解方程: (1)10-=5; 2)2=1 [解] (1)把等式10*=5改写成对数等式的形式,得 x=1g5. 这就是所求的解. (②)根据指数函数的性:质,不论x是什么实数, 2>0. 所以方程2=一克没有解. 象上面例子里的这种指数方程,叫做最简指数方程.最简指数方程的一般形式是 a=b(a>0,a≠1). 很明显,当b>0时,它有唯一的解x=l0gab,在b≤0时,它没有解. 例2 解方程2=5(精确到0.01), [解] 从原方程得x=10g?5.应用换底公式,得 x=10g5=1g5-0.6990 g20.3010≈2.82. 2。形如)=a的方程的解法 我们来看下面的例子: 例8 解方程22+1-2+。 0号110 ==========第322页========== [解] 要使两个同底数的幕相等,必须并且只须它们的指数相等,所以从原方程得 2:+-1=22+2. 就是 a2=1. ,土1. 注 上面的解法,也可以这样来叙述: 原方程的两边取对数(底可以是任意不等于1的正数),得 l0g222+1-0g2+2, .(2x2+1)l0g2=(x2+2)log2. .lcg2=0,把这个因式约去得 2a2+1=x2+2, 解这个方程得 x=士1, 这样的解法,叫做两边取对数的解法,般地,要解形如 ax)=ax)(a>0,a≠1) 的指数方程可以两边取对数(底可以是任意的不等于1的正数),得 f(a)loga=o(a)log a. 因为1oga≠0,把这个因式约去即得 f(x)=p(x). 这样,只要求出方程∫()=p()的实数根,也就得到了原来这个指数方程的解 例4 解方程22$+1(精确到0.01). [审题] 这里等式两边不是同底数的幂,不能仿照例3的解法,但是可以用两边取常用对数的方法来把它变形成一个代数方程解出. [解] 两边取常用对数,得 xlg2(+1)1g3,。°x(0g3-g2)=-g8。 ●312● ==========第323页========== 1g3 0.4771 1g321g2=-0.4771-0.3010 0.4771≈-2.71,0.1761 习题 解下列方程: 7•3 1.(1)5*=G2: (2)2-8=16; ① (320-3 (到49=是 (》-(》: 0(-(佳 2.(1)2-2=1; (2)32x-1=1; (3)ux-2x-3=1(a>0,a≠1): (4)z2--2=1(a>0,a≠1). 3.(1)10=300; (2)10=5.75; (3)2z=10: (4)3*=12 4.(1)2x+-26=96; (2)7-7-1=6; (3)2x-1+2x-9+2-8=448;(4)5*+3×5-2=140, 3,形如f(a)=0的指数方程的解法 在代数第二册里,我们曾经学过双二次方程的解法.类似地,我们可以用变量代换的方法来解形如 f(a)=0 的方程.现在我们来看下面的例子:例5 解方程: 4g-32*+2=0 (1) [审题] 这里1=2=(2)。如果令2=,就可以得到y的 二次方程y一3则十2=0.由此可以求出y的正数值,然后再解最简指数方程2”=y.[解] 令2z=y,那末42=22=y2.代入方程(1)得 y2-3y42=0. 解这个方程,得 y1=1,y=2. ·3136 ==========第324页========== 由ㄌ=1得 24=1,.x=0 由g=2得 22=2.‘.x=1. 注 用这种方法求出的结果,只要计算过程不发生错误,一定是原方程的解,所以通常可以略去检验这一步骤。例6 解方程: 4x+8-2*=15 [解] 令22=则,那末4+2=4.4=162y.代入原方程得 16y2-y=15, 就是 16y2-y-15=0. 解这个方程得 语ya=1. 但是2不能等于负值,所以y的第一个值应当舍去.由则的第二个值得 22=1,.优=0. 例7 解方程 3+2-826*0. (1) [解] 令古=3x,那末 8+=8.82=9形,81=1 3· 代入(1)得 1-6=0. (2) 方程(2)可以变形成 9t2-6t+1=0. 解这个方程得 08140 ==========第325页========== 8=1 3 =3-1。 =ー1 习题解下列方程:73 1.(1)32-3=702: (2)2×32-5×3-1323=0; 2) (3)22z+1+2x+9=16: (4)32s+5=3+9+2 2.(1)3w+2+9*+1-810=0;(2)4+2m+1=80 3.(1)22-3-3×2-8+1=0; (2)34元-4×32v元+3=0. 4.(1)5-52-”+-2年=0; (2)3+34--30=0. §7·4对数方程 在对数记号下含有未知数的方程叫做对数方程.例如 10gvsx=2,gw+1g(x-3)=1, 210g26花+10g25=3, 等等,都是对数方程 象指数方程一样,对数方程也只有在特殊情况下才能够用普通方法来解。下面我们举例说明几种简单的对数方程的解法 1.最简对数方程的解法 我们来看下面的例子: 例1 解方程: (1)1ogv至花=2; (2)10g5c=-1. [解] 根据对数的定义,这两个方程可以分别写成下面的形式 (1)=(√2),就是x=2. ∴.原方程的解是化=2。 ②)=(侵),就是2. ·815◆ ==========第326页========== ·.原方程的解是x=2. 象这个例子里的对数方程,叫做最简对数方程.最简对数方程的一般形式是 lgax=b(a>0,a≠1). 根据对数的定义知道,不论b是什么实数,它都有唯一的解: g-ab, 2。形如1ogaf(c)=10gap(c)的方程的解法 我们来看下面的例子: 例2 解方程: 1g(2-c-3)=1g(2+1). [审题] 先求出方程中未知数x可取值的范围.这样就可以根据:两个相同的底的对数相等,必须它们的真数相等,得出 一个代数方程,再解.[解] 方程里未知数可取值的范围,由下面的不等式组确定: 22--3>0, 2c-+1>0 这个不等式组的解集是心>1+W1⑧(图74). 1- 1+W/13 2 图74 当心的值在这个范围里时,要使两个同底的对数相等,必须它们的真数相等,所以得到方程 2-0-3=2十1, 就是 x2-3-4=0. 解这个方程得 ●3160 ==========第327页========== 1=一1(不在心可取值范围里,把它舍去),2=4(检验后可知适合原方程). 答:所求的解是心=4。 注意〕 从这个例子中可以看到,方程 1g(x2-x-3)=1g(2x+1) 和 22--3=2c+1 不是同解方程 一般地,要解对数方程 logaf()=loga(x), (1) 我们可以在 f()>0和p()>0 的条件下,使真数∫()和p(x)相等,得出 f(x)=p(x). (2) 解这个方程,求出能使f(x)>0,(x)>0的根,就是方程 (1)的解. 注 这里把方程(1)变换成方程(②),通常也叫做在方程的两边同时取吞 例3 解方程: 1gx-+1g(x-3)=1. (1) [审题] 这个方程等号左边的式子,可以变形成1gx(花一3)的形式(>0,龙一3>0),而右边的式子可以变形成g10.因此可以利用两边取幂的解法[解] 方程(1)中未知数可取值的范围由下列不等式组确定: c>0, 化-3>0 这个不等式组的解集是x>3. 在x>3的条件下,我们把方程()变形成 1gx(w-3)=1g10. 由此得 x(x-3)=10, 就是 c2-3am-10=0. ●317· ····ド ==========第328页========== 解这个方程得 =一2,(不合条件,舍去)g=5. (适合原方程) .∴.所求的解是x=5. 〔注意〕 方程1gx+1g(-3)=1和方程1gx(x-3)=1的解是不同的.在前面这个方程里只有一个解心=5,但是后面这个方程就有两个解1=一2和2=5。所以把前面这个方程变换到后面这个方程的时候,必须注意在什么条件下,等式 Igc-1g(a-3)=1gx(c-3) 才能成立. 例4 解方程: gx2=2. [解] 从所给方程得 1gx2=1g100,.2产=100,..=士10. 〔注意〕 本题如果把1g变形成2g,得出 21gx=2, gx=1, x=10 这样就失去了一个解=一10.这是因为 1g221g 在>0的条件下才能成立,所以把1gQ2变形成1gc就缩小了未知数心可取值的范围.正确的解法是要把1g先变形成2gx,这样就可以得到 21g2x=2,∴.1gx=1, 。x|=10,.=士10 习题 解下列方程: 74 1.(1)1gx=21g3; (2)gx=21g3; (3)1g2=2-1g3; ④gx=令g3. ·818· ==========第329页========== 2.(1)2gx+g7=g14; (2) 坞龙 1g3*8 3.(1)gx+g(x-3)=1; (②)g(c+6)-号1g(2x-3)=2-g25;*(3)21g=-lg(6-x2) 1 ;(④6-g0+1+g1 4.(1)1g(x2+1)-2g(x+3)+g2=0 ②)是(lg-g5)=g2-号g(9-). 3.形如f(1ogac)=0的方程的解法 这种方程也可以应用变量代换的方法来解,举例如下: 例5 解方程: 2(1gx)2-31gx+1=0. [解] 令gx=y,代入原方程得 22-3则+1=0. 解这个方程得 云9n*1. 由此得 g-是,=102-而, gx=1,心=10. 〔注意) (Qg)和1g表示不同的意义,(lgx)是1g龙的平方,就是心的常用对数的平方,但是g心是的常用对数. 4。对数的底中含有未知数的对数方程的解法 有些对数方程在对数底中含有未知数,在特殊情况下也可以用普通方法来解.例6 解方程 210g2sc+10g25=3。 ●3190 ==========第330页========== [审题] 这里1g:25中的底和真数恰巧是1og5x的真数和底. 我们可以利用换底公式,把1og:25变形成1es25再做.10g23x [解] 未知数x可取值的范围是x>0,≠1. 10gz25=10g32.251 10g花10gan 代入原方程得 210g9龙+ 1=3 10g9 两边同乘以1og5x,整理后得 2(10g352)3-310g2,x+1=0. 就是 (210g2s一1)(10g25-1)=0.10g35化=1 这时c=25空-5, 10gax=1, 这时x=25 检验后知化=5,x=25都是本题的解. 习题 解下列方程: 7·4 1.()ー一(2)8に1-o (3)(gx)2+lgx2-3=0;(4)lgx-2(gx)1-1=0. 2.(1)21og4c421ogs4=5; (2)2loge25-3log25x=1. *§7.5指数方程和对数方程的图象解法 象-一元二次方程的图象解法-一样,我们也可以利用图象来解某些舍有指数函数或对数函数的方程.例如 22-4=0,2]0g2x十心-8=0 等等.这类方程通常也把它们分别叫做指数方程和对数方程①,现在举例说明如下: ①指数方程和对数方程都是超越方程。超越方程在一般情况下是不能 严格把它们分类的.例如用图象法我们可以求方程1og2c+2=0的近似解,但是这个方程里既有指数函数也有对数函数,我们就不能把它分在哪-·类中。 ·820· ==========第331页========== 例1 解方程 2*-4花=0, [解] 把原方程变形成 2=4就. 作函数 u=2和 2u=4花 的图象(图7·5). 从图中可以看到,这两 礼=2 个图象有两个交点,它们的横坐标分别是 1≈0.3,cg=4,为了检验这两个值是不是原来方程的解,可以代入原方程进行验算 当=4的时候, 24-44=0, 所以=4是方程的解。 当x≈0.3的时候, 图7.5 20.3-4×0.3≈1.23-1.2≈0, 所以可看做是方程的一个近似的解. 从这个例子可以看到,用图象法求得的解,一般只是一个粗略的近似值例2 解方程 210g2c+c-8=0. [解] 把原方程变形成 10ga心=4-克, 在同一直角坐标系里,作出函数 u=10ga龙和u=4-- ●321● ==========第332页========== u=logxt 图7.6 的图象(图76). 从图中看出两个图象只有一个交点,它的横坐标是 化=4, 这就是所求的解,注 1,直接由观察知道,x=4确实是方程的解,这是因为 210g24+4-8=4+4-8=0, 但是从观察中却还不能确定方程除了这个解以外是否再有其他的解,利用图象可以解决这个问题. 2.这里我们求出了精确的解,只是一个特例,一般情况下,利用图象法也只能求出方程的粗略的近似解 习题 利用图象法解下列方程: 7.5 1.2-x一2=0 2.20-2=0; 3.2l0g2-3=0; 4.30g3+2x-6=0. *§7·6指数和对数方程组 利用解代数方程组的知识,我们可以解由指数方程和对数方程所组成的方程组.下面举儿个简单的例子来说明这种方程组的解法例1 解方程组: 了2*=8划+1 (1) 9y=3-9, (2) ·322。 ==========第333页========== [解] 从(①)得 28=28(w+1) ..=3(y+1). 从(2)得 38w=3x-9, .2则=g-9. 这样,我们就得到了一个二元一次方程组 化一3y=3, 1w-2则=9. 解这个方程组得 =21, y=6. 这就是原方程组的解。 例2 解方程组: gc+1gy=1g11+g9, (1) g(w十y)-1g(g-)=1 (2) [解] 从(①)知方程组有解的条件是 x>0,y>0, 在这种条件下,从(1)得 1gy=1g99, °。y=99, 从(2)知方程组有解的条件是 x+y>0,c-y>0. 在这种条件下,从(2)得 1g+g-g10,.G十丝=10.c--q 花-y 就是 9x=11g. 这样就得到了一个二元二次方程组 y=99, 9x=11y. 解这个方程组得 0923● ==========第334页========== 1=11,f花2=-11 y1=9,【2y2--9. 因为x和y都不能是负数,所以第二个解应该舍去.以x=11,!9代入原方程组检验,知道它们适合原方程组里的两个方程,所以是原方程组的解注 当G=11,y=9时,x十y和x一y的值都大于零,根据这一点,可知然=11,y=9同时满足方程组中两个方程所需要的条件,从而可确定这…组解是原方程的解 例3 解方程组: g=1.00 (1) 1gx+1gy=3. (2) [解] 从方程(1)知道方程组有解的条件是 x>0,y>0. 在这样的条件下,两边取常用对数得 1gylgc=2. 原方程组可以变形成 igylga=2, (3) lg+1gy=3. (4) 令1gx=u,1gy=w,得到 u+2=3, u=2. 解这个方程组得 u1=2, =1, 和 ∫g=1, 2v22. 从u1,U1得 ∫gx*2, 1gy=1. {=10,y-10. 从u2,vg得 ●324● ==========第335页========== í1gx=1, 〔=10, (1gy=2. y=100 检验后,知道这两个解都适合原方程组, 0习题解下列各方程组:76 9x+#=729, g x+1gy=5, 1.(1) (2) 3x-y-1=1; lg x-lgy=3: x+y=5, (3) E-y=90, (4) (1gx+1gy=3; 9.3r+出-3-2w=0, 2x.3型=324, 2-29=24, 2.(1) (2) 13.2=144; 24-1+2u-1=20, 3.(1) Igx+lgy=3, 1g(x+90y)-1gx=1; Igx41gy=lg 143, (2) Ig(a+y)=lg(a-y)--1g12. §77指数和对数不等式 在§7·1里,我们曾利用指数函数的性质,研究过下面这样的问题 设 31=22-8z+1,y2=2143-5 问: 当取什么值时 (1)y1=, (2)y1>U2, (3)y12x3+2x-5 (1) 2224-3x+1<2x2+3x-5 (2) 这种不等式叫做指数不等式. 同样的,由对数函数所组成的不等式叫做对数不等式例如 1g(2x2-3ax+1)>1g(ax2+2-5) (3) ·325· ==========第336页========== 是对数不等式. 解指数不等式、对数不等式的方法,主要是应用指数函数和对数函数的性质,把它转化成与之同解的代数不等式或不等式组.例如上面的这三个不等式,可以分别转化为解代数不等式: 222-3+ユ>2*+9ー5→2x2-3c+1>3+2-522x-5x+1<2x+2x-5=→2ax2-3x+11g(a2+2x-5) f2x2-3x+1>0, →{x2+2a-5>0, 2x3-3x+1>2+2c-5. 但是因为当底数a>1或0dがs(a>0,a1) [审题] 因为当底a>1和01时,指数函数是增函数,所以a*+8>ar=→x2+6>5a→2-5x+6>0 →(e-2)(-3)>0, 这时不等式的解集是x<2或c>3 (2)当底0aa→2x2+6<5→x2-5g+6<0 =→(x-2)(x-3)<0, 这时不等式的解集是2x2--2. 这样本题就可归结为解一个代数不等式组: 22-5x+2>0, 32+-2>0, 28--2>0, (2e2-5a+2)(3a2+x-2)>x2-x-2. [解] 请读者自行完成. 答:x<-1或>2. 从本题的解答过程,读者可以又一次看到:对于一些形式上比较复杂的问题,只要仔细加以分析,也就可以把它转化为一系列简单的问题,运用已经学过的知识来解决。 习题 1.解下列不等式: (侵)》 +2x-5 77 (3)3m-18.3+7≥0; (4)4+2-6≤0, 2.解下列不等式: (1)lg(2x+1)og影3a; (3)gV5+5≥1-子lg(2x-1; (4)1ogv5x0,a+1) (a>0,a+1) 定义域 -0<优<十∞ 0x<十∞ 值域 01时 (1)当a>1时 >1(c>0) I>0(x>1) =1(x=0) loga=0(c=1) 性 <1(a<0) <0(x<1) 1 (2)出00) I<0(e>1) (x=0) logox=0(x=1) >1(<0) >0(x<1) 质 (1)当a>1时 (1)当a>1时 ac是增函数 ]ogax是增函数 2 (2)当00,a中1) 1ogax=b(a>0,G≠1) 6>0,x=logab; x=ab b≤0,无解 可以用取对 (1)afx)▣a(x)=f()=中(x) loga f(x)=loga中() 数或者取幂 (2)af(r)=b) 「f()>0 解的方程 =→f()ga=b(x)lgb =中()>0 f()=中() 可以用变量 f(a)-0 f (logax)=0 代换法解的 令y=a,解方程f(y)=0, 令y=1oga化,解方程 方程 再解方程a=y(y>0) f()=0,再解方程 logax=y 图象解法 ax=f(a) loga c=f() 作函数y=ax和y=f()的图 作函数y=l0gax和y口 象,求出交点的横坐标 f()的图象,求出交点的横坐标 3.指数不等式和不等式的解法仿照指数方程和指数不等式的解法,把它归结为代数不等式或不等式组来解。在应用指数函数或对数函数的性质时,要特别注意: (1)底a>1时,函数是增函数一不等号不变向; (2)底01和0av3,a应该是怎样的数?知果a1时,函数的增 减性有什么不同? ·329· ==========第340页========== (2)如果loga0.2>oga0.3,4应该是怎样的数?如果1oga0.2 1 (2)a<1; (③)1g号>0 (④ig哥<0. 4.已经知道了函数y=3的图象,用什么方法可以作出函数y=1og3x的图象?为什么可以这样作? 5.在同一坐标系里作出函数y=log1ox和y=log1x的图象.比较它们有哪些相同的地方?哪些不同的地方? 6.(1)解下面的方程: (i)2x2=5x+7; (ii)32w2=35x+7 (2)这两个方程是不是同解方程? (3)方程a)=a)(u>0,a÷1)和方程f(x)=p(x)是不是同 解方程?为什么? 7.(1)解下面的方程: (1)log22x2=log=(x+6);(ii)2c2=x+6. (2)这两个方程是不是同解方程? (3)方程loga(x)=logp(x)和方程f()=p(x)在什么条件 下才是同解方程? 8,解下列方程: )4+g (2)3¥.2-1-3w1.2=36; {3)·lg(152+x2)=2lg(分+2); (4)(3-然+4)=2 Igx 9.如果劳动生产率平均每年比上一期提高18.9%,那末经过多少年可以提高到原来的2倍? 10.机器的价值是10万元,如果每年的折旧率是8.75%(就是每年 减少它的价值的8.75%),那米,大约几年后它的价值成为4万元? 11.解下列方程组: ·330◆ ==========第341页========== (1) 12-2学-13, .(2){以=40, 3lg2y-)=1; g=4; 12Vm+v可=512, (3)lyvi=a4; (4)llg√g=1+lg2, 12.解下列不等式: Dg-2+1>0: (2)2og25-31ogsx<1; (3)3+4-3w<80; (4)3+2+9+1-810>0. 复习题七B 1.求下列函数的定义域: (1)y=1g[1og(√m-8-2)]: (2)y=g(Vx2-3x+2-x-3)+10v+寸 2.求下列函数的单调性区间及极值: (1)y=a-3a+2(a>0,a+1); (2)y=28+2a-0; (3)y=1g(2ax2-5x-3; (4)y=(lgx)2一3lgc+2. 3.求下列方程的解集: (1)(W2-√V3)z+(W2+V3)=4; (2)loga 2.l0g2 2=log 2.4,解下列方程组: 2z.3y=648, (1)13.2=432: Ig va+lgvy=1g (4-Va), (2) (25)V7=125.5m 5.解下列不等式: (1)1og号V②c+I<1g号Vc+8+3 (2)g2-2>eV-8, 0831● ==========第342页========== 第七章测验题 1.对于怎禅的数,下列不等式是正确的: (1)log80: (3)2v3>x3v3, (4)(2V15)"<(5√3). 、2.求下列函数的定义域: (1)y=]n(e-ec; (2)y=(3-37÷)1. 3.求出下列函数的单调性区间,并画出他们的图象: (1)y=31x-21+i+; (2)y=g(x2+4+14). 4.当整数飞取什么值时,方程 c2-2x-+g2=0 有实数根? 5.解下列方程: (1)G.9-136正+64立=0 )log1) logs x2 ·332◆ ==========第343页========== 8 数列、数学归纳法 前面儿章里,我们研究的函数的定义域都是实数集或者是某些实数区间.这一章里,我们将要研究另一类重要的函数,他们的定义域是自然数的集合,或者自然数集的子集{1,2,3,…,.这种函数的值,可以按自变量等于1,2,3,…的顺序顺次地排列起来,得到一连串的数,通常把它叫做数列 结合着数列的研究,这一章里我们还将学习数学中一种重要的证明方法一数学归纳法. §81数列 1,数列的概念 我们来看下面的例子 (1)一个生产小组,把所生产的产品堆成图8·1的形状.这里从最上面的一排起,各排产品的个数依次是: 3,4,5,6,7,8,9. (2)把自然数从1起依次排列起来,得到一连串的数: 1,2,3,…,,… (3)把自然数的倒数,依 图8,1 次排列起来,就得到一连串的数: ◆333 ==========第344页========== 1111 11 (4)把√乞的不足近似值,按照精确度00产,,…的顺序排列起来,就得到一连串的有理数: 1.4,1.41,1.414,…. ()把√2的过剩近似值,同样地排列起来,就得到 一连串的数: 1.5,1.42,1.415,…. (⑥)把一是的1次茶,2次暴,3次屠,4次幂…顺 次排列起来,就得到一连串的数 1111 -24’-8,16…. (7)f()=3-2肌在自变量%依次取1,2,3,4,…时所得的函数值,排列起来就得到一连串的数: 1,-1,-3,-5,…. (8)把f(m)=n2一(n+1)(n一1)在自变量n依次取1,2,3,4,…所得的函数值,排列起来就得到一连串的数: 1,1,1,1,…. 象上面这些例子那样,依照某种规定排列着的一连串的数叫做数列. 注在一个数列里,每一个确定的位置上都有一个确定的数.例如, 在上面的数列(1)里,第3个位置上的数是5,在数列(4)里第2个位置上的数是1.41,等等.因此我们可以把一个数列里的数看做它所在位置号数的函数;自变量是位置号数,它可取的值是1,2,3,,而对应的函数的值,就是数列里的各个数. 2。数列的项 数列的每一个数叫做数列的一个项;在第1个位置上 ●334◆ ==========第345页========== 的数叫做数列的第1项,第2个位置上的数叫做数列的第2项;般,在第%①个位置上的项,就叫做第见项.数列的第%项通常可以用a来表示.例如,在数列(③)里, 数列的第项,也叫做数列的通项.如果这个项a,和项数?间的关系可以用一个公式来表示,这个公式就叫做这个数列的通项公式.例如,数列(3)里的通项公式是 1 an 数列(6)的通项公式是 ()》”, 等等 〔注意〕 不是每一个数列都有通项公式的.例如在数列(④)和 (5)里,我们就不能写出它们的通项公式. 知道了一个数列的通项公式,依次在%可取值的范围里,让%取各个不同的值代入公式,就可以求出这个数列的各个项。 例1 已知数列的通项公式是 an=2X 2n, 写出它开头的四项[解] a1=2×22=4, a2=2X22-8, a3=2×23=16,a4=2×24=32 反过来,如果知道了一个数列的开头几个项,也可以用试探的方法,写出它的一个通项公式,但是这种通项公式并不是唯一的. 例2 已知一个数列的开头几项是 18,5,7,…4 ① 本章中%都表示自然数。 ●3g60 ==========第346页========== 求这个数列的一个通项公式.[解] 从观察中,数列中从第二项起,后一项减去前一项的差都是2,我们可以认为这个数列的通项公式是 an=1+2(n-1). 〔检验) 01=10=1, ae=1+21=3, 0:=1+2.2=5, a4=1+2.3=7. 注 我们也可以写出另一个通项公式,使以这个公式算出来的前4个项就是1,3,5,7.例如取 =1+2(n-1)+(8-1)h-2)(-3)(2-4), 很明显,当=1,2,3,4的时候,因为公式中 (2-1)(6-2)(8-3)(2-4)=0, 所以有风=1,a=3,a%=5,a=7.它与an算出的结果相同.但是当>4的时候,应用公式aa和a,算出的结果就不同了,例如 a5=1+2(5-1)=1-8=9, 而 a6=1+2(5-1)+(5-1)(5-2)(5-3)(5-4)=1+8+4.3.2,1=33 通常,遇到这种题目,我们总是设法写出一个比较简单的通项公式就可以了 1.写出下列各数列(写出前面3项,和它的通项): 习题 81 (1)从小到大排列着的所有正的奇数; () (2)从小到大排列着的所有正的偶数; (3)从小到排列着的所有自然数的平方; (4)从小到大排列着的所有自然数的立方,[解法举例:(1)1,3,5,…,2-1,….] 2.根据下列通项公式,把各数列的前5项写出来: (1)au=3n; ②n平 (3)xn=(-1)2n*1; (4)an=(-1)n+1-% 21· 3.找出下列各数列的组成法则,写出每一个数列的通项公式,并写出它的第5项 (1)10,20,30,40,… (2)2345 T’艺3:4?…5 ◆886· ==========第347页========== (3)1111 12’2334}4.59 1 (4、3’9’27’81’…. 4.已知数列 1.3,24,3.5,…,12(2+2),y (1)求这个数列的第9项;第98项; (2)下列每一个数是不是这个数列的一项,如果是的话是第几项? 80,100,120,255. [解法举例:设第项是80,则 n(1+2)=80,22+22-80=0,i1=8,2=-10(不合). ,'.80是这个数列的第8项,] 3.数列的图象 象函数的图象一样,我们也可以把一个数列用图象表示出来。这时,我们把直角 An 坐标系的横轴的正方向,作为表示数列的项数2的轴, 9 把纵轴作为表示数列各项的值an的轴. 5 4 例如,数列3,4,5,6,7,8,9可以用右面的图象来表示(图8·2). 1234567 应该注意,因为这里是一个个分散开来的数,所 图82 以数列的图象是一个个分散着的一群点.但是为着容易看出数列的变化情况,我们有时也可以把这些点顺次地用虚的折线连接起来,或者把各点的纵坐标用虚线作出来例3 作下面这两个数列的图象: ●337● ==========第348页========== (1)子尽(》,… (2)豆青子品 从这两个数列的图象上可看出它们各有哪些特点?[解] (1) 12 1 1 4 1 图83 这个数列中: 各项的值一负一正,在0的左右摆动; 各项的绝对值,最大的是a一受,以后逐渐减小,并 且向零接近。 (2) an 1 1 。● 234667 1 图84 .888。 ... ==========第349页========== 这个数列中: 各项的值逐渐下降且向0接近;最大的一项是a1=1. 注 作图时,a轴上的长度单位不必与%轴的长度单位相同. 习题 1.按照下面的通项公式,作出各个数列的图象(只要画出?取1到 81 6的自然数的一部分),并用虚折线连接起来: 2) (1)an=2+1 (2)an=(-1)n.%; (3)a= 2+1 2.从上题所作出的图象上可以看出这三个数列有哪些不同的特点? 4。数列的分类 我们观察第333页里提出的那8个数列,可以看到它们分别具有不同的特点. (1)从项数上考察:数列(1)只有7个项,但是其他的数列都可以有无穷多个项. 一个数列,如果在某一项的后面,不再有其他的项,这个数列叫做有穷数列;如果在任何一项的后面,都还有跟随着的项,这个数列就叫做无穷数列. 例如数列(1)是有穷数列,数列(2)(8)都是无穷数列 (②)从前后两项的值的大小比较上考察:在函数中我们曾经学过增函数,减函数的概念;对于数列来说,也有着递增数列和递减数列的区分.就是: 一个数列里如果从第2项起,每一项都大于它的前面的一项(an+1>an),.这个数列就叫做递增数列;如果从第2项起,每一项都小于它前面的一项(a+10, .∴.an+1>a, ∴.这个数列是递增数列. 又la,m华=年<1(:n1,我们可以把这个公式改写成 a1(q"-1) q-1 (V) 利用上面的公式,在已知等比数列的四个量Q1,9,n,$中的三个时,就可求出另一个来. 徜使在公式(V)里,把a1g"用通项am表示,那么有a1g”=ag”-1·9=a9,由此就可以推得公式 ·357· 3y51。 ==========第368页========== 8,=41ng 1-q (VI) 即 Sn-ang-an q-1 (VIa) 利用这两个公式,可以解决已知等比数列的四个量a1,am,q,s中的三个,求另一个的问题例3 等比数列前5项的和是242,而公比是8.求这个等比数列的前5项 [解] 把sm=242,q=3,n=5代入公式 S.=a(g-1) 9ー1 得 242=☑(35-1) 3-12 就是 242=121a1. ‘.1=2, 所以这个等比数列的前5项是2,6,18,54,162. 例4 等比数列的前3项的和是168,后3项的和是21。求由这6个数所组成的等比数列, [审题] 从已知条件,可以看出这个等比数列里s3=168,$=168+21=189.要求出这个等比数列应该先求出1和9,因为后三项的和比前三项的和小,所以g<1.我们应用公 式8=41g2来算. 1-9 [解] 设这个等比数列的第1项是1,公比是q.根据已知条件得 a1(1-g8)=168, 1-g (1) a1(1-9)=168+21. 1-9 (2) ②得 1.-g0 189 (1 1-g 168 0358· ==========第369页========== 化简后得 1+g39 8 91 1 ,9 将q的值代入第-一式得 o(-넣) =168, 1- 就是 7 4=168.:=96 所以这个等比数列是96,48,24,12,6,3。 习“ 1,求等比数列前n项的和,如果 83 (1)41=1,g-3,n=10;(2)a1=.1 (2) .1.11 (8)a=3,g=3,an=656i (4)a1=6,g=2,an=192 2.在下列各题里,由已知的三个数,求未知的两个数 题 an 5 题次 a1 (, 3 2 5 (6) 1 -512-341 (2) 8 1 8 (7) 受 10 7 (3) -27 13 -20 8) 2 4 65 (4 7 1458 9) 2 96 189 (5) 3 26 (10) 3 心 3 3.求等比数列的第1项,公比和项数,如果 斯一5=8, a6w=216, (1)a&十=18, (2 a3-418, 3=1023; n=40。: ·5g ==========第370页========== 4.有一个七层的塔,每层所点的灯的盏数都等于上面一层的2倍, -一共点了381盏灯,求底层所点灯的盏数? 5.作一个每边长2厘米的正方形,再以这个正方形的对角线为边作第二个正方形,以第二个正方形的对角线为边作第三个正方形,这样一共作了10个正方形 (1)求第10个正方形的面积; (2)求这10个正方形的面积的和. §8·4等差中项和等比中项 在一些关于等差数列和等比数列的问题里,我们常常要用到等差中项和等比中项的概念。现在,我们来研究下面两个问题: 已知三数a,A,b成等差数列,这三个数之间有什么样的关系? 已知三数a,G,b成等比数列,这三个数之间有什么样的关系? 1.等差中项 根据等差数列的定义,我们知道如果a,A,b这三个数成等差数列,那未一定有 A-a=b一A, 由此可得 2A=a+6, 所以 4=a+6 2 反过来,蜘果A-士白,那末从 4-a=a+b-&=6-&, 2 2 6-4-d-含2-82品2 ●04 ==========第371页========== 也就可以知道a,A,b这三个数成等差数列。· 我们把 A女b 2 叫做a和b的等差中项注 等式2A=a+b,常被用来证明a,A,b三数成等差数列的问题. 例1 已知a,b,c三数成等差数列,求证 b+c,c+a,a+b 三数也成等差数列. [审题] 要证明b十c,c十a,a十b三数成等差数列,只需证明 (b+c)+(a+b)=2(c+a). 因为a,b,c成等差数列,所以有条件 a十c=2b 我们就利用这个条件来推出前面这个等式成立。 [证明] ·a,b,c成等差数列, .∴.a+c=2b. 今 (6+c)+(a+)=2b+(a+c)=(a+c)+(a+c) =2(a十C). .a+c是b+c与a+b的等差中项. .b十c,c十u,a十b这三个数成等差数列. 注 这个题目也可根据等差数列的定义,从已知条件b一a=c一b,推出(c+u)-(b+c)=(&十b)一(c+a)而得到证明。证明留给读者。 2.等比中项 根据等比数列的定义,我们知道如果a,G,b这三个数成等比数列,那末一定有 Gb 由此可得 42=ab, r8681· ==========第372页========== 当ab>0的时候①,就有 G-±ab 反过来,如果G=士ab,那末从 G+√ab B b 和 a±Td 也就可以知道..,飞这三个数成等比数列 我们把 G-士ab 叫做和b的等比中项。 ·注 等式G=b常被用来证明a,G,b三数成等比数列的问题. 例2 求a+a2b和4+a2b”(a≠0,b≠0)的等比中项. [解] G=(a4+a26)(34+a62)=a2.飞2(a2+b)2.,.G-士a6(a+b2 ±la|이(a2+62)=±ab(a3+3). 注意) 在实数范围里研究根式,当Q,b不等于零时,我们有 ,e-。(如果a>0), (如果a<0); 、-{色为(如果b>0), (如果b<0). 当G:a·b(a2+b)的时候,可以得到两个结果: G-l1(-の-{ーな)ab(a2462) (如果a,b同号),(如果a,b异号). 同样,当G=-a·b(a2+)的时候,也可得到两个结果: la:ー{(6くな+),周)ab(a2+b2) (姐果a,6异号), ①因为我死在只学到实数,面任何实数的平方都不是负数,成要 主.ab>(这个条件。 ●802● ==========第373页========== 把这两种情况合并起来,可以写成 G=±ab(a2+b). 在解题埘,应该注意到这一点,否则答案虽然是正确的,但理由讲不出,或讲错了,都是不符合要求的例日 三数成等比数列,已知它们的和是26,各项的平方和是364.求这三个数[解] 设这三个数是x,以,y2.由已知条件得 x+cy十xy2=26, 12+y2+2y=364. 就是 x(1+y+y)=26, (1) し2(1+9+y)=364. (2) 品得 x(1+y+)=14, 1+9-4 就是 x(1-yy2)=14. (3) (1)-(3)得 2y=12, 6 ,x阳y 代入(①)得 (1+y+ア)-20 化简后得 3y2-10y+3=0. 解这个方程得 y1=3, 由此得 x1=2,xg=18. 取1=2,1=3得所求的三个数是2,6,18.注 如果取=8子,得这三个数是18,6,品,结果是一样 的 363· w:wr: ==========第374页========== 习 1.求下列各组数的等差中项: 84 (1)647和895; (2) 2和24行: (3)V3+V2和√③V? /3-V2 3+V花 (4)(a+b)和(a-b)2. 2.求下列各组数的等比中项: (1)-45和-80: ②唱和 (3)7+3√/5和7-3V√5;(4)(a+b)2和(a-b)2 3.三个数成等差数列,它们的和是15,它们的平方和是83,求这 三个数 4.一个三角形的三边成等差数列,周长是12厘米,面积等于6平方厘米,求这个三角形各边的长?这个三角形有什么特点?[提示:△=√s(8-a)(s-b)(s-c).] 5.已知a,b,c成等差数列,求证 a2(b+c),b2(c+a),c2(a+b) 也成等差数列. 6.(1)三个数成等比数列,它们的和等于14,它们的积等于64, 求这三个数; (2)三个数成等比数列,它们的积等于27,它们的平方的和等于91,求这三个数; (3)三个数成等比数列,它们的和是21,它们的平方的和是 273,求这三个数 3.杂例 三数成等差数列、成等比数列,常常联系在一起,或者与数学中其它的知识联系在一起,组成综合性问题.解这类问题的关键,就在于能够灵活地应用等差中项和等比中项的概念.下面再举儿个例子:例4 已知三数a,b,c成等差数列.三个正数c,y,:成等比数列,求证: (6-c)1ga+(c-a)1gy+(a-6)1gz=0. ●384● ==========第375页========== [审题] 由么“成等比数列,可推得号一兰取常用对数得 lg-gy=1gy-g名. 由a、b、c成等差数列,可推得 b-c=a-6. 只需把这两式两边相乘,即能证得。 [证明] 因为a、b、c成等差数列, .∴.b-c=a-b. (1) 又因、y、”成等比数列, 优=里 因为>0,y>0,>0,两边取常用对数,得 Iey-lgy .‘.1gx-gy=gy-1g2 (2) (1),(2)两式两边分别相乘,得 (6-c)(Iga:-1gy)=(a-6)(1gy-1g2), .(6-c)1ga-(c-a)1gy+(a-6)1g2=0. 例5 已知x是实数,a>0、b>0、c>0,且 (a2+b)-2b(a+c)c+b3+c2=0, 求证lga、gb、gc成等差数列. [审题] w和a、b、c都是实数,因此所给关于的二次方程的判别式4≥0,由此即可推得. [证明] .a、b、c、心都是实数,方程的判别式1=4b(a+c)2-4(a2+b)(b2÷c2) ー4(64-2acb+ac")=-4(6-ac)2>0. 因为a、b、c是实数,(b3一ac)不能为负数, .·.b2-ac=0,即b2=ac. 因为a、b、c都是正数,两边取常用对数,得 21g6=1ga+lgc. ·3650 ==========第376页========== 由此可知,]gb是1g,1gc的等差中项.所以ga、gb、gc成等差数列 习题 1.已知成等差数列的三个正数的和等于15,并且这三个数分别加 84 上1,3,9就成等比数列,求这三个数 ②) 2.组成等差数列的三个数的和是30,如果从第一个数里减去5,从第二个数里减去4,第三个效不变,那未所得到的数组成等比数列,求这些数. 8.成等比数列的三个数的和等于65,如果第一个数减去1,第三个数减去19,那未就成等差数列,求这三个数. 4,有四个整数,前三个数组成等差数列,后三个数组成等比数列.己知首末两个数钓和是37,中间两个数的和是36,求这四个数. 5.四个数组成等差数列.如果从这四个数里相应地减去2,6,7,2,那末所得到的数又组成等比数列,求这些数。 §8·5数学归纳法 1. 归纳推理 在前面,我们在求等差数列或等比数列的通项公式时,采用的方法都是先考察数列中一些特殊的项所共同具有的特征,然后归纳出一般的项(通项)也应该具有这样的特征,从面得到一个表达这种特征的公式.这种由特殊到一般的推理方法,叫做归纳推理或归纳法 例如,设一个无穷数列的前5项是: 8 44= 10 5¥05=6· 我们考察:这些项中,从第二项起都是分数,分子都是项数的2倍,分母都比项数大1: 09=422 2=》113=•343+1 64+1,102582.4 6.4+1 ●86● ==========第377页========== 面第一项也可以改写成一个分数 122.1 01-i21+I? 它同样具有这种特征;于是我们就可以归纳出一个结论:这 个数列的通项公式是an=2.n 2+1 这里应用的就是归纳推理。因为在这一推理过程中,我们只是考察了几个项所共同具有的特征,由此就作出了 一般的项也都具有这样的特征的判断,所以通常也把这种推理方法叫做不完全归纳法 又如,设函数f(c)-28+心+11,2∈{1,2,3,…,9].把自变量的值逐个代入计算,可求得对应的函数值是: f(1)=1-+1+11=13,f(2)-22+2+11=17,f(3)=32+3+11=23,f(4)=48+4+11-31,f(5)-52+5+11=41,f(6)=-62+6+11=53,f(7)=.72+7+11=67,f(8)=82+8+11=83,f(9)=92+9+11=101. 可以看出,上面所算出的这些值都是质数。因此,可以得出 一个结论: 当化是1到9的自然数时,函数f(c)=x+x+11的值总是质数 这里应用的也是归纳推理.因为最后作出的判断,是考祭了各种可能情况才归纳出来的,所以这种推理方法是完全归纳法,通常把它叫做穷举法 应该注意,用完全归纳法所作出的判断,只要前提正确,推理过程中不发生错误,最后作出的结论也总是正确的;但是用不完全归纳法作出的判断,就不一定都是正确的。 例如,在上的例子中,函数的定义域,知果是自然数 3076 ==========第378页========== 集N,那末我们仅仅考察了当心=1,2,…,9时,()= 心2+心+11的值都是质数,就贸然地作出“当心为任意自然数时,f(x)的值都是质数”的结论,那就错误了,因为当ヶ=10时,f(x)=103+10+11=121=113,它就不是一个质数了、 所以应用不完全归纳法,它仅仅给我们提供了一个可能成立的结论.这个结论对于我们列举验证过的范围以外的情况是否能够成立,还必须作进一步的证明, 习题 1.已知无穷数列的前5所是: 85 (1)1=1,a2=-3,a3=5,a4=-7,a6=9; ()a-1-고1,-·-,- 观察这些已给出的项的特点,写出一个用项数”来表示它的通顶an的公式 [提示:(-1)”当为偶数时等于1,当n为奇数时等于一1.] 2.考察下面两个数列所给出的各项与项数间有怎样的关系?由此写出它们的通项公式: (1)1,4,9,1.6,25,; 3133781151 23,4,万,6,” [提系-1+号]3 2.数学归纳法 我们来考察自然数列里从1开始的%个连续奇数的和与项数之间有什么关系. 1=1=19, 1+3=4=22, 1+3+6=9=32, 1+3+5+7=16=43 1+3+5+7+9=25=59, 从上面的计算可以发现一个重要的事实:自然数列里前1 ·8680 ==========第379页========== 个,2个,…,5个连续奇数的和,恰巧都等于这个和里加数个数的平方.很自然地我们可以作出这样的一般结论 自然数列里前2个连续奇数的和等于n3. 但是,这里华竟只考察了少数儿种情况,发现有这样的规律,这个结论是不是对于任意的自然数儿都正确呢?这就必须作进一步的证明 怎样证明这个结论确实是正确的呢?企图用一一验算的办法,显然是不行的.但是,如果能够证得“当和式里连续奇数的个数是某一个自然数(例如当肌=)时这个结论正确,可以推出对和式里连续奇数的个数再增加1个(例如,当=k十1)时也一定正确”这一事实,这时,就可以应用递推的方法,从这个结论对于饥=1正确,而推出对于n=2也正确;对于%=2正确,推出对于见=3也正确;这样顺次地推下去,显然,这个结论对于所有的自然数几都正确. 我们就采用这样的思想方法来证明上面这个结论.因为任何一个奇数都可以表示成2m一1(n是自然数)的形式,所以欲证明上面这个结论,也就是要证明等式 1+3+5+…+(2n-1)=n2 (1) 对于所有的自然数%都成立.[证] 当%=1时,有1=1,所以式(1)成立.很定当=飞时等式(1)成立,即 1+3+5+…+(2k-1)=2 (2) 成立。 那末,当m=+1时,式(1)的左边是1+3+5+…+[2(k+1)-1] =[1+3+5+…+(2k-1)]+[2(k+1)-1]=2+(2k+1)=(k+1)2 [应用式(②)] ·8u9· ==========第380页========== 面这时式(1)的右边也是(+1)2,所以式(①)当%=k+1时: 左边=右边. 这就是说,式(1)当=k÷1时也成立 这样,因为已验证了等式(1)当n=1时成立,可以顺次地推出当m=2,3,4,5,…时都成立.所以式(①)对于所有的自然数%都成立. 上面证明中所采用的方法,称为数学归纳法.从证明中可以看出,应用数学归纳法来证明一个命题,有下面这两个步骤: 1°先证明当命题中%取第一个自然数值α(例如n=1,或者=2,等等)时,这个论断是正确的. 2”假定命题中取某一自然数值飞时这个论断正确:在这基锄上证明命题中%取后一个自然数值k+1时,这个论断也正确 在证实了这两步之后,就可以作出结论:“命题对于从g开始的所有自然数%都正骗.” 在实际解题时的叙述可以比上面证明中的叙述再简化 一些. 例1 证明 112+22+…+2"-1*2"-1, (1) [证] 1°当物=1时,左边=1,右边=22一1=1,式(1)成立 2°假定当n=k时式(1)成立,即 1+2+23+…+2-1=2-1, .(2) 那末,由 (1+2+22+…+2-1)+2k+10-1=2k-1+2k [应用式(2)] =22一1=2k+1一1 可知,式(1)当m=k+1时也戚立. 000 ==========第381页========== 根据1°和2°,这就证得了式(①)对于所有的自然数n都成立。 例2 证明 1-35-7+…(-1)%-1(2n-1)=(-1)n-1m.(1) [证] 1当2:1时,式(1)的左边=1,右边=(一1)1=1.所以式(1)成立. 2嘏定式(1)当2=k时成立,即 1-3÷5-7+(-1)-1(2k-1)=(-1)*-k. (2) 那未,当n=k+1时, 左边=[1-3+5-7+…+(-1)-1(2k-1)] +(-1)k+1)1[2(飞1)-1]=(-1)-1k+(-1)*(2B+1) [应用式(2)] -(-1)[(2k+1)-k]=(-1)*(k+1),右边=(-1)+1)-1(k+1)=(-1)*(飞十1), .左边=右边. 所以当见=+1时式(1)也成立. 根据1°和2°,可知式(1)对于所有的自然数2都成立。 注1.上面例1和例2在第2步的证明中,叙述方法虽然不同,但精神是一样的。一般来说,如果待证式子的右边比较简单,可以采用例1的叙述方法,待证式子的右边比较复杂,那末可以采用例2的叙述方法 2.例1和例2的证明里,在第2步都先提出了“假定待证式子当:=时成立”,这通常称为归纳法假定,下一步在证明当=k十1时待证式子也成立中必须用到这个假定, 必须注意,在应用数学归纳法近题的两个步骤中,第1个步骤是递推的基础,第2个步骡是递推的依据,缺少任一步骤都是不行的。不然就会得出错误的结论 例如,果不考虑递推的依据,只险证了当=1时,甚至x=23,…,9时, ・f(c)=a2+c+11:. ·371-e ==========第382页========== 的值都是质数,就作出:对于所有的自然数c,f()的值都是质数的结论就错误了 又如,在例1里,如果不先验证:当n=1时, 1+3+5↓…+(2%-1)=22+1 这一等式是否正确,直接考虑第2个步骤: 假定这个等式当n=k时成立,就是1+3+5+…+(2k一1)=2中1,那末容易推出 14-3+5+…+(2一1)+(2k+1)=(2+1)+2k+1=(+1)2+1, 这个等式在2=中1时也成立.从而会作出错误的结论:“自然数列里前1个连续奇数的和等于2+1”. 例3 证明 18+23+…+%3=(1+2+…+%)3. (1) [证] 根据等差数列的前项的和的公式,可知 1+2+3+…+0=n(n+1) 2 所以,本题只需证明 13+28+33++0=[m]. (2) 1°当m=1时,式(2)显然成立,2°假定当=k时,式(2)成立,即 1+243+…+-[〔生D], (3) 那末,当%=k+1时,式(2)的 左边=(13+28+3十…+8)+(6+1)3 [リ+e+) 应用式(3)] =(+1)2[+4(k+101 =++了, 右边-[+4+],左边=右边 ◆872● ==========第383页========== 所以当%=k+1时式(2)成立. 根据1°和2°,可知式(2)对于所有的自然数儿都成立,从而可知式(1)对于所有的自然数也都成立. 习题 应用数学归纳法,证明以下各题: 85 1.1+2+3-+…+2=2(m+1) (2) 2 2.1+8+9+…+31=31 2 3.12+2.334+…+(u+1)=2(n+1)(m+2). 4.1.23234+345++(z+1)(%+2) =子w+1)02)0a+-3》. 5.1-2+4-8+16-…+(-1)n-12m1=(-1)m-12+3 6.+++でa-1)2+)1 7.首项是1、公差是d的等差数列: (1)通项是am=1+(一1)d; (②)前项的和是Sn=a+2(一1)&,2 8.首项是(a1+0)、公比是9(g÷1)的等比数列: (1)通项是an-a1qn-1; (2)前n项的和是8n=a1(1-g) 1-9 9.自然数列里,前个自然数的平方和: S%-금(n+1)(3+1) n2 10.如果-个数列的通项是2-(2+那末这个数列前” 2(m+1) 项的和是22+1)· 3.数学归纳法的应用 在数学里一些对于所有自然数(或者从某一个自然数起的所有自然数)都能成立的命题,常常要应用数学归纳法来证明。下面再举儿个例子。 ·373● ==========第384页========== 例4 用数学归纳法证明,若%是一个正整数,则"一3能被+y整除 [证明] (1)当=1时,x2-y2=(x+y)(一y),显然能被x十y整除. (2)假设当n=k(飞是任意自然数)时,x一y*能被 十y整除。那末,当北=k+1时, 2c9(t+1)-y2k+1)=x3.x2一y2y2k 33—r3g+2・y*ーy2・gy8h=x2(x2-y26)+y2*(x2-y). 因为2一y与2一y都能被x+y整除,所以它们的和x2(x2-y2)十y2(x2一y)也能被x十y整除.这就是说,x3(k+1)一y2+1)能被x十)整除. 根据(1)和(2),命题成立. 例5 平面上有儿条直线,其中任何两条不平行,任何三条不过同一点.证明这条直线把平面分成 f6)-号(m2+n+2) 个部分. [证明] (1)当n=1时,直线把平面分成两部分(为了简单起见,我们就说分成两块),即 ④-壹1+1+2)-2. 因此,乳=1时,命题成立。 (2)假定乳=k时, f()-是(+k+2)成立。如果增加一条直线(图8·7中的虚线),这条直线必与前条直线交于k个交点,这 尾个点把增的这条直线分成::: 图8.7 74。 ==========第385页========== k+1段,每段把它所在的原有平面块分城两块。图此,这时平面块的总数增攤了k+1,即 f(k+1)=f()+(+1) 是(+2)++) 受[(+コ4(+1)+211 这就是说,f@一是+n+2)对于n一k+1成立根据(1)和(2)可知,这样的%条直线把平面分成 了o)-号3++2到 部分.命题成立。例6 求证 (八, ,(1) 这里a>0,b>0,n是自然数.[证] 当n=1时然有낮-으,2 这时式(①)中等 号成立 当=2时,式(1)的左、右端分别为: 左边+3 2,右边-(生八. 这时,因为 -(告y-4沙-4418 a2+-b2 2 4 -2-2b+6-a-b)2>0, 4 4 () 所以式(1)当%=2时也成立. :·2°假设式()在=(化产)时减立,晔 ·75● ==========第386页========== (. (2) 因为题设0,6>0,所以告0,在式g的两边同 乘以a+b 6,得 ()()(), 就是 ak+1abtbak+*+ () (3) 4 a'+コ+b+ユ_a'++ab+ba*+b+12 4 =a*1-ba+b*+1-a6 4 (a-8)a*-(a-8)br 4 =(a-b)(a-b Y (4) 这里,因为a,b都是正数,所以 当a>b时,有a>b,a一b,a一都是正数;当a0 4 又,当ab时,显然有Q-)0-)=0.由此可知 4 (巴生)()(. 这就是说,当=k十1时,式()也成立 根据1°和2°,即可断言:原式对于任意的自然数%都成立. 〔注意) 1.第1步的证明虽然比较简单,但是也应注意,如果 0376 ==========第387页========== 只验证乳一1时的情况,只能说明式(1)的等号成立,所以还必须验证%2时的情况, 2.第2步的证明中,如直接比较12与2 a.-+bR+1 2}的大小,是比较困难的,所以采用了在承认(1) 式的前提下,两边同乘以一个正激生(注意:在这一步中必筑证明士>0),这样,不等式的有边变成〔告的1就与所要证的一致,留下来只需证明。++ 2一不小于 (士)(士)就可以了.由此可以看出,应用数学归纳 法证明问题时,要善于应用归纳法假定,作合理的推导。同时,在叙述上也不宜拘泥于一格 3.对门子这个例题,读者还可考虑一下: ()如果>0,>0a≠b,%为自然数,那末a+b”2 与(“)”粥有怎样的关系? (2)如果题设条件改为a<0,b<0,n为自然数,这个不等式是否仍能成立?如果缺掉了a>0,b>0这一条件呢? 例7 求证:当%是1,或是不小于5的自然数时,总有 2n>22. (1) [审题] 第2步证明中,需要从(归纳法假定)2*>k2(≥5)推出 2k+1>(k+1)8, 从归纳法假定可知2+1>22.因此,要证明不等式2+1>(k+1)2成立,只需证明2k2>(k+1)”,即 k2>2k+1, 1 即k>2+方 ●877· ==========第388页========== 而当≥5时,2+是<8,所以>2+是显然成立 [证] 1°当n=1时,左边=2,右边=1,左边>右边,式(1)成立.当%=5时,左边=25=32,右边=5=25,左边>右边,式(1)也成立. 2°假设式(1)当%=k(k≥5)时成立,即 2*>2(≥5). (2) (2)式两边各乘以2,得 2k+1>2k2 (8) 从已知条件≥5,可推得2+是<8,从雨有 2+是 两边同乘以飞,得 k2>2k-+1, 两边再同加上2,得 2k>(k+1)2. (4) 由式(3)和(4)得 2+1>(k十1)2 这就是说:当1=无+1(k≥5)时,式(1)也成立. 根据1°和2°,可以断言:当n=1,或者%是不小于5的自然数时,式(1)是恒成立的. 1.用数学归纳法证明: 8.5 (1)当n为正奇数时,”+y能被x+y整除; ③) (②)当n为正整数时,%3152能被6整除. 2.证明:三个连续自然数的立方的和能被9整除, 3.用数学归纳法证明: 方+方+. 4.有2个圆,其中每两个圆相交于两点,并且每三个圆不相交于同 一点,求证这”个圆把平面分成2一》十2个部分。 0878· ==========第389页========== §8.6数列前n项的和 前面,我们曾学习过求等差数列、等比数列的前项和的方法,并导出了两个常用的求和公式: (1)a1+(am1+d)+(a1+2d)+…+[a1+(k-1)d] +…+[a1+(%-1)d] =n2a1+(-1)d 2 (2) a1+a19十a1q2+…+a1g-1+…+aq-1 a4(1-g)(q≠1). =1-9 上面这两个公式中,“=”号左边的式子,通常也把它叫做有穷级数,并把其中的第飞项[(1)中的a十(一1)d, (2)中的a1g-]叫做这一级数的通项.为了写法上的方便,我们常用符号∑(读做sgma)来表示和.应用这一符号,上面这两个公式就可以分别写成 ()S-2[a+(k-1)=n2+g-122 (2)Sn=含ag*-4=g2g≠1). 1一g 注 对于一个有穷级数,我们也可以把它的最后一项(第”项)作为通项,并在它的前面加上一个符号2来表示.例如上面的(1)可以表示成41+(一1)],(2)可以表示成a1q-1.一般的,符号∑an就表示 ∑an=a1+ag+ag十…+an. 这样,记法就更简单了. 求一个有穷级数的和,在解某些实际问题中以及今后学习高等数学中都很有用."下面介绍解决这类问题的儿种常用方法。 ◆879● ==========第390页========== 1.应用拆项法 我们先来看下面的例了求和: 는++品1 十十%(n+1) 首先,把通项1 n(%+)拆成两个分式的差,得 11 m(n+1)nn+1 (1) 这样原式就可以变形为: 모(+y(一)+(-) +()·…+()(2) 容易看出,在式(2)中除去第1项1以及最后一项1 %+1以外,中间的相邻两项互为相反数,恰巧能够抵消.由此就可求得 1규 21 +1n+1· 这种求和方法叫做拆项法.一般的,如果一个数列的通项am能够分拆成关于自变数%和n+1的同一函数的两项的差,即 an=f(n)-f(n+1), 那未就有 Σa=∑[f(n)-f(+1)] =[f(1)-f(2)]+[f(2)-f(3)]+…+[f(n)-f(n+1)] =j(1)-f(+1), 088月· ==========第391页========== 从而就可求得它的前%项的和例1 求下式的和: 1 1 1.3.5357r57.9 1 +…+2m-)(2m+)(2m+3' [解] 这里 1 1 1 am(2%-1)(2m+1(2m+3)-2m+1`(2m-)(2m+3)s1.-1[1-1_] 42m+12m-i-2n+3J 1f1 ②n-02m+可②n+或2n+],Σ%-(-등)-() -[on-可a+y+ats]}1 -引品g+a+5}1 4n2+8m+3-3 m(n+2) =12(2n+1)(2m+3)-3(2m+1(2n+3)· 2.·应用自然数中前儿个数的正整数次幂的和的公式 应用等差数列前忆项和的公式,容易求得自然数列中前2个数的和是 2m-卫. (1) 现在我们来求自然数列中前几个数的平方和 ∑n2=12+22+32+…+n2. 这个问题.不能直接用拆项的方法来解决。但是注意 ·3810 ==========第392页========== 到(1)式中和∑饥是2的二次式,就可启发我们:所求的和∑2很可能是的三次式.这样也就能想到可以利用下面的恒等式: (+1)3-923=3m2+32+1. (2) 在(2)中,分别令%=1,2,3,…,2代入,得2个等式: 23-18=3.12+3.1+1, 33-23=3.22+32+1, 43-33=3.32+33+1, (n+1)3-n3=3n2-+3n+1. 把这2个等式两边分别相加,化简后即得 (2+1)3-1=-3.∑n2-+3∑n+%. 由此应用∑%的公式,即可求得: 2号[m+)3-1-3.士1-n]2 、중[2n"+-6n2+ 6in -82-n-2]=6(2n2+3n+1)=(n+1)(2m+1)6 这样也就导出了自然数列里前?个数的平方和公式 Σn(靴+1)(2n+1) 6 应用类似的方法,可以导出自然数列里前%个数的立方和的公式: m3-景m+ 应用这些公式,就可以求出通项用的多项式来表示的级数的和 例2:·把半径相同的小球,堆成一个三棱锥形的堆垛,它的最 上一层只有1个小球,下面一层有1+2个小球,再下一层有1+2+3个小球,以下依次类推,堆垛共有10层.问这 ●2● ==========第393页========== 个堆垛共有多少个小球? [审题] 本题就是要求数列 1,12,1+2+3,1+2+3+4,…,1+2+3++10的和.这个数列的通项公式是 an=1+2十…十n=%(n+1) 2 所以只安先求出S,一公”里的公式,再用n=10代入 公式即得所求小球的个数.解] 堆垛每一层的小球数是 0:=1+2+3+…+%=n2+1D. .n层的堆垛,共有小球个数是 8。=Σn(+1)=2+n 2 2 -[(12i)+(23+2)++m2+1 -是[(12+2+…+m网)+(1+2+…+]是空+空0 =n1a+4时9]6 는na+)(n+1+ =号aa+2. 今m=10,S。言×10×11×12-20. 答:共有小球220个。: 注 从这个例子的解法中,可以发现和的符号“Σ”具有一个重要性质 Σ士心-宣公+Σ,一般的,可以证别(证明窗给谈者) 0383· ==========第394页========== (1)Cf()=C∑f(%),这里C为常数;(②)∑Cf(n)士g(n)门=Σf(2)±Σg().应用这些性质,以后解题时,中间步骤可以略去。 3.应用等差数列、等比数列的前项和的公式 有些级数,它的各项虽然不成等差数列或等比数列;但是经过适当的变形,往往可以利用等差数列或等比数列的求和公式来解,举例如下. 例3 求号+3子+5号+7+…前%项的和. [审题] 从给出的各项,可以看出各项的整数部分1,3,5,7 成等差数列,分数部分受,子合品皮等比数列,因此 按题意可知这个级数的通项公式应是 -2m-1+去 这样本题就可以把原级数分成两个部分,利用等差数列和等比数列的求和公式来解[解] s号++금+(ea-+ =[1+3+5+7+…+(2-1)] +引+++]1 1 (1+2m-1)·n+공2 =n8+1-1 n. 例4 求证:1+号+++…十3%2,3上4 9 [审题] 本题不等号左边的级数中各项的分子1,2,3,,n成等差数列,分母1,3,33,…,3”-1成等比数列。设这个 ●88" ==========第395页========== 级数的和是⑧,那末就可求得风.-号③是一个等比数列1+号++中3品与票的器,从而应用等比数列 的求和公式来解。 [证明] 设 9メ Sn=1十3+3 3中十3品, (1) 那末 8-+++·+-+景 (2) (1)-(2)得 。-(い+号+()+()·++(3)]-끓1 1-( 1、 (3) 3 8.-(1-)÷] =9-1 ー(+2ー)< 从上面的这些例子,可以看到解有穷级数的求和问题需要有一定的技巧、另外还得指出:并不是所有的有穷级数都能找到它的和的公式,例如级数 1+号++…+ 就找不到它的求和公式.注 各项的倒数成等差数列的级数称调和级数,上面这个级数是调和级数中最简单的一个。 习题 1,求以下数列前船项的和,已知: 86 (1)an=n2-I9 (2)an=2(+1). 2.求下列有穷级数的和: (1)12+32+52+…+(2%-1)2, ●385● ==========第396页========== (2)15+3.7+5.9+…+(22-1)(22+3) 3求证g++3 ”(b+2)4·. 4.求下列有穷级数的和: d是+g+红+12t可1 (2)13-2+32-49+…+(2+1)2 本章提要 1.数列 (①)定义:按照某种规定排列着的一连串的数叫做数列. (2)数列的通项:a=∫(%). (3)数列的分类: 有穷数列(n取1到某一个 自然数N为止) 按化可取值的范围来分 无穷数列(n取1开始的全体自然数) 有界数列(an}an)递减数列(an+i1) 2 9-1 .=[2+-1少边 =(1-9n)(g<1) 2 前项和的公式 1-9 $w=.9-a1 9-1 (q>1) a1-ang 1-4 (q<1) a,b的等落中项 a,b的等比中项 頊 A=a4b G=士Vb 3.数学归纳法应用数学归纳法证题的一般步骤是:1°先验证2取第一个自然数值时命题成立(这是递推的基础); 2°再作出归纳法假定:“设%=(k≥)时命题成立”,然后利用这个假定以证明:“当%=k十1时命题也成立”(这是递推的根据) 证实了这两点,就可作出结论:“对于从a开始的所有自然数乳,命题都成立”(此即是在1°的基础上利用2的结论进行递推). 4.数列前项的和的几种求法(们)应用拆项法: Σan-E[f()-f(+1)]=f(1)-f(n+1). ●38Te ==========第398页========== (2)应用自然数列中前%个数的正整数次幕的和的公式:1°∑n=n(n+1) 2 2°m2=n(n+1)(2m+1), 6 8”Σ-[nh卫. (3)应用等差数列、等比数列前%项的和的公式。 复习题八A 1.对于下列各种数列各举一个例子: (①)有穷数列; ()无穷数列; (3)递增数列; (4)递减数列; (5)摆动数列; (6)常数列; (7)有界数列; (8)无界数列; (9)递增有界数列; (1.0)递减有界数列; (11)递增无界数列; (12)递减无界数列, 2.一个数列的通项公式是: n2 an=-n2+1) (1)求证这个数列是递增数列;(②)求证这个数列是有界数列; (3)作出这个数列的图象(取%=1,2,3,4): (4)这个数列的第几项等于0.98? 3.(1)一个等差数列的第2项是一2,第4项是一8,求这个等差 数列的第6项; (②)一个等比数列的第2项是一2,第4项是一8,求这个等比 数列的第6项, 4.(1)一个等差数列前4项之和是30,公差是3,求它的第5项; (2)一个等比数列前4项之和是30,公比是3,求它的第5项。 5.(1)三个数成等差数列,它们的和是9,积是15,求这三个数;(②)三个数成等比数列,它们的和是7,积是8,求这三个数。 ◆088● ==========第399页========== 6已知以识中成等然数列,求证。。中。也 成等差数列 (2)已知a,b,c,d成等比数列,求证a+b,b+c,c+d也成等 比数列; (3)已知三个正数a,b,c成等比数列,求证lga,gb,lgc成等差数列 7.有四数,首三数成等差数列,末三数成等比数列,第一数与第四数的和是16,第二数与第三数的和是8,求这四数. 8.(1)证明恒等式: (n+1)2-n2=2m+1; (2)利用这个恒等式证明 1+2+3+…+2=%(n+1) 2 9.求前n项的和: (1)(a-1)+(a2-2)+(-3)+…+(an-),这里&+1; ②((e+)+(e+子)+(+)+…+(e+子) 这里x+1,y+1. 10.用数学归纳法,证明: (1)1+x+x2++00=1-+1 1-x(x≠1) (2)1+3+6+…+n(%+1)-n+1)(+2)2 6 ,1 1 (3)128+234+…+nm+1(n+2) n(n+3) =4(0+1(n+2) ④1-晋+++(-1少兰@--a-n+22! =(-1)n(c-1(x-2)…(-n) 2! 注 !=1·2.3…n,表示自然数列里前2个自然数的连乘积. 11.(1)证明:当≥4时,5>32+3k+1(是自然数): (2)要使不等式 2n>3 ·889· ==========第400页========== 成立,可取哪些自然数的值? [提示:n=1.是显然成立的,除此之外,应先通过直接检验,看 取哪-一个最小的自然数值才能使这个不等式成立,然后应用数学归纳法证明n取此这个自然数大的一切自然数都能使这个不等式成立.] 复习题八B 1.证明下列的命题 (1)如果a,b,c成等差数列,那末x2(b+c),b3(c+),c(d +b)也成等差数列; (2)知果a,b,c成等比数列,那末 a2+b+c2=(x+b+c)(x+b-c)5 (3)如果a,b,c的倒数成等差数列(a>c>b>0),那末 Ig(a+c),lg(a-c),1g(a+c-26) 也成等差数列; (4)如果b+c,c中-a,a+b的倒数成等差数列,那末a,的,c2也成等差数列; 上面这些命题的逆命题是不是也能成立? 2.已知a,b,c都是实数,证明以下的命题: (1)如果方程a(b-c)x+b(c一a)x+c(u-b)=0有等根,那末 %,b,c的倒数必成等差数列; (2)如果方程(a2+b)x2一2b(a+c)x+b2+c2=0有实数根,那 未方程的根必是等比数列a,b,c的公比, 3.从1.到200这些整数中,有多少个数是3的倍数?这些整数的 和是多少? 4.把自然数列里的数从1开始,依次取1个、2个、3个…进行分组: {1},{2,3},{4,5,6},. (①)求第%组里最小一个自然数的表达式: (2)求第10组里各个数的和. 5.一个等比数列共有10项,公比为2.、如果各项都取以2为底的对数,那末他们的和是25,求这个等比数列各项的和, ◆99● ==========第401页========== 6,已知三个正数,,成等比数列,他们的和是62,积是1000,其中最小的一个正数是x,求这三个数7,求下列数列前n项的和: a)1액166,2 32…; ®是品奇 1,35 (3)3127’24g’: (4,-2,3,-45 ~す,27,-81243 8.应用数学归纳法证明 (1)三个连续自然数的立方和能被9整除: 1 1 0+n中2+…+1 (2)>132+224 (n≥2). 第八章测验题 1.等差数列的第1项与第26项的比为1:21,前26项的和为140,求这个等差数列的第10项及前10项的和. 2.已知一个等比数列的各项都是正数,且满足条件 a7一(5=a6十45=48, 求这个等比数列的首项、公比及前10项的和, 3.有a,b,c,以,e五个数,其中前三数成等差数列,中间三数成等 比数列,最后三数的倒数也成等差数列,求证奇数项各数(%,G e)也成等比数列. 4.在12和2之间,插进两个数和y以后,前三个数成等差数列,后面三个数的常用对数也成等差数列,求插进的这两个数 5,已知一个数列: 1 1 1 1+√3・√3+√5・√5+√7 1 1 W7+V⑨?V9+Vi," 0881● ==========第402页========== (①)根据这些项的特征,写出它的通项公式; (2)求这一数列前%项的和; (3)用数学归纳法证明(2)所求得的结果, 6.用数学归纳法证明: 34n+2+52m+1 能被14整除,这里%是非负整数. ·392● ==========第403页========== “数列的极限 上一章里,我们主要研究了有穷数列的通项公式和前n项和的求法,这一章,我们将在这基础上学习关于无穷数列的一些知识,包括(1)当一个无穷数列的项数无限增加时.它的变化情况;(②)怎样求一个无穷数列所有项的和(如果这个和存在的话).这类问题叫做数列的极限问题,它是学习高等数学的主要基础知识之一, §91数列极限的意义 我们来看下面这些数列的特征: (1)数列0,是景草” 这个数列的图象如图 an 91所示 从图上可以看出,随着n的增大,表示一对实 an m 1 数(n,an)的点就逐渐向直线a=1接近,但是这 012845678 些点始终在直线an=1的下方 图91 通过计算可以知道,随着n的增大,差an一1的绝对值就逐渐减小,例如 当=1的时候, ·393● i: ==========第404页========== 4-1|={0-1=1, 乳=2的时候, 1g-1-号-1-受 n=6的时候, la-1-나용-=금 n=100的时候, 1aw-1--17.99 进-一步,还可发现:不管我们事先指定一个怎样小的正数ε,总能够找到这个数列的某一项,使在这一项以后, 数列的各项Q与常数1的差的绝对值都小于8.就是,当 n>N时,总有 am-1<6. 例如,如果我们事先给定了一个很小的正数0,郑 末从第101项起,数列的各项与1的差的绝对值就都小于1 100· 再如,如果我们事先给定了一个很小的正数000,从 a--"-괴- 容号看到,只要>100,那末是就小于do因此从第1001项起,数列的各项与1的差的绝对值就小于0· 一般的,设给定的这个很小的正数是ε,那末从 a-피-"π--< ●894● ==========第405页========== 就可求出,只要m>是,那未是就小于8,如果把子的整 数部分记作(例刻8-0.08,音-s一8.8,就取 N=33),那末从第N+1项起,数列各项与1的差的绝对值 就小于8. 我们把数列(1)的这一特征,简单地记做当见-→∞的时候, |a-1|-0 (读做当饥趋向于无穷大的时候,差an一1的绝对值趋向于0),并且说1是数列(1)的极限. 345.%+1 (②)数列2之,宫,4,,n一… 这个数列的图象 an 如图9·2所示 从这个图上可以看到,随着n的增大,表示一对实数(,an)的点就逐渐向直线a=1接近,但是这些点始终在直线an=1的上方. 通过计算可以看到,随着几的增大,差 图92 am一1的绝对值就逐渐减小,例如 当n=1的时候, a1-1|=|2-1|=1, n=2的时候 la--용-기- ●a08● ==========第406页========== %=3的时候, Ja-1-|-- m=6的时候, 1a-과=-피- %=100的时候, aw-1-1-- 进一步,还可发现:不管我们事前指定一个怎样小的正数8,总能找到这个数列的某一项av,使在这一项以后,数列的各项与常数1的差的绝对值都小于8,就是,当n>N时,总有 a4-1<8. 例如,如见事先给定正数0,那末数列从第101项起,各项与1的差的绝对值就小于等等. 我们把数列(2)的这一特征,记做当-→∞的时候, an-1→0. 并且说1是这个数列的极限, @数列1,-受君金,《-景… 这个数列的图象如图93所示 从这个图上可以看到,随着%的增大,表示一对实数(n,a)的点就逐渐向直线a=0(n轴)接近,但是这些点,根据见是奇数或者是偶数,时面在这条直线的上方,时面在这条直线的下方。 ●898◆ ==========第407页========== 2 图93 象上面的两个例子一样,从计算中可以看到:随着%的逐渐增大,am一0就逐渐减小;并且不管我们事先指定怎样小的一个正数e,总能找到数列的某一项aw,使在这一项以后,数列的各项an与常数0的差的绝对值都小于8, 就是当>N时,总有 an-이<ε。 我们把数列(3)的这一特征,记做当1-→∞的时候, |am-0->0 并且说0是这个数列的极限, 一般地,对于无穷数列 a1,g,a3,…,n,…, 如果存在一个这样的数A,不管我们事先指定怎样小的正 数E,总能找到数列的某一项aw,使从这一项以后数列的各项an与常数A的差的绝对值都小于6,就是,当%>N时,总有 |an-A<8, 那末就说A是这个无穷数列的极限. 数列a,a2,ag,…,a,…的极限是A,又常简单地记做 ●397· ==========第408页========== lim an=A① 1小co 读做“当%趋于∞时,的极限等于A”。 注 用我们上面用过的记号,数列极限的上述意义,可以简单地说成:如果当8-→o的时候,青an一A-0,那末 lim an=A.n-oo 例1 画出下面这个数列的图象,并且从图象上说明这些数列没有极限 (1)数列1,2,3,4,5,…,%,; (2)数列1,一1,1,-1,…,(-1)"41,…. [解] 这两个数列的图象如图94和图95所示。 an 图94 图9.5 在第1个数列里,当%愈来愈大的时候,=n也愈来愈大,点(n,am)并不向某一条平行于n轴的直线接近,所以这个数列没有极根 在第2个数列里,按照%是奇数或是偶数,a=(一1)”+1忽而是1忽而是一1,点(n,an)也并不向某一条平行于?轴的固定直线接近,所以这个数列也没有极限。例2 画出下面这两个数列的图象,并且从图象上说明这两个数列的极限都是0. ①1im是拉门字“imes”的缩写,就是极限的意思。在研究数列极限时, 1im记号下的n-o也可省略不写。 ●898◆ ==========第409页========== (1)数列1,导景 (2)数列是子寻,(侵》, [解] 这两个数列的图象如图96和图9·7所示. Qa c=(得》 an=1 72 2345 图9.6 图97 对数列(国),当∞时,是→0,就是是-0-0,所以克- 对于数列(2),当∞时,(合”0就是(侵)广-→0,所以m(》”-. 注 用同样的方法,我们可以说明当{g|<1的时候,imq”=0。 n-o0 例3 已知无穷数列 m (1)计算。引=2 (2)找出一个正整数N,使当n>N时,不等式 都能成立; (③)证明是这个无穷数列的极限。 ●399d ==========第410页========== [解] (1).·n是自然数,2m-1>0, ,1 4n-2. (2)要使 1 am-<1000成立,只要 11 4%-21000’ 4n-2>1000, 102-250号, 不妨取N=250,那末当%>N时,不等式就都能成立. (3)设ε是一个任意小的正数,由不等式 1 1 <8 4n-2 可解得 n>1+1 因此只要取比8+令大的一个整数N,那末当%>N 时,不等式a,一号<:成郑能成立. 根据定义,可知是这个数列的极限。 习题 1.一个无穷数列的通项公式是。=1-” 2% 91 (1)画出这个数列前5项的图象; ②计算a+号引 (3)对于下表给出的e,各找出一个对应的正整数N,使当 2>N时,不等式 a+< 总能成立? e 0.1 0.01 0.001 0.0001 N (4)对于任何预先给定的正数8,找出一个对应的正整数N, ●400· ==========第411页========== 使得当>N时,不等式 <8 总能成立; (5)确定这个数列的极限 2。已知一个无穷数列的通项公式是a“=(-1”,仿照上.题的 步骤,求lim an, [提示:先从图象上估计出这个数列的极限是什么,再计算.] 8.已知-个数列的通项公式是42,求证 2lim an=n-30 [提示:只要证明对于任意给定的正数,能够找到一个正整数 ,使得当a>N时,不等式4-引<成立】 §9·2关于数列极限的一些定理 在研究数列极限的时候,常常要用到下面这些定理,现在我们不加证明地加以采用① 定理1 如果一个数列有极限,那末它只能有一个极限 例如,由第5页的(②图,数列2,告票 n+1 ,…的极限只能是1. 定理2 如果一个数列是递增有界数列,或者是递减有界数列,那末它一定有极限 例如,§91里讨论过的数列(1)是递增有界数列,因为各项的值愈来愈大,但总不大于1,所以它有一个极限.同样,§9·1里讨论过的数列(2)是递减有界数列,因为各项的 ①这些定理的证明,超出了本书的范围,以后在学习高等数学的时候, 将会学到。 ·401● ==========第412页========== 值,愈来愈小,但总不小于1,所以它也有一个极限。 〔注意) 1.不是所有有极限的数列都是递增有界数列或递诚有界数列.例如§91里讨论过的数列(③)既不是递增数列,也不是递减数列,但它却是→个有极限的数列 2.有极限的数列,一定是有界数列,但有界数列却不 一定有极限,例如: 数列1,一1,1,一1,…,(-1)+1,… ·是一个有界数列,但它却没有极限 定理3 如果两个数列都有极限,就是 lim as=A,lim on =B,oa 那末,由这两个数列各对应项的和、差、积、商所组成的数列,也有极限,并且 lim (an+on)=lim an+lim on=A+B, n-so6 lim (an-on)=lim an-lim on=A-B,lim (anon)=lim an'lim on=A.B, n-00 n-+o3 liman 6脑0,“合,这里B+0.lim an= A -oo 例如: 数列 0, 的极限是1. 数列 2945,,%+1 1?2,宫4“, 的极限也是1, 以这两个数列各对应项的和作数列: -,출++후,"+,0+言+ %… 就是 2,2,2,2,…,2,…。 ●402· ==========第413页========== 很明显,它的极限是2,也就是原来这两个数列的极限的和。 以这两个数列各对应项的差作数列: -움-용-,,- 0、 就是 -2,-1,-是-2 2 3 当-→心的时候, --이- 所以它的极限是0,也就是原来这两个数列的极限的差。 以这两个数列各对应项的积作数列: 0早是告…,”,就是0, 22-132-142-1n3-1 28一,3,48…, 当-→∞的时候, 所以它的极限是1,也就是原来这两个数列的极限的积。 以这两个数列各对应项的商作数列: 123 m-1 0234 234 忆 就是 0,京景是 %-1 九十1,。 当乳-→c的时候, -1-1=0 2 n41 +1>0. 所以它的极限是1,也就是原来这两个数列的极限的商 例1 已知 ma安6,-子-oo ·408· ==========第414页========== 求: (1)lim(an+bn); (2)1im(a.-bw); 枚-+00 (3)lim(an·bn); (④i四。· [解] 因为四和m6,都荐在,且n6-号+0,所以n+0 可应用定理3,得 山四a)-06号+号-名-哈n-o 邦+内 6i (2)lim (an-6n)=lim an-lim on12.1 2ー3=-6 121 (3)lim(anb)=i.=立·名- 1 liman (④)imaa=””2 3 4→0的 四6。2 3 例2 求Iim化 29%+1 C解] lim —i加 n→02m+1 2+- m 因为1的极限就是1,面2的极限就是2.面工的极限是0,所以我们可以应用定理3,得 1 lim 1 m2n-2 n→o lim(2+) m 韩+0 1 11 1 Iim 24lim 2+0=2. 700 B→的亿 〔注意〕 因为1im%和1im(m+1)都不存在,所以如果不先把 ·4040 ==========第415页========== 1 2m+1化做 就不能用定理3来解。 2 m 习题 1.把等于1个单位长的线段0S用A1点平分,再把A18用A,点 92 平分,把A2S用A8点平分,这样无限地进行下去, (1)写出线段0A1,0A2,OA3,…的长组成的无穷数列; (2②)应用定理2证明这个数列必定有极限, 2.i如い号求 (1)lim(an+Un); (2)lim(an-n); 1100 n→% (3)lim anUn; (4)im n→ (5)Iim(2xn+3vn); h→o (层 (7)im3g品+2xn-1 (8)]im x员. n-a 3n+1 3.已知无穷数列 51015 52 3’452十2’…, 123 'n+2・ (1)求这两个数列的极限; (②)作出每一项等于这两个数列的对应项的和的数列,并验证这个数列的极限等于原来两个数列极限的和. §9·3无穷递缩等比数列 我们来考察下面的问题: 图98的△ABC是一个等腰直角三角形,它的一条 直角边AC的长是1个长度单位.从C点作AB的垂直线 交AB于D,得到一个等腰直角三角形BCD,设它的面积 是S1个平方单位;再从D点作AC的垂线,得到一个等腰 直角三角形CD,设它的面积是S个平方单位.用同样 的方法作等腰直角三角形EDF,FG,GF丑,…,它们的 ·405· ==========第416页========== 面积分别是S8,S4,S6,…平方单位.现在来求这些三角 形面积的和的极限. 从图中可以直观地看出,当 H 这种三角形愈来愈多的时候,它 F 84 们面积的和就接近于原来这个等 D 腰直角三角形ABO的面积,就 S2 是京个平方单位. 现在,我们来证明这个事实, 图98 从平面儿何知道△BCD≌△CDA,所以△BDC的面 积是△4B0国积的一单,由此可知S:-子 同可知s,8-는8~面 所以这些三角形的面积的量数组成无穷等比数列 11111 4’8116132,64,… (1) 这个数列的第1项~是公比g-子,它的通项是 侵》1 现在的问题就是要求数列(①)所有项的和,根据等比数列前儿项的和的公式,我们知道 。491--侵11一g 1 这样,我们的问题也就是要求当饥-→∞的时候8的极限,就是求 im&-i[1-(侵》”]. 因为当m→o的时候(侵)》”→0,就是1i(安)”=0, ·408● ==========第417页========== 所以,我们可以应用上节的定理3,得 lin-Hm受[ュー(受)]ー受m1-()]1-lim()]--이]=受 这就是所要证明的 上面这个数列(1)的公比q的绝对值q小于1.我们把这样的数列叫做无穷递缩等比数列.当%无限增大的时候,无穷递缩等比数列前n项的和的极限,我们就叫做这个无穷递缩等比数列的和.一般地说: 设在无穷等比数列 a1,a19,1q2,…,a1q-1,… 中,g<1,这个数列就叫做无穷递缩等比数列.当n无穷增大的时候,它的前%项的和 =(1-g") 1-9 的极限 lim s-lim (1-9") 花→的 n→00 1-9 就叫做这个无穷递缩等比数列各项的和,记做 s-limsn=lim a(1-9") 00 *81一g 因为当q<1的时候,1imq"=0,所以 的 -m=g2-g】191 1-9 2은g-은,· =1-g-1-g0 这就是说,第1项是a,公比是q(q|<1)的无穷等比数列各项的和是 8=1-g(q<1). 这个公式叫做无穷递缩等比数列各项的和的公式, ●0Y· ==========第418页========== 例1 求无穷递缩等比数列 8,1,3… 各项的和 [解] g~를 1=3 9.8=1-g32 =4 1 3 例2 求无穷递缩等比数列 0.9,0.09,0.009,… 各项的和,并且求这个和与这个等比数列前5项的和的差。[解] 这里g=0.09.1 0.910 0.90.9 1-91-101 =1. 0.9 又 s6=0.9+0.0940.009+0.0009+0.00009=0.99999 .∴.8-$6=1-0.99999=0.00001. 习题 1.判别下列各无穷数列中哪些是无穷递缩等比数列,哪些不是: 93 (1)4,39. 3,1,星6 (2)10,9,8,7, (를)3,-1-공, ,2,144 3.222 (5)63,13,15,75;(6)V2・Vる3Vる w品”√3+1' (8)1,一x,2,-x3,…(<1); oa,a品,a,…(>11 1 a0,-是京,-,-<. ·408· ==========第419页========== 2.求上题中各无穷递缩等比数列的和, 3.求下列无穷递缩等比数列的和,并且求这个和与这个等比数列前3项的和的差: (1)0.1,0.01,0.001, (2)0.12,0.0012,0.000012,…。 4.在无穷递缩等比数列里: (1)a1=66,8=110,求q; (阅=144g=。求a: 3)4=0.4,93号,求a:(国=1导,4-号,求引2 2 (5)2=162,s=729,求a1;(6)4=334)8=36,求a1. 5.在每边等于a的正方形里,用连接各边中点的方法,作一个内接正方形,在这个正方形里再用同样的方法作新的内接正方形,这样无限地继续下去、 (1)求这些正方形(包括原来的正方 形)周长的和的极限; (2)求这些正方形(包括原来的正方形)面积的和的极限 6.一个皮球,由高处落下,每次跳起的高 (第5题) 度是原来下落距离的号,如果皮球由6.0米高的地方落下,到 停在地面上为止,这个球上下运动的距离的总和是多少米? §9·4化循环小数为分数 在算术里我们已经知道,要把分数化成小数,只要把分数的分子除以分母,但是,如果原来这个分数经约简以后,分母含2和5以外的因数,那末化得的结果是无限循环小数.例如 1 3=0.333…=0.3, 合=0.1666=0.16 ●409◆ ==========第420页========== 等等 现在,我们来研究相反的问题:怎样把一个已知的循环小数化成分数. 1. 化纯循环小数为分数 例如,我们要把纯循环小数0.3化做分数,可以先把它写成无限小数的形式 0.3=0.3333… 这里等号右边的0.3333…还可以用和的形式表示成 0.3=0.3383…=3+3+3 3 10+100+1000+10000+…. 容易看出,这里的各项组成首项是公比是的无穷 递缩等比数列: 름듬름(름):()… 应用无穷递缩等比数列各项的和的公式,就可以得到 3 10 331 S1-9之工10-193· 10 由此,就得到 0.3-1 例1 化纯循环小数0.16为分数. 解] 0.16=0.161616… 16,16 16 =100+10000+1000000+… -16+16.1+16/.1)3100+00`100+100(00+… 16 10016 99· 100 ●41g· ==========第421页========== 从上面所举的例子可以看出:纯循环小数可以化威个分数,这个分数的分子就是一个循环节的数字所组成的数,分母的各位数字都是9,9的个数和一个循环节的位数相同, 例2 化0.123,0.142857成分数. [解] (1)0.128=123 41 999=333 (2)0.142857=1428571 999999=7· 读者可以把41化成小数来验算.) 333,7 2.化混循环小数成分数 例如我们要把混循环小数0.16化成分数,我们把 0.16改写成 010-0.1608六+78o+品0 6 +0000+. 这里等号右边的式子,从第2项起,组成无穷递缩等比数列 8,808(品)》.66.16 这个无穷递缩等比数列各项的和是 6 8=10 6 1一1 10 所以 016-듬+듦-+으~90 90 151 0=6 例8 化循环小数0.223成分数 ●411● ==========第422页========== C解] 0.223=0.2232323.… 2 23 23 23 6+1000+100000+1000000+.…品++2 23,231 000100 +8(+门23 23 2 1000 2 10+、 23 11 10+990 100 2(100-1)+23=223-2221 990 990 990· 从上面的例子可以看出:混循环小数可以化成一个分数,这个分数的分子就是小数第二个循环节前面的数字组成的数减去不循环部分的数字所组成的数的差,分母的头几位数字是9,末几位数字是0,9的个数和一个循环节的位数相同,0的个数和不循环部分的位数相同, 例4 化0.123,0.023为分数 [解] (1)0.123=123-1211137 900 900300 (2)0.023=23-023 990990· 读者可以把3723 300’990化成小数来验算. 习题 1.把下列循环小数写成无穷数列各项和的形式,再利用无穷递绵 9·4 等比数列各项和的公式把它化成分数: (1)0.4; (2)0.523; (3)0.57: (4)15.423; (5)0.1444; (6)3.333… 2.利用循环小数化分数的法则直接把下列各循环小数化成分数,然后把所得的分数化成循环小数进行检验: (1)0.131313 (2)0.13888… ●412● ==========第423页========== 3,先把下列各式里的循环小数化成分数,然后进行计算(结果仍细写成小数的形式): (1)0.23+0.23; (2)0.54-0.54; (3)0.48×0.48; (4)0.36÷0.036 §95无穷数列各项的和 在§9·3里,我们曾应用求数列极限的方法,找出了求无穷递缩等比数列各项和的公式. 对于一个一般的无穷数列 a1,Cg,a3,",☑n,"g (1) 它的前1频、2项、…、%项、…的和 S1,S,Sa,…,Sm,… (2) 也构成一个数列,如果 lim S.=lim(a1+a3+.+an)=A 存在,那末我们就把A叫做无穷数列{a}各项的和. 表达式a1十ag十ag+…+am十… 也称无穷级数,当]imSn=A存在时,这个级数叫做收敛缓 数,A叫做这个无穷级数的和. 但是并不是所有的无穷级数都有和,例如无穷级数 1-1+1-1+1-1+5 1+2+3+…+%+… 等等的和都是不存在的.这类级数都叫做发散级数,下面我们举例说明怎样求某些无穷级数的和。 例 求和 8=1+11 1228+84+…+m0++… 0413● ==========第424页========== [解] 1·=1-1 2(n+1)%%+1 s-n(号)+(를一) S =lim S.=lim=1。 n→00 =lim 1 1+1 从这个例子可以看出,求无穷级数的和,一般有以下的步骤 (1)求出和Sn=a1+ag+…+a的公式; (2)再求Sn的极限1imSm=A.如果这个极限存在, 那末A就是所求的和.否则这个级数就是发散的,它的和 、不存在 因为并不是所有的数列都能找到它的前儿项和的公式,所以应用这种方法,我们只能求某些特殊的有穷级数的和. 习题 1. 已知无穷效列,”1 95 ()根据写出的各项,求通项公式a; (2)求这-一数列前h项的和8x;(3)求im8. 特 2.求下列各无穷级数前项的和及所有项的和: d급+++·++)の+z+ (2)名+品+多++公+, 本章提要 1,数列的极限 (1)定义:设数列a,,a…,A是常数,对于 0414 ==========第425页========== 事先指定的不管怎样小的正数8,总存在这样的一项W, 使得当n>N时,恒有下式成立: |an-A<8, 则称数列有极限,这个极限是A. (2)记法:当n-→∞时, am-A→0, 或 lim an=A(也可简单地记做liman-A)。 (3)定理: (1)一个数列如果有极限,那末它只能有一个极限.(ⅱ)递增(递减)有界数列一定有极限.(iii)如果imam=A,lim on=B,那末 lim(an+0n)=lim an+lim on, ∞ lim(an-on)=lim an-lim on H-+00 lim aon=lim anlim on, ∞ 1-+0 liman lim an= (1imbn≠0). on limon n-oo 2.无穷递缩等比数列 (1)定义:无穷等比数列 a1,19,a192,…,a19m-1, 中,如果|g<1,这个数列叫做无穷递缩等比数列. (2)无穷递缩等比数列各项的和: s=lim sn=1g 8.无穷数列各项的和 (1)定义:如果1imSn=lim(a1+a2十+an)=A存 12-0 在,则称S=inSn=A为无穷数列a十ag十…十a十…的和。 0415· ==========第426页========== (2)求法:1°先求出Sn=1+ag+…+am; 2°再求极限limS. 复习题九A 1,下面的命题是不是正确?如果不正确,举出一个反例来驳斥:()一个无穷数列如果有极限,它一定是一个有界数列; (2)一个无穷数列如果是有界数列,这个数列一定有极限: (3)一个无穷数列如果是递增数列(或递减数列),它就不可能 有极限; (4)一个无穷数列如果是递增〔或递减)有界数列,那末它就一 定有极限; (⑤)一个无穷数列如果有极限,它必须是递增有界数列或递减 有界数列; (6)有穷数列都有极限, 2.判断下面这些数列中哪些有极限,哪些没有极限: の1공1· (2)1,3,5,…,(2n-1),…; (3)1,22,32,…,2; ④1-01-品,1-是w1- 2y… 3.求满足下列条件的最小正整数九: 부-s19~(2)2+3 <10-3. 4.已知一个无穷数列: 0.3,0.33,0.333,…,0.333…3,…. n个 (①)求证:这个无穷数列的通项可以表示成 (2)计算:2-号: 0416· ==========第427页========== (3)对于下表所示的e,各找出一个对应的V,使得当n>N时,必有a-3<8. 1 0.01 0.001 0.0001 任意正数 V (4)根据上面求出的结果,可知这个数列的极限是什么? 5.求下列各极限: (1)1im323-4n2+2 713+5n-33 n-0 1+为国照(n-oo 6.求无穷项的和: ++++; 号++++는+1 7,求证:8x1+21+3g2+…+g-1=1-90 -(1-q)产-1-g(g≠1) [提示:仿照求等比数列前项的和的公式的方法,作出等式 9S=q+2q+3q3+…+ng",再设法化简.] 8.求下列各极限: 1+2+3+…+% (1)im1+3+5+千(2n-i5n-)co (2)1im12+22+3+…+%2 1c0 [提示:先把分子化简.] 9.在每边等于4的等边三角形里,用连接各边中点的方法,作一个 (第9题) (第10题) ·417• ==========第428页========== 内接三角形,在这个三角形里,再用同样的方法作新的内接三角形;这样无限地继续下去 (I)求这些三角形(包括原来这个三角形)的周长的极限; (②)求这些三角形(包括原来这个三角形)的面积的和的极限. 10.在半径等于比的圆里,作一个内接正方形,在这个正方形里作内 切圆,再在这个圆里作内接正方形,这样无限地继续下去 (1)求这些圆的面积的和的极限; (2)求这些正方形面积的和的极限, 复习题九B 1,应用数列极限的定义证明: (1)lim=1} o (2)img=0,这里|g1<1. [提示:gn}0,b>0); (2)lim cos"-sinn nc0sn0+Binn日 (0≤9≤〉 4.i已知数列{k}的前5项是 1×5,2×11,3×17,4×92,5×145 (①)写出这个数列的一个(最简单的)通项公式; 四泉严房是 5.已知√a+36+1a+2b-3到=0,求无穷等比数列b,4,号,…的和 6.已知sin9+cos日=√②,0为锐角,求无穷级数 04180 ==========第429页========== 1+8in0+sin20+...+gin+.. 的和 7.已知函数y=√x的定义域是0<<1 (1)画出它的图象; (2)在图象上取点P1,P,,Pk,,P-1,使它们的横坐标 顺次是克号一,会,2是,并分别与点4,0)连 接得线段A亚,A亚2,…,APk,,APn-1.求证 s=网-[%少+a-少2a-D]12 (3)求imS-2 12 第九章测验题 1。应用数列极限的定义,证明无穷数列 1234 3'57す 的限是子, 2.求下列各式的极限: (1)lim 1+3+5+…+(2B一1) 1+2+3十…+2 (2)1im(v2-+1-V%). 3.已知无穷等比数列里: 1)前三项是y3+1,1,-1 W3-1 V3+1求各项的和; (②)前四项的和是3是,各项的和是36,求首项及公比, 4.算下列各式(答鉴用分数表示): (1)0.12+0.12; (2)0.545-0.545 5.在半径为R的圆里,作内接正方形;在这个正方形里作内切圆. 再在这个圆里作内接正方形,在正方形里作内切圆,这样无限制地继续下去.求 (1)这些圆的周长的和的极限; (2)这些正方形的周长和的极限。 ●419● ==========第430页========== 总复习题A 1.(1)什么叫做绝对不等式? (2)证明2:c2+3c+4>0是一个绝对不等式; (3)不等式42-12.c+9>0是不是一个绝对不等式?为什么? 2.证明下面的不等式: ①)++c十a+a+b>6,这里a,b,c是不相等的正数; b (2)a+b+c≥√ab+V√c+√ca,这里a,b,c是正数. 3.已知a1+a2+…十an=0,求证 +a1+x+ag+..+x+an>nx 4.解不等式: 1)-5+6≥0: (2)2-3=469<1. 2x-1 x2+1 5.解不等式组: 〔9+4-11>-3 (1) 7 5 (2+x)2+8x2<(3x-1)2-12;[(x2-2+1)(x2+1)>0, (2) x2-4 贮2+4<0. 6.(1)设a,b,c都表示实数,求证方程: (x-4)(x-b)=c2 一定有实数根; (2)是什么实数的时候,方程: x2+2c-11+(x-3)=0 的根是实数? 7.已知方程组: 3x+2y=m十1, 4x+3y=m-1. (1)解这个方程组; (2)m是什么数的时候,x和y的符号相反? (3)是什么数的时候,x的值大于y的值? ●420· ==========第431页========== 8.解不等式: (1)√x2-2<2; (2)12-21<2. 9.求证: (1)正的真分数的分子和分母分别加上同一个正数,分数的值就增大; (2)正的假分数的分子和分母分别加上同一个正数,分数的值就减小。 10.(1)什么叫做函数的定义域? (2)确定下面这些函数的定义域(i)y=3x2+2:x+1; 1 (ⅱ)y=ーコ+1 x2+1 (iii)y=Vx-3+√5-; 2 (i)y=√+s+ (v)y=a六(w>0); (vi)y=log.(4x2-1)(a>0,a+1). 11.圆柱的体积是由公式V=π2h来决定的,这里V是圆柱的体积,是它的底面的半径,五是它的高. (①)在是常量的时候,V和h间的关系是什么关系?为什么? (②)在五是常量的时侯,V和底面积A(r2)间的关系是什么关系?为什么? (3)在V是常量的时候,A和h间的关系是什么关系?(④)在h是常量的时候,V和"间的关系是不是正比例关系?为什么? (⑤)在V是常量的时候,和h间的关系是不是反比例关系?为什么? (6)画出=1的时候,V随着h变化的图象;(T画出Y=6的时候,A随着h变化的图象; ⑧面出=是的时候,P随着,变化的图象 12.已知y=f(x)是一个二次函数,按照下列条件,确定这个函数,并且画出它 的图象 (1)f(-1)=0,f(1)=0,f(2)=-3; (2)函数的图象是以(一2,一1)为顶点的抛物线,并且这图象经过点(0,3); (3)在x=5的时候,函数有极大值2,在x-一5的时候,函数的值是-98; (4)函数的图象是对称于直线x一1=0的抛物线,并且这图象经过原点和点(-1,3). 13.(①)x取什么值的时候,二次函数 0421· ==========第432页========== y=2ac2-5c+2 的值是正数?是负数?等于零?有极大值或极小值?这些值是什么? (2)对于二次函数y=2-5x一2x2呢? 14.(1)什么叫做奇函数、偶函数?奇函数或者偶函数的图象各有什么特点? (2)确定下面这些函数,哪些是奇函数?哪些是偶函数?哪些既不是奇 函数也不是偶函数?(i)y=心+5 (i)y=V√x2+I; (iii)y=vx+1; (iv)y=2; (v)y=log21; (i)y=+/死, 15,(1)什么則叫做某一区间上的增函数、减函数?在某一区闻上的增函数或 者减函数的图象有什么特点? (2)证明函数y=x4在区间(-∞,0)上是减函数,在区间(0,+∞)上是 增函数 16.(1)什么叫做有界函数、有上界的函数、有下界的函数、无界函数?它们 的图象各有什么特点? (②证明函效y-云子是-个省界函致 17.(1)已知x为正数,求证函数彩≈c+2 的值不小于22. (2)这个函数是不是有界函数?为什么? 18.(1)什么口叫做单值函数、多值函数? (2)已知变量心和y之间有下面的关系: 42+y2-4+4y-4=0, 把y用的代数式来表示.这样得到的函数y=()是单值函数还是多值函数?如果是多值函数,把它改用一些单值函数来表示,并写出每一个单值函数的定义域和值域 19。(1)已知函数y=p(心)是函数y=f(x)的反函数,如果在同一直角坐标 系里画出它们的图象,这两个图象有什么关系?(②)求下面这些函数的反函数:(i)y=3x-5; (i)y=√花-4; (ii)y=/x+1; (iv)y=2w+1. 20.化简下列各式(答案不用负指数或分指数): 022● ==========第433页========== a-]g,a-b 其中a>0,b>0,a>b; 路》( (3)ユ-[(1+a3)}-ユ·(ユ+)-ユ。[(ユ+)ーァ(ユ+)J. 21.(1)作出函数y=22的图象; (2)i利用这个图象作出函数y=2;y=22;y=2-1的图象. 22.作出下列函数的图象: (1)y=-og2x; (2)y=og2(-z): (3》g=}g4 (4)y=log2lx1. 23,解不等式: の()*>(侵)" (2)2=2<8; (3)2l0gx0,a≠1)? (1)logax2=2l0ga (-) (2)loga+logay3=logay8; gax到-lgyl=ag.引 25,证明: (1)1bg%=loge a (2) logeb' loga N=1+-loga0; logat N 1ga(1g。a) (3)1Dg6m2n=0g6x; (4)a 108a=10goa. 26.求证:在不等式 logm a>logn a(m,n为大于1的正数) 中:(1)如果a>1,则mn. 27.不查对数表,解答下面的问题. (1)填写下面的表: N 1 2 6 7 8 9 10 Ig N 0.30100.4771 0.8451 (②)求下面这些数的常用对数: 0.428◆ ==========第434页========== 4.5;2.8;V12.5;(6.25)2, (3)20是儿位整数? (4)(0,2)10的小数点后连续有多少个零? 份的多少式深可人于10 (6)0.9的多少次幂可小于0.00001 (7):较(万)3和(⑨)10哪一个大? (8)求证0.01.· 0.001. 28.查表计算这里的数垌都是近似数): 1)78310.0:870.865 0.86:378 (2)-0.0931:10.042・ 29.(1)圆环的外宜径定22.5痈米,内直径是11.B厘米,求环形面 (2)二角形内切圆的半径和它的三边、b、c之间有下面的关系: (8-1(s-b)(s-r) 这里s是b.最s ≈15.4c1.b≈21.4m,c≈13.8c2, . 30.下列各题中的两个方程是不是同解方程: (1)g(2-1)#1.和1g(片1)+1g-1)=1; (2)ux和2=y(a>0,&≠1). 81.解方程: (1)4-52-tt=6;(2)3446安-2.9克; (3)(W/2+5)(Q-V3)=4 (4)g(1524x8)=3g(32);⑤(会+)g号-g网 (⑥号1g2-1)÷gV-9-1; *(7)1ogRr【l0g4花÷10g2=7;0(8)1g10e2+2)-1=lg龙.32。解方程组: 182-1.0y, (1) 2t=5; (2) y («>0,ッ>の了t+=y12, (10-)-3=10, (3) ígx-2)gy一)=0,x-y1=0; (④{ge-2》-leg-3)*0. ●424◆ ==========第435页========== 3.(1)什么叫做有穷数列、无穷数列、有界数列、无界数列、递增数列、递减 数列、常数列、摆动数列? (2)举出一个递增而有界的无穷数列的例子; (3)举出:一个递减而无界的无穷数列的例子, (4)个有穷数列,能不能是无界数列?为什么? 34.在直角坐标系里作出下面这些无穷数列一部分的图象,并且指出它们各 有什么特点,已知它们的通项公式分别是: ()。=2+元 8)a=(-1(2+是) ④4-2+(-1r是 35.能否有这样的三个数a1,a4,ag,它们既顺次地是等差数列的第1项,第2项,第3项,同时义顺次地是等比数列的第1项,第2项,第3项? 36.(1)等差数列的前2项的和是8n=2m2一32,求这个数列的前三项及通项 公式; [提示:S1=u1,S2=Q1十42.] (2)等差数列的前2项的和是8n=322一2,求它的第10项, [提示:a10=S10一Sg.] 37.一个等差数列共有10项,其中奇数项的和是12.5,偶数项的和是15,求 这个等差数列的第1项和公差 38.一个等比数列的前三项的和是3.5,前三项平方的和是5.25,求这个等比 数列的第1项和公比. 39.用数学归纳法证明: (1)12-+-+…+(3-3)32=2(623-1); (2),1 1.명+35+5-+·+2n-1)(2+1)n2 (n+1) 2(2%+1) “40.(1)一个无穷递缩等比数列的和等于9,而各项平方的和等于40.5,求这 个数列; (2)一个无穷递缩等比数列的第1项是1,第1项与第3项的和等于第 项的2弓倍,求这个无穷递缩等比数列的和。 41,计算(答案仍用无限小数表示): (1)0.444…↓0.2323; (2)0.31.22…-0.20101…. 42.已知是自然数,解不等式: ●425● ==========第436页========== (1)2m+1-2<0.001; n+1 (2)是引0.o, *(3)()-0<6.01. 43.已知-…个数列的通页公式是 ,小1 3%+2, (1)求证这个数列是递诚有界数列? (2)这个数列有没有极限?为什么?a安使k-子<0.0i,n至少是多少9 (4)求im÷1 77e3%+2· 44.已知为自然数, (1)解方程: 1+35+…(22一1)=169; (2)解不等式: 2+4+6+…+2>182; (3)求im1+3+5+…+(22-1) 7-+o0 2+4+6+…+2n 45.已知n为自然数 1)解方程: 1+2+4+÷2m-1=255: (2)解不等式: 1是+++(》 )求m1+受+星+…+(侵》门 总复习题B 1.已知全集1={1,2,3,4,5,6,7,8,9,10},有三个真子集 A={1,2,3,4}.B={3,4,5,6,7},C=1,3,5,7,9}, 验证集合的运算,具有以下的性质: (1)交换律:AUB=BUA,A∩B=BA; (2)结合律:(AUB)C=AU(BUC),(AnB)nC=An(Bn): ·4260 ==========第437页========== (3)分配律:(AUB)nC=(AnC)U(BnC): (AnB)UC=(AUC)n(BUC片 (4)吸收律:(AUB)nA=且,(AnB)UA=A;(⑤)德摩根律:A1B=AUB,AJB=An范 2,设(A)表示有限集A中所含元素的个数,应用上面这些集合,验证: (1)(AJB)=(且)+(B)n(A门B); (2)n(A JB JC)=n(A)+n(B)4-n(C)-n(AnB)-n(BnC) -n(CA)十(是nBnC), 3.设x∈N.用列举法写出下列各式的运算结果: (1)({c|c<7}」{x|c>}) {|>3}; (2)x∈[1,5](1,5). 4.班级中有48个学生,期中考试语文成绩不低于85分的有27人,数学成鲼不低于85分的有31人,其中两种成绩都不低于85分的有19人,问这次考试两种成绩都不满85分的有几人? 5.图书室借出甲种书35本,乙种书30本,丙种书25本。其中同时借甲乙两种,乙丙两种,甲丙两种的分别有12人、10人、8人,同时借这三种书的有5人.问向图书室借书的一共有多少人?6。证明下列不等式: (1)知如果x>0,y>0,那末 然+丝≥2 2-トyi (2)如果十b+c=1,那宋 a2+b2+c2≥2 (3)如果a>0,b>0,a+b=1,那末 (a+a-1)2+(6+b-1)2≥25 ¥ (4)知果:为不等于1的正数,那末 1g(÷1)+lg(a2+1)+lg(a3+1)>3g2a, 7.(1)设1,2,1,b2都是实数,证明 (a1b1+a2b2)2≤(a+a)(b+b2). 等号在什么时候成立? [提示:考虑方程(a+a)x2-2(ab!+a2b2)x+(b+)=0什么时候才有实数解.] (②)设a,b(i=1,2,…,n)都是实数,证明 ●427· ==========第438页========== (a1b1+a2b++abn)?≤(好哈++a)(b++…+b).等号在什么时候成立? (3)知果a,b,c,d都是正数,证明 ee1の()6 这个命题能不能广到2个数的情况? 8.已知为自然数,证订下列不等式: (②)立之++…+ a+1)c1; 安+…是 [提示:用(1),(②)证得的论.] 9.求下列各函数的记义城: (2)()ー() +log:V②-., 10.(1)证明不论是什2实数,函数 f(e〉=12+-13)2~2(+2)c+-3 的值总是正的; (2) 已ッ電,추特)を直ー1的下方。不与这 直线相交,求的取值范围. 11.在直径为风的半圆内,以直径为底作一内接等腰涕形, (1)写出梯形的周长y共腰x之间的画数关系式,及这一函数的定义域;(②)什么时候这个梯形的厢长有段大值?最大值是仆么? 12。(1)已知G是正数,求证: 生413. 等号在什么时候成立? b [示、4ー4が] (②)要制作一只体积是的圆柱形的杯子,怎样设 计,才能使应的树科最省? B 13.通过平行四边形ADCD的一个顶点C引直线交 (第13题) AB于E,交AD于1(见图).如果BC=a:DC=b.求1E+AF的最小值。 ●428· ==========第439页========== 这时BE,DF各长多少? 14.(①求函数y3知+√3+2的极值,这里公是不等于琴的常数, (2)求函数g-c+V√-x+i0:c一23a2(a+0)的最大值. 15.解下列方程组 (1) (x-3)+(y-1)言=(+y)克,(1g(x-6)+g(y-10)=1; |+”=y0, (2) (,y都是整数), ,y+9=x15 16.(1)求满足以下不等式的最小正整数n(g2=0.3010): 1+营+器+…+(》>396: (②)数列: 1g1000cos60°,lg1000cos260°,lg1000cos360°,…,1g1000cos"60°,… 的前多少项的和为最大?这个最大值是多少? 17.(1)已知△ABC的三条边a,b,c顺次成等比数列,公比为g,求证: V5-1<20, 3-2x (1)z-1:}x (2) (3c+1)(c4<0, W/4-x+1>0 2。求下列函数的定义域(答案用区间符号表示): 3x2-2x+1 (1)j(c)=lg3-2r-x ②a)-V+V+1 3+优 4, 8.已知a,b都是正数,求证: (1)V√a+VE≥Vab; (2)a+b+1的最小值是3 4.已知二次函数y=f(x)在x=一2时有极大值3,它的图象与x轴的两个交点间的距离等于√6. (1)写出这个二次函数的解析式,并画出它的图象; (2)写出它在单调递减区间里的反函数y=-1(c)的解析式、定义域和值域. 5.试就函数yー()的定义域、奇偶性、单调性、极值、值域等方面先作出讨论,然后画出它的图象6,已知有二个对数函数: y=f(x)=lcg(x2+2x-5)和y=中(x)=10g(2x2一3x+1),问x取什么值时:()f(x)=4(x); (2)f(x)>(c). 7.(1)求一切被4除后余数为1的二位数的和; (2)在2和5G之间插入怎样的两个数,才能使前面三个数的常用对数成 等差数列,后面三个数也成等差数列? 8、应用数学归纳法证明: マ+V(>1) 9,一个弹性球认100米高处自由落下,如果每次着地后可以弹回原来高度 的号润自出落下,问: ·431◆ ==========第442页========== (1)当球第5次着地时,它所经过的距离约多少米?(②)这个球能够几次经过离地面20米高度的地方? 10.求下列极限值: (1)1im1+3+5+…+(2-1) ns1+-4+7+…+(32-2) (2)im1小(1+2)+(1+2+3)+…+1+2+3+…+) n ++32+…+n2 ◆482· ==========第443页========== 习题答案 第一章 习题1.11.(1)2,3,5,7,11,13,17,19},(2){士1,±3,士5,±7,9}, (3)1,2,4,8},(4)2,3,5},(⑤){±2,±V2},(6){(x-1),(-4)};2.(1)-2,-1,0,1,(3)-2,-1,0,1,2;3.(1), (2)年,(3)∈,(4)∈;4.(④)3-4+1=,{得,② -2-1=0,{←1Y5. 习题1.2【.(1)元限集,(2)0,1,2},(3)中,(4){1]:2.(1)无限 兔@有限,两化,}表示 习题1.31.(1)G,(2)c,(③)∈,(4)c,(⑤C,(6)p,(7)=, (8);3.9,1,2},3},{1,2},{1,3},2,3},{12,3},有6个.习题1.41.(1)正方形,(2)等腰直角三角形,(3)等腰三角形,(4)不等于1的自然数;2.(1){1,2,3,4,5,6,7,8},(2)中(3){3,4,5,6, (4){1,2,3,4,G:3.(1)=,(2)=,(3),(4)c: 4,0){{-)-(월) ②{{*}=a包》 1.1.(1)AUB,(2)4nB,()4UBU(AnB),(4)4U(4n B);2.{无理数};3.{d,e,f,g},{,b,g},{g},{a,b,,e,f,g},{a,b,c,g,{a,b};4.{xx<3}={l-3<3}. 复习题一A 2.(1)gc0},(2)0∈{0},(3)AUA=1,(4)An五=: 4433● ==========第444页========== 3.(1) B (2) U B A A A B B B B 4.(1)n中 (2) U A 19 0 0 9 A 0 A A A A 5.(1){-2,-1,0,1,2},(2)1,2,3,4,5: 6.(1)U,(2)n,(3)C,(4). 复习题一B 1.出,1va,@2{g4 4.(1)15,13,2,30,32,(2)26,19,13,11. 第一章测验题 1.(1)∈,(2)丰,(3)c,(4)p,(c,(6)∈,(7)=,(8)U, (9)n,(10)U,n;2.(1)x<0或x>5,(2)0<<53.(1){1,2,3,4,5,7,9},(2),(3){1,2,3,4,5,6,8},(4){0,2,4,6,7,8,9},(5{0,2,4,6,7,8,9},(6){2,4; 4.(1){x2.x4-3x2+1=0}={±1,±V2/2}, ea-7+w-g1g”gg 第二章 习题2.11.前<后;2.前>后;3.前<后;4.当a+b时,前>后,当a=b时,前=后;5.前<后;7.(1)绝对,(2)条件,(3)条件, (4)矛盾. 习题2.22.(2)不一定,(3)不一定,(4)一定,(5)一定。 ●434· ==========第445页========== 习题2.日1.1>0时,<名(②a<0时,会同)a=0,b>0时, 6 3 全体实数,b≤0时,无解:2.(1)x>,(②r<-星,(3){实数, ④0,《同-号0<,3.1时,年呈,1 k-2 时,x<-k=1时,{实数},(②)>g时,xp+g:4仙星②是)是 习题341.山无解,(②)x<-子③)>1号④无解:2.dx>1,(2)-79,(7)x<2,(8)x>0,(9)无解,(10)x<0, 习题2.4(2)2.(1)x>a,(2)x2,(2)-3<ベーユ,(8)くー或 (4)<-或>2,(の)<4成>9,(6)-1<で8,(のE<1 成,(-()-V0ς0,(1)<ュ東>号 2.(1)aB. 习262)2()-号1,(3)-ases13 )용:3。()e3 、2或x六3+y店 3-13 ,(2)2-V2,(间(-∞,+∞),(⑥)-号<<3,の)2V ユ,(9)x+3,(10) 无解。 习题2.71.(1)-13,(2)x<-2,(3)x<-4或x>3,(4)-1< <山2.(-15,(2)x<1或3<6<5,(3)x<-2戈 -1<<1或>2,(4)-2√2,(2)<-2或-12 1 71 习题2.9【.(1)-3・ (4)-3a<움2。(1)1<<2,(2)-943.()z>1,(②)<,4.④5-2或-1<<3, (2)x<-4或一1<<1或x>3,(3)-3<然<一1或x>3,(4)x<-3或35时为正数,2后. 习题21山1.号<<圣2.<-行3.>多:4.无解:5.-11;7.x≥5;8.x≤-1或x>0.习题2.12(1)6.(1)w=b,c=d,(2)a=b=c=又. 习题2.131.(1)x√万时,有最小值a+2√万,(2)x=士√日时,有最 小值3,()=时,有最大值,国=15时,有最大值 2.(2②)不是:3.(山)=1时,有最小值x=-1时,有最大值3,(②)x=1时,有最大值圣,无最小值;4.10,10;5.x:y=2:1;6.土墙长4米,木板墙6米时,造价48a元为最省. 习题2.141)1①)<营或六是②)子<≤品,(<-是或5 >(4)こ4に<0或0<で;2。(1)-<<1成>1,(2)<1Y咸年,3.a0,网场4四-6G-京 ●436· ==========第447页========== (2)<2. 复习题二A 2.(1)>1时,前>后,,(2)>0时,前<后,3.b<1≥b时,b时,b-0,,2 8.(④1<<是,0<63:9.3t18,②54-2V5成42,③-心(の-ユ @无根,2后2VE;10-4,②22 1,(3)-√2<コV2,(4)y<2或y・3;11。(1)√<ェ<√ ,(份-号-ー12.4)ー号-2. 成>受,(21或で法13。 (1)2で8(2)1<8 14.(1)无解,(2)4;15.(1)∈,Ⅱ.n>8,(2)2X,且2>74; 16.(4)-]<に0或で,()ベー√或=0或>V17. (1)24,35,(2)86:18.有三解,10,59,11,63,12,67;19.(1) <1m>),(e)윤ーVG,()-,()-25 (3)号<<음,()8 x<4或x>12;2.(2)a=b,c=0;3.(1){0,1},(2){3,4,5,6 5.有两解(14尺,42尺)或(15尺,45尺). 第三章 习题3.13.(1){实数},(2)x≠1,(3)-1≤x≤1,(4)-12,(3)-1≤x<0或x>0,(4)g≤-1或x≥1◆438●· ==========第449页========== 5.(1),(2),(3)都是-3≤x≤3,(4)-31,(2)≥1,(3){实数},(4){实数}.9.(1)y 告e*0,②)y=8g2,③y=2V三,(国y=编2.(②=24,3)108开,(④4号. 复习题三B 1.(1①=2y,yeR,②)y=8-,xeR,(3)y=1-2,x<0或 x>0},(4)都不能;2.(1)(0,2]U[3,4),(2)-1,1)U(1,2)U(2,3];3.(1)0,(2)a+b;4.(1)定义域、值域都是R,(2)定义域 是ae<0或e>0,值城是{-3,3:5.y=5x+9(e*0: 6.(2)正比例,(3)反比例. 第三章测验题 1.y=,能,(2)y=±2√正,不能,(3)y=—√4s-1,能,( y:2.(2(-1 +)(2)[-23).(3)[2,U .1.((-,3=y):(y,34)(+,+) 3.(1)定义域{-3,-2,-1,0,1,2,3},值域0,√z,1,√3},(2)定义 城{-3,-2,-,0,1,值城信V5,V瓦,}:4.(1)正比例, @反监流,e)s国y-+会阅景 第四章 习题413.山骨品是-2-学2-号 3-2,(2)34,34,146, 14;5.0g=-号+营ey号4-京 习题431。(4)y=ー음*+()リーー+5。(3)vーー8 (4 v=+2。 v-+(2)v=ー,(8)ッにーー3 0439● ==========第450页========== ④y=号x+3,⑤y=V3+2,(⑥y=√3x-2V. 习题431.四-昌品,②青-4,0,乡之②)告是 (3)23.(1)7.5,-6.5,(2)2.5,3.5,(3)6,-6,(4)-1,1; 4.+2时有唯一解,1=2且1=2时有无穷多个解,1=但m1+2 2 时无解;5.(1)≠上1时有唯--解 挑十1, -=1时有无穷多个解,シミm+3m%+1 1 〔心k-i’ m=一1时无解,(2)k+上1时有唯一解{ =1时无解,k -k+1. k-1“ =-1时有无穷多个解;6.(1){(9,1),(4,4)},(2){(8,8)}.习题4.4(1)1.(1)≈20米,(2)≈2.9秒;2.(1)2.3,5.3,2.0,(②) ±1.4,±1.7,±2.1,(3)√52.2,V72.6. 习44(2)2(の)y=-(+)+(2"y=(G+-1()yー(-불)+,()ッー(い)アー;3.对称轴:(1)=1,e2)s=-1,(8)r,(4)-3,顶k:()(4,0),(2)(ー1,-8)5 (层盟》份(-3,-2,a:向上,⑧向上,向 上,(4)向下. 习厘4.51.项点坐标:(1)(3,-5),(2)(-1,8),(3)(1,-1):(4)(2,3),对称轴:(1)x=3,(2)=…1,(3)=1,(4)=2. 习题4.61.(1)y=然+3.÷2,(2)y=a25中4,(3)y=x2-10+23 2.(1)y=2x2-6x-4,(2)y=2x2+4-6. 习题4.7(1)1.(1)<3时减,x>3时增,(2)=3时,y极小=一5; 2<3时端,3时减,四)=3,然-3.(=寻g=0,园)=1,y-1,(倒e-音,y默=4路国,=130. 习题4.7(2)·1.P,2,R,S是各边的中i;2.800平方厘米;3.2500平 方米:4.)y-6-Y百品(心等边三角形边长约1,4米助. 习题4.81.1.6,-0.6;2.无解3.1.6,-3.2,4.1.6,-1.2. ●.440● ==========第451页========== 复习题四A 3.(3)y=r1,(4)k=1,a=45°,b=1;4.(2)(4,1),(8,9),(2,3); 5.÷+号-1,(2y-b=m(x- 6.y方x+10,10厘米; 7.(1)1y),(2)이-8):8.(4)下降16度,(2) -17,3是ku到1是km之同;10.(d)y=(e+2)-9,(2z= -5,x-1时y=0,x<-5,c>1时y>0,-5一2时增. 1. y=0时y>0时y<0时 y增加y减小 函数的表示式 极值时E时x x的值x的值x的值 的值的值 全体 x=一2 (1)y=(x+2)2+1 x>一2x<-2 实数 y极小=1 x<…2 化=-2 (2)y=(x+2)3 x=-2 x>-2 x<-2 x>-2 极小=0 x▣-3-3-1 无a1 <1x>1 y极大=8 国g=量g-2)2 c<2 x=2 W=2 然<2 x>2 x>2 y楼大口0 就=-1 y=-(+ー 全体 U极大 x<-1 x>-1 实数 1 2 12.<2,c>3时是止数,2<0<3时是负数,=2,如=3时是零,公=号5 时,有极小值一圣3.山面数值总是正的,“=圣时,2,.(2)函数谊在<-3,>子时是正的,-3<号时是负的,=-3,g=宁时等于至,当云-子,y=-6分,(③)函效值在3<2<4时是正的,<8,>4时是负的,”=3,-4时等了零,当=子时肤=子(4)函数值在<>时是负的,“=是时等于嘴,但不会是正数,当号时 ●4410 ==========第452页========== y酸大=0;14.(1)y=2c2-16+24,(2)y=-4x2+4+24;15.(1)y=3x2-36c+96,(2)y=-2c2+8c-156.(1)2.7,-0.7,(2)2.4, -0.9:17.(1)-4<<-2,251-yI或>1+V 2 2 (3){实数},(4)无解: 18.(1)y=20x-R2,(2)边长为10米的正方形时面积是100平方米为最大 复习题四B 1.平行于y轴的直线不能,平行于x轴的直线不是;3,(1)直线2x一y=3,直线2c一y=3上方(下方)的半平面,(3){(1,1),(1,2),(1,3),(2,1), (33), (3,1);:4.(1)计-1和ん时有一解(3欲一子・)飞=-1时有无穷多个解。=子时无解,(2)飞=-3时有唯一解(3,5), -8时天5。(1) 0,bに0(2)yー-如+3g2, 2a; 6.(3)11秒,5秒,176.4米;7。(1)ya大f(0=ユ,yトコツ小=f(2)=一3,无极大值,(2)y最小=f(2)=-3,无极小值,极大值,最大值,(3)y最小=9(0)=一1,y最大=y大=9(2)=3,无极小值,(4)无最大值、最小值、极大值、极小值;8.y=一x2+2c+8;9.约49分钟后最近距离约是12 浬;10.宽43+y②)≈2.52米,高22V-)≈0.52米 7 7 第四章测验题 1.(1)a=c=0但b≠0,(2)&=0但b+0,(3)8+0,(4)a>0,b=c=0, ()a<0,b<), a--0,2.()()-2+8,4)는-3+号 (2)(11,30),(3){1,2,3,…,10;3.y=-2x2+8a-5;4.(1)k=0 或k=4,(2)k<0或>4(3)04,(倒-是<<4,(④国公景⑤<营 6.四1=4+V-z+6,s=-Yx+V3,“(②◆=6cm, (8)大=√3cm2. ●42· ==========第453页========== 第五章 习题5.1(1)2.(1){-1,0,3},(3)是,(4)不是;3.(1)不是,(2)不是, (3)不是,(4)是;4。(1)fy→ーyー1=s,(2)g!y→+ 2y-3=. 5.1(2)1.(2)y=fx)={_2,r<0:2.(2)y=0.6,xe红,2, 4,5,6},(3)y∈{0.6,1.2,1.8,2.4,3.0,3.6}. 习题5.2(1)1.(1)非,(2)奇,(3)偶,(4)偶,(5)奇,6)非: 2.b=0是偶函数,不能是奇函数。· 习题6.2(2)2.(1)一4≤x≤4,(2)1≤y≤5,(3)偶函数,(4)有界函数. 习题6.2(8)2.(3)减函数;3.(1)x≠0,y≠0,(2)奇函数,(3)减函数。 习题B.81.(1)x≠0,y≠1,(2)无奇偶性,减函数,无界、习题5.41.(1)≈3.2,(2)≈4.6; 2.(1)0,±2.8,(2)2.5; 4.(1)x≥0,y≥0,(2)不具有奇偶性,增函数,有下界,y=0是它的一个下 界,但无上界;5.(1)[-6,2),(2)(-1,1]U[2,3). 习题5.5011.y=是+1,z+0,g1;2.y=√-1,x∈R, VE욱3。ー1,이+,이+;:4。ッにーV+,付>-1) {yy<0};5.y=x2+4x+3,{xx≥-2},{yy≥-1;6.y=(x+1)月∈R,y∈R. 习题6.61.(1)y=±V4-x,(2)y=士Vx2-4,(3)y=士V2+4;3,(1)∈R,y∈[-1,+∞),反函数①y=-x+I,x∈[-1,+∞),y∈(-∞,0],②y=x+1,x∈[-1,+∞),y∈0,+∞), ②)r(-0,0)U(0,+0),y∈(1,+∞),反函数①y=-√-7 x∈(1,+∞),y∈(-∞,0), 1 @y=√ix(1,+∞,∈(0,+o). ●448● ==========第454页========== 复习题五A 4 函 数 类 型 定义域 值 域 (1)划=g2+÷1 有期整函数 R R 阅=+号 有理分函数 -o,0)U0,÷c∞){(-co,-2]U[2,+0∞) ()y=1十、/21 花理两教 -o,-1门[1,小心) [1,÷∞) 1 (4)yー1 龙理函数 fe.÷心) -1,十∞) 5. 题次 定 义 域 : 减 奇偶性: 有界性 减性 x<0时减(1) [i,o∞) 有下界 (o心,fo) 偶 y=1 >0时抛 (2) (-∞,-3)(-…1,0)-,0U0,∞) 无 无 减 .(2) 题次 反此数的解折弋 定义域 值 域 (i) (1,÷co) (-的,0) (i) 侣-° (-o,0)U(0,十o) (-∞,-1)U(-1,4) 7.(0)专호y-1我-合, (2)-√3sVW3, f(c)0. 复习题五B 01:··(·)( 1 +o∞),(2)y=1+ーも・定叉域∞,-5J]U(ー4,の 04440 ==========第455页========== [1,+∞)月2.(1)y=-1+V15+2c-e,-3≤c≤5,-1≤y≤3,y=ー1-√15+2x-2,-3<<5,-5≤y≤-1,(2)y=1+Va+2:-24,x≤一6或x≥4,y≥1,y=1√x2+2-24,x≤-6或c≥4,y≤1; 4.(1)定义域[0,2],零值点(0,0),(2,0),y大=f(1)=1.在区间(0,1)上递增,在区间(1,2)上递减,(2)定义域(一∞,+∞),偶函数,零值点(士2,0),(±VΣ,0),y极小=f(士V3)=-1,y大=f(0)=8,在区间(-,一√3)上递减,在区间(一√3,0)上递增,在区间(0,√3)上递减,在区间(√3+∞)上递增;5.(1)是增函数,(3)x∈(f(4),f(b), (4)也是增函数. 第五章测验题 1.((④y=1-r或y-号+3,非单值对应,因此不能,@=1+立,单值对应,所以可站2.d定义蚊[-1,,值城受,当=0时y4=字(@)定义城(-0,+∞,值城(-,2-Y,当一亭时ya= 2-3.0)=V,定ヌ城似아,城コ! (2)y=V2-2c+2,定义域{x{≥1},值域{ylg≥1};4.(1)c*0, 1- ,(x<0), (2)y=2-1,(0<<2),(3)函数的值域是y>0,它是无界函数,函数 是, 1 在区间(一∞,0)上递增,在区间(0,2)上递减,在区间(2,+∞)上递增,当=2时函数有极小值y=0;5.(1)定义域(-∞,+∞),值域(1,2],偶函数,在区间(一∞,0)上递增,在区间(0,十∞)上递减,当x=0时y欧=2. 第六章 习题611.(子,图易2.(a0.10日②35月, >,如果a>1, 〔1, (3)w=心,如果a=1, (4)a=a如果a=1, <,如果0a如果00时,(2)x>1y1时;5.(1)x>10,(2)x<-10, (<-V西或V画,(国0<品或>0,向>, (6)01,(8)a<1;3.(1)x>,(2)心<划, (3)>y,(4)xy,当x<0或x>1时1<;2.(1)x<1域c>3,(②)11,则x<1或>3,若0<<1则1-1,(4)全体实数;5.(2)2,3, (3)2,3,(4)-3,-2,(-1,0,(⑥)0,1;6.(1)m>,(2)m0,@)=-1,=4时ニ,一青いー1>4时。10,0<<4时习题7.311.②)4,4:-子6》-5;2.(1)2,(3)2,3, 3.(1)2.4771,(2)0.7597,(3)3.322,(4)2.262;4.(1)5,(3)9。习题7.3(2)1.(1)3,(2)3,·(3)1,(4)-2; 2.(1)2,(2)3; 3.(品,1,(②)0,量4.0,②1,3 习题7.4(1)1.(1)9,(2)300,(3)333,(4)V3;2.(1)√, (2)25;3.(1)5,(2)6,14,(3)√2±1,(4)100,1000: 4.(1)7,-1,(2)5,4. 习题7.42)1.(④10,(2)六,丽,3)10,动1 1(4)10, i2.()a,16,2)55,露1 习题7.51.≈-1.7,2;2.≈-0.8,2,4;3.≈2.8;4.≈1.8 习题7.61.(12,1,(②1000,10,(3)10,10,(④,-7 2.(1)2,4,(2)5,3; 3.(1)100,10,(2)13,11. 习题7.71.(④2<<3,②)-号<<子(3)>lg2,(④520号园2<3,国1<2. ◆480 ==========第459页========== 复习题七A 1.(8)as1時Jタマ2,0にべくユー克22 2.(3)a>1时 <0,0<1,0-1时-,고,3.(1)4>1, ②)0≤u1,(8)011,4m1:6.()-1,3号,(2》同解,()同解,7.()-1号2,②司解,()f()0,(0时才同解; 8.(1)1,0.2031,(2)3,(3)37,(4)4;9.4年;10.10年; 11.(1)17,9,2)10,4,4,10,(3)1,1,2,4,(4)1G,25,25,16: 2.()<品成,2欲多或心,(③)x0,国>2 复习题七B 1.(1)734+G/33,(2)-2<-V√3,或V32 第七章测验题 1.()0<<1,(②0<(0<1,(④>05 2.(1)x>0,(②)(-∞,0)U(0,1)U(1,+∞): 3.(1)在区河(一∞,-1)上递减,区间(2,+)上递增,区间[-1,2].上不增不减,(2)在区间(-,)上递减,在区可(-2,+)上递增; 4.土1,土2,山3; 5.(1)±1(2)±. ·448· ==========第460页========== 第八章 习题8.1(1)1.(2)2,4,6,,2%,…,(4)13,23,33,…,n3, 2)-1-1,-1,-1,-1,④子--告吾 3.(1)10,50,(2)+1,6, (-) 3484.(1)99,980,(2)ao-120,45=-25. 习题8.1(3)2.(1)有穷,递增,有界,(2)无穷,递增,有界,(3)无穷,常数,有界,(4)有穷,摆动,有界,(5)无穷,摆动,无界; 3.=8品(无宽,递照,有职习题83<1)1.d4-1,②15-%⑧)号-号则 2.(1)67, (a 1l, (3)-6, ()33.()릉,품,-,(3),+운, p+1 0,+20;4.(山-12②)0,(3)05.(8,2②)±6,±8, 9+1 ±10,或±10,士8,士6(四解). 习题8.2X2))1.(山2,5848,②)22,9期.(8)34,-10哈,(④4140, (5)7,28,(6)0.1,51,(7)-38,-360,(8)-45,45,(9)4.5,10,(10)26,-7;2.(1)2+2%,(2)6m-3;3.(1)122.5≈123(米), (2)20.0秒;4.180下;5.6秒;6.4条;7.(1)25,(2)1: 8.(1)10,2.5,19,(2)5,5,6. 习题8.3(1)1.(8)3-2V②,(W区-1)n-3,(4《a-b),-b)-1 a+b’(au+b)n-8i 2.1)(3)2916,3.2)±3,(④子:4.()%5.(②160,80,40,20,10,6.-16,2或1,2:7.≈15.47升;8.≈25%.习题8.3(2)2.()96,189,2②)7,15g(3)4,1,(④3,2186,或-3,1094,(5)3,18,或-4,32,(6)-2,10,(7)3584,7161,(8)27,8, (9)3,6,(10)3,-1;3.(1)1,2,10,(2)1,3,44.192盏; 5.(1)2048平方厘米,(2)4092平方厘米. 习题8.4(1)1.(3)5,(4)a2+b2;2.(3)±2,(4)±a2-b2; 3.3,5,7;4.3cm,4cn1,5cm,直角三角形;6.(1)2,4,8,(2)1,8,9,或-1,3,-9,(3)1,4,16. ●450· ==========第461页========== 日题8.4(2)1.3,5,7;2.17,10,3或8,10,12;3.45,15,5或5,15,45;4.12,16,20,25;5.5,12,19,26. 习题8611四(-少2-1,因”装是, 2.()n2,(2)n2+1 %+1· 习题8日1+②)者u1Dm+2片2.1)寻-, ②青4u+12-4.0,②a+1(2+. 复习题八A 2.(4)第7项;3.(1)一14,(2)-32;4.四15,②)c0星:5.四 1,3,5,(②1,2,生7.-22,6,18;9.4)a1=a-”, 1-a 2 (②)a1-c)+-1 1-(y-1)・ 复习题八B 3.66个,63,4.(1)壹2-m+2,(②)505;5.255星6.2,10,50,7. (1)(+1)(2л-+1)+1-, (2)3-3-,4.3 35,④(-1少船是+3 163r+6 第八章测验题 1.a10=41,S10=230;2.1=1,g=2,S10=1023;4.x=8,y=4; 54v市(中:因回到)-L.2 第九章 习题911.②女m-是:2(-1]-0,习题.21.(1)1-号,1-(侵,1-(侵),,1-(合)》,,②)这个数列是递增数列,又是有界数列,所以它一定有极限:2.)1,(②)一手 04510 ==========第462页========== 员④间导,@4经00,⑧3.5,1.8 习题9.32.)号(④2,(⑤3号,(⑥是V6,5+E2 e+1;3.,,嘉感动丽 4 1 4.d号,(②)9,(3③)1.28,(④7分,(⑤)23或48,(⑥)18家54 5.(1)4(2+V√②)a,(2)2a;6.12米. 习题941.1)÷@器)器④15锡句品@3号990 2e:3.(少0.465,(2-0.01,(8)0.230,(④10. -2可(②8,“7()m8.= 1 习题9.51.(1)4n= 2.四8-音0++8宝@)成-g-法21 2n,lim8、 =2。 复习题九A 2.(1)有,(2)没有,(3)没有,(4)有;3.(1)71,(2)334; 4.@)六,④3;5.是②)1,(3)8,④-1: 6.a2②号:&.d字a京9.6,@4 10.(1)2x,(2)4R3. 复习题九B 2.(3)1001;3.(1)a≥b时为a,a-6;8.(1)-√60,<2y0,减-2,-,--1(2)프4 4 时y>0,<ご軍成でご时y0,ー5V 4 4 时 =0,=-蛋时数5是:14.(2)(i)奇,(ii)偶,(iii)非, (i)非,()偶,(i)奇;17.(2)不是; 18.(2)y=-2+2V2小-, -1≤a≤2,-2y・1; y=-2-22+m-,-1<0<2,-50时ek,k<时즉 (2)a>0时y大=7a,a<0时y大=3a;15.(1)无解, (2)(c,y)∈{(1,1),(1,-1),(25,5),(36,-6)};16.(1)n=21, (2)n=9,8≈13.45,18.21.(④)寻,(②)22.49 总测验题 1.(1){0},(2){0,1;2.(1)(-3,1),(2)(-3,-1]: 4.(1)y=3-2(-2),(2)ー3+3,3,>ー2,5定又 一∞1时2<<3,当03; 7.①1210,②)3,32,9.四约421米,(②7次:10.山)寻因子 ●454· ==========第465页==========